Contenu | Rechercher | Menus

Annonce

Si vous avez des soucis pour rester connecté, déconnectez-vous puis reconnectez-vous depuis ce lien en cochant la case
Me connecter automatiquement lors de mes prochaines visites.

À propos de l'équipe du forum.

#1 Le 19/09/2016, à 20:59

goliath60

un démarrage qui dure 25 min [résolu]

bonsoir,

je crois que j'ai cliqué sur un truc qu'il  ne fallait pas...

il y a deux jour, la mise à jour automatique m'indique que des mises à jours sont à faire.
je fini ce que je faisait, je fermes les pages là sur le bureau une fenêtre avec le bouton "install"
sans prendre la peine de lire ce qui allait être installé, je clic, l'installation est relativement longue, 10 minutes environ, et pendant ces 10 min je comprends que je n'avais pas été sur l'onglet des mises à jours, trop tard, je laisse finir.

le problème c'est que maintenant, l'allumage de l'ordi ou un redémarrage dur 25 min

l'ordinateur démarre normalement, le logo avec la marque de l'ordi apparaît, puis une page violette (couleur de fond de ubuntu), puis écran noir, l'écran est en mode veille (bouton d'allumage orange), le ventilo de latour est allumé, on entend le disque dur tourner.

il n'y a pas de cd dans le lecteur.

donc:
y a t il une solution à ce problème?

Ubuntu 14.04 LTS

merci beaucoup

Dernière modification par goliath60 (Le 22/09/2016, à 17:21)

Hors ligne

#2 Le 20/09/2016, à 07:49

malbo

Re : un démarrage qui dure 25 min [résolu]

goliath60 a écrit :

il y a deux jour, la mise à jour automatique m'indique que des mises à jours sont à faire.

Est-ce que ce ne serait pas le pop-up dont je parle dans ce post : http://forum.ubuntu-fr.org/viewtopic.ph … #p21593546

Hors ligne

#3 Le 20/09/2016, à 17:27

goliath60

Re : un démarrage qui dure 25 min [résolu]

bonjour,

a vrai dire, ça y ressemble, mais je n'en suis pas sur.
j'ai bien vu que symbole de la mise à jour sue la gauche, j'ai fermé la fenêtre sur laquelle je travaillait, j'ai cliqué sur install,
c'est après que je me suis rendu compte.

il n'empêche, que un redémarrage dur 25 min.

je sais que c'est pas top au niveau des informations, mais, une solution?

Hors ligne

#4 Le 20/09/2016, à 17:50

malbo

Re : un démarrage qui dure 25 min [résolu]

Je te prie de passer la commande suivante dans un terminal (ne tape pas cette commande : utilise le copier-coller) :

cat /var/log/apt/history.log

Tu colleras le résultat obtenu dans ta réponse en suivant attentivement les indications de ce post : http://forum.ubuntu-fr.org/viewtopic.php?id=1614731

Dernière modification par malbo (Le 20/09/2016, à 17:52)

Hors ligne

#5 Le 20/09/2016, à 19:40

goliath60

Re : un démarrage qui dure 25 min [résolu]

Start-Date: 2016-09-17  20:20:25
Commandline: apt-get install my-weather-indicator
Install: python3-tz:amd64 (2012c-1ubuntu0.1, automatic), python3-dateutil:amd64 (2.0+dfsg1-1, automatic), my-weather-indicator:amd64 (0.8.1-0extras16.04.1), gir1.2-json-1.0:amd64 (0.16.2-1ubuntu1, automatic), geoclue-hostip:amd64 (0.12.99-3ubuntu1, automatic), python3-requests-oauthlib:amd64 (0.4.0-1, automatic), libgeocode-glib0:amd64 (3.10.0-1ubuntu1, automatic), gir1.2-geocodeglib-1.0:amd64 (3.10.0-1ubuntu1, automatic)
End-Date: 2016-09-17  20:20:40

Start-Date: 2016-09-17  20:32:59
Commandline: aptdaemon role='role-commit-packages' sender=':1.65'
Install: linux-headers-3.19.0-68:amd64 (3.19.0-68.76~14.04.1), xserver-xorg-video-mach64-lts-xenial:amd64 (6.9.5-1build2~trusty1, automatic), linux-headers-4.4.0-36-generic:amd64 (4.4.0-36.55~14.04.1, automatic), libdrm-amdgpu1:amd64 (2.4.67-1ubuntu0.14.04.1), xserver-xorg-video-all-lts-xenial:amd64 (7.7+13ubuntu3~trusty2, automatic), xserver-xorg-video-amdgpu-lts-xenial:amd64 (1.1.0-1~trusty1, automatic), xserver-xorg-video-cirrus-lts-xenial:amd64 (1.5.3-1ubuntu3~trusty1, automatic), xserver-xorg-video-radeon-lts-xenial:amd64 (7.7.0-1~trusty2, automatic), xserver-xorg-video-mga-lts-xenial:amd64 (1.6.4-1build2~trusty1, automatic), xserver-xorg-video-trident-lts-xenial:amd64 (1.3.7-1build2~trusty1, automatic), xserver-xorg-video-fbdev-lts-xenial:amd64 (0.4.4-1build5~trusty1, automatic), xserver-xorg-video-tdfx-lts-xenial:amd64 (1.4.6-1build2~trusty1, automatic), linux-image-4.4.0-36-generic:amd64 (4.4.0-36.55~14.04.1, automatic), libglapi-mesa-lts-xenial:amd64 (11.2.0-1ubuntu2~trusty1, automatic), xserver-xorg-video-ati-lts-xenial:amd64 (7.7.0-1~trusty2, automatic), libwayland-egl1-mesa-lts-xenial:amd64 (11.2.0-1ubuntu2~trusty1), xserver-xorg-input-all-lts-xenial:amd64 (7.7+13ubuntu3~trusty2), linux-headers-4.4.0-36:amd64 (4.4.0-36.55~14.04.1, automatic), xserver-xorg-video-neomagic-lts-xenial:amd64 (1.2.9-1build2~trusty1, automatic), xserver-xorg-video-savage-lts-xenial:amd64 (2.3.8-1ubuntu3~trusty1, automatic), linux-generic-lts-xenial:amd64 (4.4.0.36.26), libegl1-mesa-lts-xenial:amd64 (11.2.0-1ubuntu2~trusty1), xserver-xorg-input-vmmouse-lts-xenial:amd64 (13.1.0-1ubuntu2~trusty1, automatic), xserver-xorg-input-evdev-lts-xenial:amd64 (2.10.1-1ubuntu2~trusty1, automatic), libgles2-mesa-lts-xenial:amd64 (11.2.0-1ubuntu2~trusty1), linux-headers-generic-lts-xenial:amd64 (4.4.0.36.26, automatic), xserver-xorg-video-vesa-lts-xenial:amd64 (2.3.4-1build2~trusty1, automatic), linux-image-extra-4.4.0-36-generic:amd64 (4.4.0-36.55~14.04.1, automatic), xserver-xorg-video-siliconmotion-lts-xenial:amd64 (1.7.8-1ubuntu6~trusty1, automatic), libxatracker2-lts-xenial:amd64 (11.2.0-1ubuntu2~trusty1), xserver-xorg-input-wacom-lts-xenial:amd64 (0.32.0-0ubuntu3~trusty1, automatic), linux-image-extra-3.19.0-68-generic:amd64 (3.19.0-68.76~14.04.1), xserver-xorg-video-nouveau-lts-xenial:amd64 (1.0.12-1build2~trusty1, automatic), linux-image-3.19.0-68-generic:amd64 (3.19.0-68.76~14.04.1), xserver-xorg-video-openchrome-lts-xenial:amd64 (0.3.3+git20160310-1~trusty1, automatic), xserver-xorg-video-qxl-lts-xenial:amd64 (0.1.4-3ubuntu3~trusty1, automatic), libgbm1-lts-xenial:amd64 (11.2.0-1ubuntu2~trusty1, automatic), xserver-xorg-video-intel-lts-xenial:amd64 (2.99.917+git20160325-1ubuntu1~trusty1, automatic), xserver-xorg-video-sisusb-lts-xenial:amd64 (0.9.6-2build5~trusty1, automatic), libgl1-mesa-dri-lts-xenial:amd64 (11.2.0-1ubuntu2~trusty1, automatic), libgl1-mesa-glx-lts-xenial:amd64 (11.2.0-1ubuntu2~trusty1), linux-image-generic-lts-xenial:amd64 (4.4.0.36.26, automatic), xserver-xorg-video-vmware-lts-xenial:amd64 (13.1.0-2ubuntu3~trusty1, automatic), linux-headers-3.19.0-68-generic:amd64 (3.19.0-68.76~14.04.1), xserver-xorg-core-lts-xenial:amd64 (1.18.3-1ubuntu2.2~trusty3), xserver-xorg-video-r128-lts-xenial:amd64 (6.10.0-1build2~trusty1, automatic), libgles1-mesa-lts-xenial:amd64 (11.2.0-1ubuntu2~trusty1), xserver-xorg-lts-xenial:amd64 (7.7+13ubuntu3~trusty2), xserver-xorg-input-synaptics-lts-xenial:amd64 (1.8.2-1ubuntu3~trusty1, automatic), libllvm3.8v4:amd64 (3.8-2ubuntu3~trusty4, automatic)
Upgrade: update-manager-core:amd64 (0.196.19, 0.196.21), oxideqt-codecs-extra:amd64 (1.16.5-0ubuntu0.14.04.1, 1.17.7-0ubuntu0.14.04.1), isc-dhcp-common:amd64 (4.2.4-7ubuntu12.5, 4.2.4-7ubuntu12.6), whoopsie:amd64 (0.2.24.6ubuntu2, 0.2.24.6ubuntu3), eog:amd64 (3.10.2-0ubuntu5.1, 3.10.2-0ubuntu5.2), libharfbuzz-icu0:amd64 (0.9.27-1ubuntu1, 0.9.27-1ubuntu1.1), libupnp6:amd64 (1.6.17-1.2, 1.6.17-1.2+deb7u1build0.14.04.1), python3-update-manager:amd64 (0.196.19, 0.196.21), chromium-codecs-ffmpeg-extra:amd64 (51.0.2704.79-0ubuntu0.14.04.1.1121, 52.0.2743.116-0ubuntu0.14.04.1.1134), flashplugin-installer:amd64 (11.2.202.632ubuntu0.14.04.1, 11.2.202.635ubuntu0.14.04.1), linux-image-generic-lts-vivid:amd64 (3.19.0.66.48, 3.19.0.68.50), libidn11:amd64 (1.28-1ubuntu2, 1.28-1ubuntu2.1), curl:amd64 (7.35.0-1ubuntu2.8, 7.35.0-1ubuntu2.9), libharfbuzz0b:amd64 (0.9.27-1ubuntu1, 0.9.27-1ubuntu1.1), gnome-contacts:amd64 (3.8.3-1ubuntu1, 3.8.3-1ubuntu1.1), lsb-base:amd64 (4.1+Debian11ubuntu6.1, 4.1+Debian11ubuntu6.2), libwhoopsie0:amd64 (0.2.24.6ubuntu2, 0.2.24.6ubuntu3), liboxideqtcore0:amd64 (1.16.5-0ubuntu0.14.04.1, 1.17.7-0ubuntu0.14.04.1), gnome-keyring:amd64 (3.10.1-1ubuntu4.2, 3.10.1-1ubuntu4.3), linux-headers-generic-lts-vivid:amd64 (3.19.0.66.48, 3.19.0.68.50), lsb-release:amd64 (4.1+Debian11ubuntu6.1, 4.1+Debian11ubuntu6.2), isc-dhcp-client:amd64 (4.2.4-7ubuntu12.5, 4.2.4-7ubuntu12.6), linux-generic-lts-vivid:amd64 (3.19.0.66.48, 3.19.0.68.50), libpam-gnome-keyring:amd64 (3.10.1-1ubuntu4.2, 3.10.1-1ubuntu4.3), libcurl3:amd64 (7.35.0-1ubuntu2.8, 7.35.0-1ubuntu2.9), liboxideqtquick0:amd64 (1.16.5-0ubuntu0.14.04.1, 1.17.7-0ubuntu0.14.04.1), libp11-kit-gnome-keyring:amd64 (3.10.1-1ubuntu4.2, 3.10.1-1ubuntu4.3), liboxideqt-qmlplugin:amd64 (1.16.5-0ubuntu0.14.04.1, 1.17.7-0ubuntu0.14.04.1), linux-libc-dev:amd64 (3.13.0-93.140, 3.13.0-95.142), update-manager:amd64 (0.196.19, 0.196.21), file-roller:amd64 (3.10.2.1-0ubuntu4.1, 3.10.2.1-0ubuntu4.2), libcurl3-gnutls:amd64 (7.35.0-1ubuntu2.8, 7.35.0-1ubuntu2.9)
Remove: xserver-xorg-video-neomagic-lts-vivid:amd64 (1.2.8-1ubuntu1~trusty1), libgl1-mesa-dri-lts-vivid:amd64 (10.5.9-2ubuntu1~trusty2), libgl1-mesa-glx-lts-vivid:amd64 (10.5.9-2ubuntu1~trusty2), libglapi-mesa-lts-vivid:amd64 (10.5.9-2ubuntu1~trusty2), xserver-xorg-input-mouse-lts-vivid:amd64 (1.9.1-1~trusty1), xserver-xorg-video-sisusb-lts-vivid:amd64 (0.9.6-2build3~trusty1), libgles1-mesa-lts-vivid:amd64 (10.5.9-2ubuntu1~trusty2), xserver-xorg-input-synaptics-lts-vivid:amd64 (1.8.1-1ubuntu1~trusty1), xserver-xorg-video-nouveau-lts-vivid:amd64 (1.0.11-1ubuntu2build1~trusty1), xserver-xorg-video-all-lts-vivid:amd64 (7.7+7ubuntu3~trusty1), xserver-xorg-input-vmmouse-lts-vivid:amd64 (13.0.0-1ubuntu1~trusty1), xserver-xorg-lts-vivid:amd64 (7.7+7ubuntu3~trusty1), xserver-xorg-input-wacom-lts-vivid:amd64 (0.25.0-0ubuntu1.1~trusty1), xserver-xorg-video-radeon-lts-vivid:amd64 (7.5.0-1ubuntu2~trusty1), xserver-xorg-video-r128-lts-vivid:amd64 (6.9.2-1ubuntu1~trusty1), xserver-xorg-video-tdfx-lts-vivid:amd64 (1.4.6-0ubuntu1~trusty1), xserver-xorg-video-mach64-lts-vivid:amd64 (6.9.4-2ubuntu1~trusty1), libwayland-egl1-mesa-lts-vivid:amd64 (10.5.9-2ubuntu1~trusty2), xserver-xorg-video-siliconmotion-lts-vivid:amd64 (1.7.7-2ubuntu2~trusty1), xserver-xorg-video-savage-lts-vivid:amd64 (2.3.7-2ubuntu4~trusty1), libgbm1-lts-vivid:amd64 (10.5.9-2ubuntu1~trusty2), xserver-xorg-input-all-lts-vivid:amd64 (7.7+7ubuntu3~trusty1), xserver-xorg-video-vmware-lts-vivid:amd64 (13.1.0-0ubuntu1build1~trusty1), xserver-xorg-video-ati-lts-vivid:amd64 (7.5.0-1ubuntu2~trusty1), xserver-xorg-input-evdev-lts-vivid:amd64 (2.9.0-1ubuntu2~trusty1), libgles2-mesa-lts-vivid:amd64 (10.5.9-2ubuntu1~trusty2), xserver-xorg-core-lts-vivid:amd64 (1.17.1-0ubuntu3.1~trusty1.1), xserver-xorg-video-intel-lts-vivid:amd64 (2.99.917-1~exp1ubuntu2.2~trusty1), xserver-xorg-video-vesa-lts-vivid:amd64 (2.3.3-1build3~trusty1), xserver-xorg-video-trident-lts-vivid:amd64 (1.3.6-0ubuntu6build1~trusty1), libegl1-mesa-lts-vivid:amd64 (10.5.9-2ubuntu1~trusty2), xserver-xorg-video-mga-lts-vivid:amd64 (1.6.3-2ubuntu1~trusty1), xserver-xorg-video-openchrome-lts-vivid:amd64 (0.3.3-1ubuntu1~trusty1), xserver-xorg-video-cirrus-lts-vivid:amd64 (1.5.2-2ubuntu1~trusty1), libxatracker2-lts-vivid:amd64 (10.5.9-2ubuntu1~trusty2), xserver-xorg-video-fbdev-lts-vivid:amd64 (0.4.4-1build3~trusty1)
End-Date: 2016-09-17  20:38:13

Start-Date: 2016-09-19  17:33:31
Commandline: aptdaemon role='role-commit-packages' sender=':1.107'
Remove: firefox-locale-en:amd64 (48.0+build2-0ubuntu0.14.04.1), thunderbird-locale-en-us:amd64 (45.2.0+build1-0ubuntu0.14.04.3), thunderbird-locale-en:amd64 (45.2.0+build1-0ubuntu0.14.04.3), language-pack-gnome-en:amd64 (14.04+20160720), gimp-help-en:amd64 (2.6.1-1), language-pack-en-base:amd64 (14.04+20160720), language-pack-gnome-en-base:amd64 (14.04+20160720), language-pack-en:amd64 (14.04+20160720)
End-Date: 2016-09-19  17:33:43

Start-Date: 2016-09-19  19:33:36
Commandline: apt-get remove my-weather-indicator
Remove: my-weather-indicator:amd64 (0.8.1-0extras16.04.1)
End-Date: 2016-09-19  19:33:44

Hors ligne

#6 Le 20/09/2016, à 20:58

malbo

Re : un démarrage qui dure 25 min [résolu]

Oui, ça s'est passé le 17 sept 2016 aux environs de 20H30 : il y a eu installation du HWE stack de Xenial (ce sont tous les paquets qui se terminent par lts-xenial, tel le paquet linux-image-generic-lts-xenial) et désinstallation d'un certain nombre de paquets qui se terminent par lts-vivid. Il s'agit donc bien de ce que j'ai mentionné dans le post #2.
Je te prie de faire un Boot-Info depuis ta session du Ubuntu installé et de donner l'URL de ce Boot-Info dans ta réponse.

Hors ligne

#7 Le 20/09/2016, à 22:31

goliath60

Re : un démarrage qui dure 25 min [résolu]

voici le résultat demandé:

http://paste.ubuntu.com/23208697/

Hors ligne

#8 Le 21/09/2016, à 05:55

malbo

Re : un démarrage qui dure 25 min [résolu]

Ton Boot-Info est là :

 Boot Info Script cfd9efe + Boot-Repair extra info      [Boot-Info 26Apr2016]


============================= Boot Info Summary: ===============================

 => Grub2 (v2.00) is installed in the MBR of /dev/sda and looks at sector 1 of 
    the same hard drive for core.img. core.img is at this location and looks 
    for (,msdos1)/boot/grub. It also embeds following components:
    
    modules
    ---------------------------------------------------------------------------
    fshelp ext2 part_msdos biosdisk
    ---------------------------------------------------------------------------
 => libparted MBR boot code is installed in the MBR of /dev/sdb.

sda1: __________________________________________________________________________

    File system:       ext4
    Boot sector type:  -
    Boot sector info: 
    Operating System:  Ubuntu 14.04.5 LTS
    Boot files:        /boot/grub/grub.cfg /etc/fstab 
                       /boot/grub/i386-pc/core.img

sda2: __________________________________________________________________________

    File system:       Extended Partition
    Boot sector type:  -
    Boot sector info: 

sda5: __________________________________________________________________________

    File system:       swap
    Boot sector type:  -
    Boot sector info: 

sdb2: __________________________________________________________________________

    File system:       ntfs
    Boot sector type:  Windows Vista: NTFS
    Boot sector info:  No errors found in the Boot Parameter Block.
    Operating System:  
    Boot files:        

============================ Drive/Partition Info: =============================

Drive: sda _____________________________________________________________________

Disk /dev/sda: 1000.2 GB, 1000204886016 bytes
255 heads, 63 sectors/track, 121601 cylinders, total 1953525168 sectors
Units = sectors of 1 * 512 = 512 bytes
Sector size (logical/physical): 512 bytes / 4096 bytes

Partition  Boot  Start Sector    End Sector  # of Sectors  Id System

/dev/sda1    *          2,048 1,921,662,975 1,921,660,928  83 Linux
/dev/sda2       1,921,665,022 1,953,523,711    31,858,690   5 Extended
/dev/sda5       1,921,665,024 1,953,523,711    31,858,688  82 Linux swap / Solaris


Drive: sdb _____________________________________________________________________

Disk /dev/sdb: 1000.2 GB, 1000204886016 bytes
255 heads, 63 sectors/track, 121601 cylinders, total 1953525168 sectors
Units = sectors of 1 * 512 = 512 bytes
Sector size (logical/physical): 512 bytes / 512 bytes

Partition  Boot  Start Sector    End Sector  # of Sectors  Id System

/dev/sdb2               2,048 1,953,521,663 1,953,519,616   7 NTFS / exFAT / HPFS


"blkid" output: ________________________________________________________________

Device           UUID                                   TYPE       LABEL

/dev/sda1        496e9007-632f-49ee-a0ce-7f2a5eb1a540   ext4       
/dev/sda5        4dc511c8-8d84-43fc-83f7-23488cbba3fa   swap       
/dev/sdb2        6509CCA977639C2C                       ntfs       

========================= "ls -l /dev/disk/by-id" output: ======================

total 0
lrwxrwxrwx 1 root root  9 Sep 19 19:46 ata-ASUS_DRW-24F1ST_a_S13A68EG300173 -> ../../sr0
lrwxrwxrwx 1 root root  9 Sep 20 22:27 ata-WDC_WD1002FBYS-18A6B0_WD-WMATV3361500 -> ../../sdb
lrwxrwxrwx 1 root root 10 Sep 19 19:46 ata-WDC_WD1002FBYS-18A6B0_WD-WMATV3361500-part2 -> ../../sdb2
lrwxrwxrwx 1 root root  9 Sep 20 22:27 ata-WDC_WD10EZEX-00BN5A0_WD-WCC3F6HLC90R -> ../../sda
lrwxrwxrwx 1 root root 10 Sep 19 19:46 ata-WDC_WD10EZEX-00BN5A0_WD-WCC3F6HLC90R-part1 -> ../../sda1
lrwxrwxrwx 1 root root 10 Sep 19 19:46 ata-WDC_WD10EZEX-00BN5A0_WD-WCC3F6HLC90R-part2 -> ../../sda2
lrwxrwxrwx 1 root root 10 Sep 19 19:46 ata-WDC_WD10EZEX-00BN5A0_WD-WCC3F6HLC90R-part5 -> ../../sda5
lrwxrwxrwx 1 root root  9 Sep 19 19:46 usb-Generic_STORAGE_DEVICE_000000000903-0:0 -> ../../sdc
lrwxrwxrwx 1 root root  9 Sep 20 22:27 wwn-0x50014ee056ef363b -> ../../sdb
lrwxrwxrwx 1 root root 10 Sep 19 19:46 wwn-0x50014ee056ef363b-part2 -> ../../sdb2
lrwxrwxrwx 1 root root  9 Sep 20 22:27 wwn-0x50014ee2617c8b1a -> ../../sda
lrwxrwxrwx 1 root root 10 Sep 19 19:46 wwn-0x50014ee2617c8b1a-part1 -> ../../sda1
lrwxrwxrwx 1 root root 10 Sep 19 19:46 wwn-0x50014ee2617c8b1a-part2 -> ../../sda2
lrwxrwxrwx 1 root root 10 Sep 19 19:46 wwn-0x50014ee2617c8b1a-part5 -> ../../sda5

================================ Mount points: =================================

Device           Mount_Point              Type       Options

/dev/sda1        /                        ext4       (rw,errors=remount-ro)
/dev/sdb2        /media/vadee/6509CCA977639C2C fuseblk    (rw,nosuid,nodev,allow_other,default_permissions,blksize=4096)


=========================== sda1/boot/grub/grub.cfg: ===========================

--------------------------------------------------------------------------------
#
# DO NOT EDIT THIS FILE
#
# It is automatically generated by grub-mkconfig using templates
# from /etc/grub.d and settings from /etc/default/grub
#

### BEGIN /etc/grub.d/00_header ###
if [ -s $prefix/grubenv ]; then
  set have_grubenv=true
  load_env
fi
if [ "${next_entry}" ] ; then
   set default="${next_entry}"
   set next_entry=
   save_env next_entry
   set boot_once=true
else
   set default="0"
fi

if [ x"${feature_menuentry_id}" = xy ]; then
  menuentry_id_option="--id"
else
  menuentry_id_option=""
fi

export menuentry_id_option

if [ "${prev_saved_entry}" ]; then
  set saved_entry="${prev_saved_entry}"
  save_env saved_entry
  set prev_saved_entry=
  save_env prev_saved_entry
  set boot_once=true
fi

function savedefault {
  if [ -z "${boot_once}" ]; then
    saved_entry="${chosen}"
    save_env saved_entry
  fi
}
function recordfail {
  set recordfail=1
  if [ -n "${have_grubenv}" ]; then if [ -z "${boot_once}" ]; then save_env recordfail; fi; fi
}
function load_video {
  if [ x$feature_all_video_module = xy ]; then
    insmod all_video
  else
    insmod efi_gop
    insmod efi_uga
    insmod ieee1275_fb
    insmod vbe
    insmod vga
    insmod video_bochs
    insmod video_cirrus
  fi
}

if [ x$feature_default_font_path = xy ] ; then
   font=unicode
else
insmod part_msdos
insmod ext2
set root='hd0,msdos1'
if [ x$feature_platform_search_hint = xy ]; then
  search --no-floppy --fs-uuid --set=root --hint-bios=hd0,msdos1 --hint-efi=hd0,msdos1 --hint-baremetal=ahci0,msdos1  496e9007-632f-49ee-a0ce-7f2a5eb1a540
else
  search --no-floppy --fs-uuid --set=root 496e9007-632f-49ee-a0ce-7f2a5eb1a540
fi
    font="/usr/share/grub/unicode.pf2"
fi

if loadfont $font ; then
  set gfxmode=auto
  load_video
  insmod gfxterm
  set locale_dir=$prefix/locale
  set lang=fr_FR
  insmod gettext
fi
terminal_output gfxterm
if [ "${recordfail}" = 1 ] ; then
  set timeout=30
else
  if [ x$feature_timeout_style = xy ] ; then
    set timeout_style=hidden
    set timeout=0
  # Fallback hidden-timeout code in case the timeout_style feature is
  # unavailable.
  elif sleep --interruptible 0 ; then
    set timeout=0
  fi
fi
### END /etc/grub.d/00_header ###

### BEGIN /etc/grub.d/05_debian_theme ###
set menu_color_normal=white/black
set menu_color_highlight=black/light-gray
if background_color 44,0,30,0; then
  clear
fi
### END /etc/grub.d/05_debian_theme ###

### BEGIN /etc/grub.d/10_linux ###
function gfxmode {
	set gfxpayload="${1}"
	if [ "${1}" = "keep" ]; then
		set vt_handoff=vt.handoff=7
	else
		set vt_handoff=
	fi
}
if [ "${recordfail}" != 1 ]; then
  if [ -e ${prefix}/gfxblacklist.txt ]; then
    if hwmatch ${prefix}/gfxblacklist.txt 3; then
      if [ ${match} = 0 ]; then
        set linux_gfx_mode=keep
      else
        set linux_gfx_mode=text
      fi
    else
      set linux_gfx_mode=text
    fi
  else
    set linux_gfx_mode=keep
  fi
else
  set linux_gfx_mode=text
fi
export linux_gfx_mode
menuentry 'Ubuntu' --class ubuntu --class gnu-linux --class gnu --class os $menuentry_id_option 'gnulinux-simple-496e9007-632f-49ee-a0ce-7f2a5eb1a540' {
	recordfail
	load_video
	gfxmode $linux_gfx_mode
	insmod gzio
	insmod part_msdos
	insmod ext2
	set root='hd0,msdos1'
	if [ x$feature_platform_search_hint = xy ]; then
	  search --no-floppy --fs-uuid --set=root --hint-bios=hd0,msdos1 --hint-efi=hd0,msdos1 --hint-baremetal=ahci0,msdos1  496e9007-632f-49ee-a0ce-7f2a5eb1a540
	else
	  search --no-floppy --fs-uuid --set=root 496e9007-632f-49ee-a0ce-7f2a5eb1a540
	fi
	linux	/boot/vmlinuz-4.4.0-36-generic root=UUID=496e9007-632f-49ee-a0ce-7f2a5eb1a540 ro  quiet splash $vt_handoff
	initrd	/boot/initrd.img-4.4.0-36-generic
}
submenu 'Options avancées pour Ubuntu' $menuentry_id_option 'gnulinux-advanced-496e9007-632f-49ee-a0ce-7f2a5eb1a540' {
	menuentry 'Ubuntu, avec Linux 4.4.0-36-generic' --class ubuntu --class gnu-linux --class gnu --class os $menuentry_id_option 'gnulinux-4.4.0-36-generic-advanced-496e9007-632f-49ee-a0ce-7f2a5eb1a540' {
		recordfail
		load_video
		gfxmode $linux_gfx_mode
		insmod gzio
		insmod part_msdos
		insmod ext2
		set root='hd0,msdos1'
		if [ x$feature_platform_search_hint = xy ]; then
		  search --no-floppy --fs-uuid --set=root --hint-bios=hd0,msdos1 --hint-efi=hd0,msdos1 --hint-baremetal=ahci0,msdos1  496e9007-632f-49ee-a0ce-7f2a5eb1a540
		else
		  search --no-floppy --fs-uuid --set=root 496e9007-632f-49ee-a0ce-7f2a5eb1a540
		fi
		echo	'Chargement de Linux 4.4.0-36-generic…'
		linux	/boot/vmlinuz-4.4.0-36-generic root=UUID=496e9007-632f-49ee-a0ce-7f2a5eb1a540 ro  quiet splash $vt_handoff
		echo	'Chargement du disque mémoire initial…'
		initrd	/boot/initrd.img-4.4.0-36-generic
	}
	menuentry 'Ubuntu, with Linux 4.4.0-36-generic (recovery mode)' --class ubuntu --class gnu-linux --class gnu --class os $menuentry_id_option 'gnulinux-4.4.0-36-generic-recovery-496e9007-632f-49ee-a0ce-7f2a5eb1a540' {
		recordfail
		load_video
		insmod gzio
		insmod part_msdos
		insmod ext2
		set root='hd0,msdos1'
		if [ x$feature_platform_search_hint = xy ]; then
		  search --no-floppy --fs-uuid --set=root --hint-bios=hd0,msdos1 --hint-efi=hd0,msdos1 --hint-baremetal=ahci0,msdos1  496e9007-632f-49ee-a0ce-7f2a5eb1a540
		else
		  search --no-floppy --fs-uuid --set=root 496e9007-632f-49ee-a0ce-7f2a5eb1a540
		fi
		echo	'Chargement de Linux 4.4.0-36-generic…'
		linux	/boot/vmlinuz-4.4.0-36-generic root=UUID=496e9007-632f-49ee-a0ce-7f2a5eb1a540 ro recovery nomodeset 
		echo	'Chargement du disque mémoire initial…'
		initrd	/boot/initrd.img-4.4.0-36-generic
	}
	menuentry 'Ubuntu, avec Linux 3.19.0-68-generic' --class ubuntu --class gnu-linux --class gnu --class os $menuentry_id_option 'gnulinux-3.19.0-68-generic-advanced-496e9007-632f-49ee-a0ce-7f2a5eb1a540' {
		recordfail
		load_video
		gfxmode $linux_gfx_mode
		insmod gzio
		insmod part_msdos
		insmod ext2
		set root='hd0,msdos1'
		if [ x$feature_platform_search_hint = xy ]; then
		  search --no-floppy --fs-uuid --set=root --hint-bios=hd0,msdos1 --hint-efi=hd0,msdos1 --hint-baremetal=ahci0,msdos1  496e9007-632f-49ee-a0ce-7f2a5eb1a540
		else
		  search --no-floppy --fs-uuid --set=root 496e9007-632f-49ee-a0ce-7f2a5eb1a540
		fi
		echo	'Chargement de Linux 3.19.0-68-generic…'
		linux	/boot/vmlinuz-3.19.0-68-generic root=UUID=496e9007-632f-49ee-a0ce-7f2a5eb1a540 ro  quiet splash $vt_handoff
		echo	'Chargement du disque mémoire initial…'
		initrd	/boot/initrd.img-3.19.0-68-generic
	}
	menuentry 'Ubuntu, with Linux 3.19.0-68-generic (recovery mode)' --class ubuntu --class gnu-linux --class gnu --class os $menuentry_id_option 'gnulinux-3.19.0-68-generic-recovery-496e9007-632f-49ee-a0ce-7f2a5eb1a540' {
		recordfail
		load_video
		insmod gzio
		insmod part_msdos
		insmod ext2
		set root='hd0,msdos1'
		if [ x$feature_platform_search_hint = xy ]; then
		  search --no-floppy --fs-uuid --set=root --hint-bios=hd0,msdos1 --hint-efi=hd0,msdos1 --hint-baremetal=ahci0,msdos1  496e9007-632f-49ee-a0ce-7f2a5eb1a540
		else
		  search --no-floppy --fs-uuid --set=root 496e9007-632f-49ee-a0ce-7f2a5eb1a540
		fi
		echo	'Chargement de Linux 3.19.0-68-generic…'
		linux	/boot/vmlinuz-3.19.0-68-generic root=UUID=496e9007-632f-49ee-a0ce-7f2a5eb1a540 ro recovery nomodeset 
		echo	'Chargement du disque mémoire initial…'
		initrd	/boot/initrd.img-3.19.0-68-generic
	}
	menuentry 'Ubuntu, avec Linux 3.19.0-66-generic' --class ubuntu --class gnu-linux --class gnu --class os $menuentry_id_option 'gnulinux-3.19.0-66-generic-advanced-496e9007-632f-49ee-a0ce-7f2a5eb1a540' {
		recordfail
		load_video
		gfxmode $linux_gfx_mode
		insmod gzio
		insmod part_msdos
		insmod ext2
		set root='hd0,msdos1'
		if [ x$feature_platform_search_hint = xy ]; then
		  search --no-floppy --fs-uuid --set=root --hint-bios=hd0,msdos1 --hint-efi=hd0,msdos1 --hint-baremetal=ahci0,msdos1  496e9007-632f-49ee-a0ce-7f2a5eb1a540
		else
		  search --no-floppy --fs-uuid --set=root 496e9007-632f-49ee-a0ce-7f2a5eb1a540
		fi
		echo	'Chargement de Linux 3.19.0-66-generic…'
		linux	/boot/vmlinuz-3.19.0-66-generic root=UUID=496e9007-632f-49ee-a0ce-7f2a5eb1a540 ro  quiet splash $vt_handoff
		echo	'Chargement du disque mémoire initial…'
		initrd	/boot/initrd.img-3.19.0-66-generic
	}
	menuentry 'Ubuntu, with Linux 3.19.0-66-generic (recovery mode)' --class ubuntu --class gnu-linux --class gnu --class os $menuentry_id_option 'gnulinux-3.19.0-66-generic-recovery-496e9007-632f-49ee-a0ce-7f2a5eb1a540' {
		recordfail
		load_video
		insmod gzio
		insmod part_msdos
		insmod ext2
		set root='hd0,msdos1'
		if [ x$feature_platform_search_hint = xy ]; then
		  search --no-floppy --fs-uuid --set=root --hint-bios=hd0,msdos1 --hint-efi=hd0,msdos1 --hint-baremetal=ahci0,msdos1  496e9007-632f-49ee-a0ce-7f2a5eb1a540
		else
		  search --no-floppy --fs-uuid --set=root 496e9007-632f-49ee-a0ce-7f2a5eb1a540
		fi
		echo	'Chargement de Linux 3.19.0-66-generic…'
		linux	/boot/vmlinuz-3.19.0-66-generic root=UUID=496e9007-632f-49ee-a0ce-7f2a5eb1a540 ro recovery nomodeset 
		echo	'Chargement du disque mémoire initial…'
		initrd	/boot/initrd.img-3.19.0-66-generic
	}
	menuentry 'Ubuntu, avec Linux 3.19.0-65-generic' --class ubuntu --class gnu-linux --class gnu --class os $menuentry_id_option 'gnulinux-3.19.0-65-generic-advanced-496e9007-632f-49ee-a0ce-7f2a5eb1a540' {
		recordfail
		load_video
		gfxmode $linux_gfx_mode
		insmod gzio
		insmod part_msdos
		insmod ext2
		set root='hd0,msdos1'
		if [ x$feature_platform_search_hint = xy ]; then
		  search --no-floppy --fs-uuid --set=root --hint-bios=hd0,msdos1 --hint-efi=hd0,msdos1 --hint-baremetal=ahci0,msdos1  496e9007-632f-49ee-a0ce-7f2a5eb1a540
		else
		  search --no-floppy --fs-uuid --set=root 496e9007-632f-49ee-a0ce-7f2a5eb1a540
		fi
		echo	'Chargement de Linux 3.19.0-65-generic…'
		linux	/boot/vmlinuz-3.19.0-65-generic root=UUID=496e9007-632f-49ee-a0ce-7f2a5eb1a540 ro  quiet splash $vt_handoff
		echo	'Chargement du disque mémoire initial…'
		initrd	/boot/initrd.img-3.19.0-65-generic
	}
	menuentry 'Ubuntu, with Linux 3.19.0-65-generic (recovery mode)' --class ubuntu --class gnu-linux --class gnu --class os $menuentry_id_option 'gnulinux-3.19.0-65-generic-recovery-496e9007-632f-49ee-a0ce-7f2a5eb1a540' {
		recordfail
		load_video
		insmod gzio
		insmod part_msdos
		insmod ext2
		set root='hd0,msdos1'
		if [ x$feature_platform_search_hint = xy ]; then
		  search --no-floppy --fs-uuid --set=root --hint-bios=hd0,msdos1 --hint-efi=hd0,msdos1 --hint-baremetal=ahci0,msdos1  496e9007-632f-49ee-a0ce-7f2a5eb1a540
		else
		  search --no-floppy --fs-uuid --set=root 496e9007-632f-49ee-a0ce-7f2a5eb1a540
		fi
		echo	'Chargement de Linux 3.19.0-65-generic…'
		linux	/boot/vmlinuz-3.19.0-65-generic root=UUID=496e9007-632f-49ee-a0ce-7f2a5eb1a540 ro recovery nomodeset 
		echo	'Chargement du disque mémoire initial…'
		initrd	/boot/initrd.img-3.19.0-65-generic
	}
	menuentry 'Ubuntu, avec Linux 3.19.0-64-generic' --class ubuntu --class gnu-linux --class gnu --class os $menuentry_id_option 'gnulinux-3.19.0-64-generic-advanced-496e9007-632f-49ee-a0ce-7f2a5eb1a540' {
		recordfail
		load_video
		gfxmode $linux_gfx_mode
		insmod gzio
		insmod part_msdos
		insmod ext2
		set root='hd0,msdos1'
		if [ x$feature_platform_search_hint = xy ]; then
		  search --no-floppy --fs-uuid --set=root --hint-bios=hd0,msdos1 --hint-efi=hd0,msdos1 --hint-baremetal=ahci0,msdos1  496e9007-632f-49ee-a0ce-7f2a5eb1a540
		else
		  search --no-floppy --fs-uuid --set=root 496e9007-632f-49ee-a0ce-7f2a5eb1a540
		fi
		echo	'Chargement de Linux 3.19.0-64-generic…'
		linux	/boot/vmlinuz-3.19.0-64-generic root=UUID=496e9007-632f-49ee-a0ce-7f2a5eb1a540 ro  quiet splash $vt_handoff
		echo	'Chargement du disque mémoire initial…'
		initrd	/boot/initrd.img-3.19.0-64-generic
	}
	menuentry 'Ubuntu, with Linux 3.19.0-64-generic (recovery mode)' --class ubuntu --class gnu-linux --class gnu --class os $menuentry_id_option 'gnulinux-3.19.0-64-generic-recovery-496e9007-632f-49ee-a0ce-7f2a5eb1a540' {
		recordfail
		load_video
		insmod gzio
		insmod part_msdos
		insmod ext2
		set root='hd0,msdos1'
		if [ x$feature_platform_search_hint = xy ]; then
		  search --no-floppy --fs-uuid --set=root --hint-bios=hd0,msdos1 --hint-efi=hd0,msdos1 --hint-baremetal=ahci0,msdos1  496e9007-632f-49ee-a0ce-7f2a5eb1a540
		else
		  search --no-floppy --fs-uuid --set=root 496e9007-632f-49ee-a0ce-7f2a5eb1a540
		fi
		echo	'Chargement de Linux 3.19.0-64-generic…'
		linux	/boot/vmlinuz-3.19.0-64-generic root=UUID=496e9007-632f-49ee-a0ce-7f2a5eb1a540 ro recovery nomodeset 
		echo	'Chargement du disque mémoire initial…'
		initrd	/boot/initrd.img-3.19.0-64-generic
	}
	menuentry 'Ubuntu, avec Linux 3.19.0-61-generic' --class ubuntu --class gnu-linux --class gnu --class os $menuentry_id_option 'gnulinux-3.19.0-61-generic-advanced-496e9007-632f-49ee-a0ce-7f2a5eb1a540' {
		recordfail
		load_video
		gfxmode $linux_gfx_mode
		insmod gzio
		insmod part_msdos
		insmod ext2
		set root='hd0,msdos1'
		if [ x$feature_platform_search_hint = xy ]; then
		  search --no-floppy --fs-uuid --set=root --hint-bios=hd0,msdos1 --hint-efi=hd0,msdos1 --hint-baremetal=ahci0,msdos1  496e9007-632f-49ee-a0ce-7f2a5eb1a540
		else
		  search --no-floppy --fs-uuid --set=root 496e9007-632f-49ee-a0ce-7f2a5eb1a540
		fi
		echo	'Chargement de Linux 3.19.0-61-generic…'
		linux	/boot/vmlinuz-3.19.0-61-generic root=UUID=496e9007-632f-49ee-a0ce-7f2a5eb1a540 ro  quiet splash $vt_handoff
		echo	'Chargement du disque mémoire initial…'
		initrd	/boot/initrd.img-3.19.0-61-generic
	}
	menuentry 'Ubuntu, with Linux 3.19.0-61-generic (recovery mode)' --class ubuntu --class gnu-linux --class gnu --class os $menuentry_id_option 'gnulinux-3.19.0-61-generic-recovery-496e9007-632f-49ee-a0ce-7f2a5eb1a540' {
		recordfail
		load_video
		insmod gzio
		insmod part_msdos
		insmod ext2
		set root='hd0,msdos1'
		if [ x$feature_platform_search_hint = xy ]; then
		  search --no-floppy --fs-uuid --set=root --hint-bios=hd0,msdos1 --hint-efi=hd0,msdos1 --hint-baremetal=ahci0,msdos1  496e9007-632f-49ee-a0ce-7f2a5eb1a540
		else
		  search --no-floppy --fs-uuid --set=root 496e9007-632f-49ee-a0ce-7f2a5eb1a540
		fi
		echo	'Chargement de Linux 3.19.0-61-generic…'
		linux	/boot/vmlinuz-3.19.0-61-generic root=UUID=496e9007-632f-49ee-a0ce-7f2a5eb1a540 ro recovery nomodeset 
		echo	'Chargement du disque mémoire initial…'
		initrd	/boot/initrd.img-3.19.0-61-generic
	}
	menuentry 'Ubuntu, avec Linux 3.19.0-59-generic' --class ubuntu --class gnu-linux --class gnu --class os $menuentry_id_option 'gnulinux-3.19.0-59-generic-advanced-496e9007-632f-49ee-a0ce-7f2a5eb1a540' {
		recordfail
		load_video
		gfxmode $linux_gfx_mode
		insmod gzio
		insmod part_msdos
		insmod ext2
		set root='hd0,msdos1'
		if [ x$feature_platform_search_hint = xy ]; then
		  search --no-floppy --fs-uuid --set=root --hint-bios=hd0,msdos1 --hint-efi=hd0,msdos1 --hint-baremetal=ahci0,msdos1  496e9007-632f-49ee-a0ce-7f2a5eb1a540
		else
		  search --no-floppy --fs-uuid --set=root 496e9007-632f-49ee-a0ce-7f2a5eb1a540
		fi
		echo	'Chargement de Linux 3.19.0-59-generic…'
		linux	/boot/vmlinuz-3.19.0-59-generic root=UUID=496e9007-632f-49ee-a0ce-7f2a5eb1a540 ro  quiet splash $vt_handoff
		echo	'Chargement du disque mémoire initial…'
		initrd	/boot/initrd.img-3.19.0-59-generic
	}
	menuentry 'Ubuntu, with Linux 3.19.0-59-generic (recovery mode)' --class ubuntu --class gnu-linux --class gnu --class os $menuentry_id_option 'gnulinux-3.19.0-59-generic-recovery-496e9007-632f-49ee-a0ce-7f2a5eb1a540' {
		recordfail
		load_video
		insmod gzio
		insmod part_msdos
		insmod ext2
		set root='hd0,msdos1'
		if [ x$feature_platform_search_hint = xy ]; then
		  search --no-floppy --fs-uuid --set=root --hint-bios=hd0,msdos1 --hint-efi=hd0,msdos1 --hint-baremetal=ahci0,msdos1  496e9007-632f-49ee-a0ce-7f2a5eb1a540
		else
		  search --no-floppy --fs-uuid --set=root 496e9007-632f-49ee-a0ce-7f2a5eb1a540
		fi
		echo	'Chargement de Linux 3.19.0-59-generic…'
		linux	/boot/vmlinuz-3.19.0-59-generic root=UUID=496e9007-632f-49ee-a0ce-7f2a5eb1a540 ro recovery nomodeset 
		echo	'Chargement du disque mémoire initial…'
		initrd	/boot/initrd.img-3.19.0-59-generic
	}
	menuentry 'Ubuntu, avec Linux 3.19.0-58-generic' --class ubuntu --class gnu-linux --class gnu --class os $menuentry_id_option 'gnulinux-3.19.0-58-generic-advanced-496e9007-632f-49ee-a0ce-7f2a5eb1a540' {
		recordfail
		load_video
		gfxmode $linux_gfx_mode
		insmod gzio
		insmod part_msdos
		insmod ext2
		set root='hd0,msdos1'
		if [ x$feature_platform_search_hint = xy ]; then
		  search --no-floppy --fs-uuid --set=root --hint-bios=hd0,msdos1 --hint-efi=hd0,msdos1 --hint-baremetal=ahci0,msdos1  496e9007-632f-49ee-a0ce-7f2a5eb1a540
		else
		  search --no-floppy --fs-uuid --set=root 496e9007-632f-49ee-a0ce-7f2a5eb1a540
		fi
		echo	'Chargement de Linux 3.19.0-58-generic…'
		linux	/boot/vmlinuz-3.19.0-58-generic root=UUID=496e9007-632f-49ee-a0ce-7f2a5eb1a540 ro  quiet splash $vt_handoff
		echo	'Chargement du disque mémoire initial…'
		initrd	/boot/initrd.img-3.19.0-58-generic
	}
	menuentry 'Ubuntu, with Linux 3.19.0-58-generic (recovery mode)' --class ubuntu --class gnu-linux --class gnu --class os $menuentry_id_option 'gnulinux-3.19.0-58-generic-recovery-496e9007-632f-49ee-a0ce-7f2a5eb1a540' {
		recordfail
		load_video
		insmod gzio
		insmod part_msdos
		insmod ext2
		set root='hd0,msdos1'
		if [ x$feature_platform_search_hint = xy ]; then
		  search --no-floppy --fs-uuid --set=root --hint-bios=hd0,msdos1 --hint-efi=hd0,msdos1 --hint-baremetal=ahci0,msdos1  496e9007-632f-49ee-a0ce-7f2a5eb1a540
		else
		  search --no-floppy --fs-uuid --set=root 496e9007-632f-49ee-a0ce-7f2a5eb1a540
		fi
		echo	'Chargement de Linux 3.19.0-58-generic…'
		linux	/boot/vmlinuz-3.19.0-58-generic root=UUID=496e9007-632f-49ee-a0ce-7f2a5eb1a540 ro recovery nomodeset 
		echo	'Chargement du disque mémoire initial…'
		initrd	/boot/initrd.img-3.19.0-58-generic
	}
	menuentry 'Ubuntu, avec Linux 3.19.0-56-generic' --class ubuntu --class gnu-linux --class gnu --class os $menuentry_id_option 'gnulinux-3.19.0-56-generic-advanced-496e9007-632f-49ee-a0ce-7f2a5eb1a540' {
		recordfail
		load_video
		gfxmode $linux_gfx_mode
		insmod gzio
		insmod part_msdos
		insmod ext2
		set root='hd0,msdos1'
		if [ x$feature_platform_search_hint = xy ]; then
		  search --no-floppy --fs-uuid --set=root --hint-bios=hd0,msdos1 --hint-efi=hd0,msdos1 --hint-baremetal=ahci0,msdos1  496e9007-632f-49ee-a0ce-7f2a5eb1a540
		else
		  search --no-floppy --fs-uuid --set=root 496e9007-632f-49ee-a0ce-7f2a5eb1a540
		fi
		echo	'Chargement de Linux 3.19.0-56-generic…'
		linux	/boot/vmlinuz-3.19.0-56-generic root=UUID=496e9007-632f-49ee-a0ce-7f2a5eb1a540 ro  quiet splash $vt_handoff
		echo	'Chargement du disque mémoire initial…'
		initrd	/boot/initrd.img-3.19.0-56-generic
	}
	menuentry 'Ubuntu, with Linux 3.19.0-56-generic (recovery mode)' --class ubuntu --class gnu-linux --class gnu --class os $menuentry_id_option 'gnulinux-3.19.0-56-generic-recovery-496e9007-632f-49ee-a0ce-7f2a5eb1a540' {
		recordfail
		load_video
		insmod gzio
		insmod part_msdos
		insmod ext2
		set root='hd0,msdos1'
		if [ x$feature_platform_search_hint = xy ]; then
		  search --no-floppy --fs-uuid --set=root --hint-bios=hd0,msdos1 --hint-efi=hd0,msdos1 --hint-baremetal=ahci0,msdos1  496e9007-632f-49ee-a0ce-7f2a5eb1a540
		else
		  search --no-floppy --fs-uuid --set=root 496e9007-632f-49ee-a0ce-7f2a5eb1a540
		fi
		echo	'Chargement de Linux 3.19.0-56-generic…'
		linux	/boot/vmlinuz-3.19.0-56-generic root=UUID=496e9007-632f-49ee-a0ce-7f2a5eb1a540 ro recovery nomodeset 
		echo	'Chargement du disque mémoire initial…'
		initrd	/boot/initrd.img-3.19.0-56-generic
	}
	menuentry 'Ubuntu, avec Linux 3.19.0-51-generic' --class ubuntu --class gnu-linux --class gnu --class os $menuentry_id_option 'gnulinux-3.19.0-51-generic-advanced-496e9007-632f-49ee-a0ce-7f2a5eb1a540' {
		recordfail
		load_video
		gfxmode $linux_gfx_mode
		insmod gzio
		insmod part_msdos
		insmod ext2
		set root='hd0,msdos1'
		if [ x$feature_platform_search_hint = xy ]; then
		  search --no-floppy --fs-uuid --set=root --hint-bios=hd0,msdos1 --hint-efi=hd0,msdos1 --hint-baremetal=ahci0,msdos1  496e9007-632f-49ee-a0ce-7f2a5eb1a540
		else
		  search --no-floppy --fs-uuid --set=root 496e9007-632f-49ee-a0ce-7f2a5eb1a540
		fi
		echo	'Chargement de Linux 3.19.0-51-generic…'
		linux	/boot/vmlinuz-3.19.0-51-generic root=UUID=496e9007-632f-49ee-a0ce-7f2a5eb1a540 ro  quiet splash $vt_handoff
		echo	'Chargement du disque mémoire initial…'
		initrd	/boot/initrd.img-3.19.0-51-generic
	}
	menuentry 'Ubuntu, with Linux 3.19.0-51-generic (recovery mode)' --class ubuntu --class gnu-linux --class gnu --class os $menuentry_id_option 'gnulinux-3.19.0-51-generic-recovery-496e9007-632f-49ee-a0ce-7f2a5eb1a540' {
		recordfail
		load_video
		insmod gzio
		insmod part_msdos
		insmod ext2
		set root='hd0,msdos1'
		if [ x$feature_platform_search_hint = xy ]; then
		  search --no-floppy --fs-uuid --set=root --hint-bios=hd0,msdos1 --hint-efi=hd0,msdos1 --hint-baremetal=ahci0,msdos1  496e9007-632f-49ee-a0ce-7f2a5eb1a540
		else
		  search --no-floppy --fs-uuid --set=root 496e9007-632f-49ee-a0ce-7f2a5eb1a540
		fi
		echo	'Chargement de Linux 3.19.0-51-generic…'
		linux	/boot/vmlinuz-3.19.0-51-generic root=UUID=496e9007-632f-49ee-a0ce-7f2a5eb1a540 ro recovery nomodeset 
		echo	'Chargement du disque mémoire initial…'
		initrd	/boot/initrd.img-3.19.0-51-generic
	}
	menuentry 'Ubuntu, avec Linux 3.19.0-49-generic' --class ubuntu --class gnu-linux --class gnu --class os $menuentry_id_option 'gnulinux-3.19.0-49-generic-advanced-496e9007-632f-49ee-a0ce-7f2a5eb1a540' {
		recordfail
		load_video
		gfxmode $linux_gfx_mode
		insmod gzio
		insmod part_msdos
		insmod ext2
		set root='hd0,msdos1'
		if [ x$feature_platform_search_hint = xy ]; then
		  search --no-floppy --fs-uuid --set=root --hint-bios=hd0,msdos1 --hint-efi=hd0,msdos1 --hint-baremetal=ahci0,msdos1  496e9007-632f-49ee-a0ce-7f2a5eb1a540
		else
		  search --no-floppy --fs-uuid --set=root 496e9007-632f-49ee-a0ce-7f2a5eb1a540
		fi
		echo	'Chargement de Linux 3.19.0-49-generic…'
		linux	/boot/vmlinuz-3.19.0-49-generic root=UUID=496e9007-632f-49ee-a0ce-7f2a5eb1a540 ro  quiet splash $vt_handoff
		echo	'Chargement du disque mémoire initial…'
		initrd	/boot/initrd.img-3.19.0-49-generic
	}
	menuentry 'Ubuntu, with Linux 3.19.0-49-generic (recovery mode)' --class ubuntu --class gnu-linux --class gnu --class os $menuentry_id_option 'gnulinux-3.19.0-49-generic-recovery-496e9007-632f-49ee-a0ce-7f2a5eb1a540' {
		recordfail
		load_video
		insmod gzio
		insmod part_msdos
		insmod ext2
		set root='hd0,msdos1'
		if [ x$feature_platform_search_hint = xy ]; then
		  search --no-floppy --fs-uuid --set=root --hint-bios=hd0,msdos1 --hint-efi=hd0,msdos1 --hint-baremetal=ahci0,msdos1  496e9007-632f-49ee-a0ce-7f2a5eb1a540
		else
		  search --no-floppy --fs-uuid --set=root 496e9007-632f-49ee-a0ce-7f2a5eb1a540
		fi
		echo	'Chargement de Linux 3.19.0-49-generic…'
		linux	/boot/vmlinuz-3.19.0-49-generic root=UUID=496e9007-632f-49ee-a0ce-7f2a5eb1a540 ro recovery nomodeset 
		echo	'Chargement du disque mémoire initial…'
		initrd	/boot/initrd.img-3.19.0-49-generic
	}
	menuentry 'Ubuntu, avec Linux 3.19.0-47-generic' --class ubuntu --class gnu-linux --class gnu --class os $menuentry_id_option 'gnulinux-3.19.0-47-generic-advanced-496e9007-632f-49ee-a0ce-7f2a5eb1a540' {
		recordfail
		load_video
		gfxmode $linux_gfx_mode
		insmod gzio
		insmod part_msdos
		insmod ext2
		set root='hd0,msdos1'
		if [ x$feature_platform_search_hint = xy ]; then
		  search --no-floppy --fs-uuid --set=root --hint-bios=hd0,msdos1 --hint-efi=hd0,msdos1 --hint-baremetal=ahci0,msdos1  496e9007-632f-49ee-a0ce-7f2a5eb1a540
		else
		  search --no-floppy --fs-uuid --set=root 496e9007-632f-49ee-a0ce-7f2a5eb1a540
		fi
		echo	'Chargement de Linux 3.19.0-47-generic…'
		linux	/boot/vmlinuz-3.19.0-47-generic root=UUID=496e9007-632f-49ee-a0ce-7f2a5eb1a540 ro  quiet splash $vt_handoff
		echo	'Chargement du disque mémoire initial…'
		initrd	/boot/initrd.img-3.19.0-47-generic
	}
	menuentry 'Ubuntu, with Linux 3.19.0-47-generic (recovery mode)' --class ubuntu --class gnu-linux --class gnu --class os $menuentry_id_option 'gnulinux-3.19.0-47-generic-recovery-496e9007-632f-49ee-a0ce-7f2a5eb1a540' {
		recordfail
		load_video
		insmod gzio
		insmod part_msdos
		insmod ext2
		set root='hd0,msdos1'
		if [ x$feature_platform_search_hint = xy ]; then
		  search --no-floppy --fs-uuid --set=root --hint-bios=hd0,msdos1 --hint-efi=hd0,msdos1 --hint-baremetal=ahci0,msdos1  496e9007-632f-49ee-a0ce-7f2a5eb1a540
		else
		  search --no-floppy --fs-uuid --set=root 496e9007-632f-49ee-a0ce-7f2a5eb1a540
		fi
		echo	'Chargement de Linux 3.19.0-47-generic…'
		linux	/boot/vmlinuz-3.19.0-47-generic root=UUID=496e9007-632f-49ee-a0ce-7f2a5eb1a540 ro recovery nomodeset 
		echo	'Chargement du disque mémoire initial…'
		initrd	/boot/initrd.img-3.19.0-47-generic
	}
	menuentry 'Ubuntu, avec Linux 3.19.0-43-generic' --class ubuntu --class gnu-linux --class gnu --class os $menuentry_id_option 'gnulinux-3.19.0-43-generic-advanced-496e9007-632f-49ee-a0ce-7f2a5eb1a540' {
		recordfail
		load_video
		gfxmode $linux_gfx_mode
		insmod gzio
		insmod part_msdos
		insmod ext2
		set root='hd0,msdos1'
		if [ x$feature_platform_search_hint = xy ]; then
		  search --no-floppy --fs-uuid --set=root --hint-bios=hd0,msdos1 --hint-efi=hd0,msdos1 --hint-baremetal=ahci0,msdos1  496e9007-632f-49ee-a0ce-7f2a5eb1a540
		else
		  search --no-floppy --fs-uuid --set=root 496e9007-632f-49ee-a0ce-7f2a5eb1a540
		fi
		echo	'Chargement de Linux 3.19.0-43-generic…'
		linux	/boot/vmlinuz-3.19.0-43-generic root=UUID=496e9007-632f-49ee-a0ce-7f2a5eb1a540 ro  quiet splash $vt_handoff
		echo	'Chargement du disque mémoire initial…'
		initrd	/boot/initrd.img-3.19.0-43-generic
	}
	menuentry 'Ubuntu, with Linux 3.19.0-43-generic (recovery mode)' --class ubuntu --class gnu-linux --class gnu --class os $menuentry_id_option 'gnulinux-3.19.0-43-generic-recovery-496e9007-632f-49ee-a0ce-7f2a5eb1a540' {
		recordfail
		load_video
		insmod gzio
		insmod part_msdos
		insmod ext2
		set root='hd0,msdos1'
		if [ x$feature_platform_search_hint = xy ]; then
		  search --no-floppy --fs-uuid --set=root --hint-bios=hd0,msdos1 --hint-efi=hd0,msdos1 --hint-baremetal=ahci0,msdos1  496e9007-632f-49ee-a0ce-7f2a5eb1a540
		else
		  search --no-floppy --fs-uuid --set=root 496e9007-632f-49ee-a0ce-7f2a5eb1a540
		fi
		echo	'Chargement de Linux 3.19.0-43-generic…'
		linux	/boot/vmlinuz-3.19.0-43-generic root=UUID=496e9007-632f-49ee-a0ce-7f2a5eb1a540 ro recovery nomodeset 
		echo	'Chargement du disque mémoire initial…'
		initrd	/boot/initrd.img-3.19.0-43-generic
	}
	menuentry 'Ubuntu, avec Linux 3.19.0-42-generic' --class ubuntu --class gnu-linux --class gnu --class os $menuentry_id_option 'gnulinux-3.19.0-42-generic-advanced-496e9007-632f-49ee-a0ce-7f2a5eb1a540' {
		recordfail
		load_video
		gfxmode $linux_gfx_mode
		insmod gzio
		insmod part_msdos
		insmod ext2
		set root='hd0,msdos1'
		if [ x$feature_platform_search_hint = xy ]; then
		  search --no-floppy --fs-uuid --set=root --hint-bios=hd0,msdos1 --hint-efi=hd0,msdos1 --hint-baremetal=ahci0,msdos1  496e9007-632f-49ee-a0ce-7f2a5eb1a540
		else
		  search --no-floppy --fs-uuid --set=root 496e9007-632f-49ee-a0ce-7f2a5eb1a540
		fi
		echo	'Chargement de Linux 3.19.0-42-generic…'
		linux	/boot/vmlinuz-3.19.0-42-generic root=UUID=496e9007-632f-49ee-a0ce-7f2a5eb1a540 ro  quiet splash $vt_handoff
		echo	'Chargement du disque mémoire initial…'
		initrd	/boot/initrd.img-3.19.0-42-generic
	}
	menuentry 'Ubuntu, with Linux 3.19.0-42-generic (recovery mode)' --class ubuntu --class gnu-linux --class gnu --class os $menuentry_id_option 'gnulinux-3.19.0-42-generic-recovery-496e9007-632f-49ee-a0ce-7f2a5eb1a540' {
		recordfail
		load_video
		insmod gzio
		insmod part_msdos
		insmod ext2
		set root='hd0,msdos1'
		if [ x$feature_platform_search_hint = xy ]; then
		  search --no-floppy --fs-uuid --set=root --hint-bios=hd0,msdos1 --hint-efi=hd0,msdos1 --hint-baremetal=ahci0,msdos1  496e9007-632f-49ee-a0ce-7f2a5eb1a540
		else
		  search --no-floppy --fs-uuid --set=root 496e9007-632f-49ee-a0ce-7f2a5eb1a540
		fi
		echo	'Chargement de Linux 3.19.0-42-generic…'
		linux	/boot/vmlinuz-3.19.0-42-generic root=UUID=496e9007-632f-49ee-a0ce-7f2a5eb1a540 ro recovery nomodeset 
		echo	'Chargement du disque mémoire initial…'
		initrd	/boot/initrd.img-3.19.0-42-generic
	}
	menuentry 'Ubuntu, avec Linux 3.19.0-41-generic' --class ubuntu --class gnu-linux --class gnu --class os $menuentry_id_option 'gnulinux-3.19.0-41-generic-advanced-496e9007-632f-49ee-a0ce-7f2a5eb1a540' {
		recordfail
		load_video
		gfxmode $linux_gfx_mode
		insmod gzio
		insmod part_msdos
		insmod ext2
		set root='hd0,msdos1'
		if [ x$feature_platform_search_hint = xy ]; then
		  search --no-floppy --fs-uuid --set=root --hint-bios=hd0,msdos1 --hint-efi=hd0,msdos1 --hint-baremetal=ahci0,msdos1  496e9007-632f-49ee-a0ce-7f2a5eb1a540
		else
		  search --no-floppy --fs-uuid --set=root 496e9007-632f-49ee-a0ce-7f2a5eb1a540
		fi
		echo	'Chargement de Linux 3.19.0-41-generic…'
		linux	/boot/vmlinuz-3.19.0-41-generic root=UUID=496e9007-632f-49ee-a0ce-7f2a5eb1a540 ro  quiet splash $vt_handoff
		echo	'Chargement du disque mémoire initial…'
		initrd	/boot/initrd.img-3.19.0-41-generic
	}
	menuentry 'Ubuntu, with Linux 3.19.0-41-generic (recovery mode)' --class ubuntu --class gnu-linux --class gnu --class os $menuentry_id_option 'gnulinux-3.19.0-41-generic-recovery-496e9007-632f-49ee-a0ce-7f2a5eb1a540' {
		recordfail
		load_video
		insmod gzio
		insmod part_msdos
		insmod ext2
		set root='hd0,msdos1'
		if [ x$feature_platform_search_hint = xy ]; then
		  search --no-floppy --fs-uuid --set=root --hint-bios=hd0,msdos1 --hint-efi=hd0,msdos1 --hint-baremetal=ahci0,msdos1  496e9007-632f-49ee-a0ce-7f2a5eb1a540
		else
		  search --no-floppy --fs-uuid --set=root 496e9007-632f-49ee-a0ce-7f2a5eb1a540
		fi
		echo	'Chargement de Linux 3.19.0-41-generic…'
		linux	/boot/vmlinuz-3.19.0-41-generic root=UUID=496e9007-632f-49ee-a0ce-7f2a5eb1a540 ro recovery nomodeset 
		echo	'Chargement du disque mémoire initial…'
		initrd	/boot/initrd.img-3.19.0-41-generic
	}
	menuentry 'Ubuntu, avec Linux 3.19.0-39-generic' --class ubuntu --class gnu-linux --class gnu --class os $menuentry_id_option 'gnulinux-3.19.0-39-generic-advanced-496e9007-632f-49ee-a0ce-7f2a5eb1a540' {
		recordfail
		load_video
		gfxmode $linux_gfx_mode
		insmod gzio
		insmod part_msdos
		insmod ext2
		set root='hd0,msdos1'
		if [ x$feature_platform_search_hint = xy ]; then
		  search --no-floppy --fs-uuid --set=root --hint-bios=hd0,msdos1 --hint-efi=hd0,msdos1 --hint-baremetal=ahci0,msdos1  496e9007-632f-49ee-a0ce-7f2a5eb1a540
		else
		  search --no-floppy --fs-uuid --set=root 496e9007-632f-49ee-a0ce-7f2a5eb1a540
		fi
		echo	'Chargement de Linux 3.19.0-39-generic…'
		linux	/boot/vmlinuz-3.19.0-39-generic root=UUID=496e9007-632f-49ee-a0ce-7f2a5eb1a540 ro  quiet splash $vt_handoff
		echo	'Chargement du disque mémoire initial…'
		initrd	/boot/initrd.img-3.19.0-39-generic
	}
	menuentry 'Ubuntu, with Linux 3.19.0-39-generic (recovery mode)' --class ubuntu --class gnu-linux --class gnu --class os $menuentry_id_option 'gnulinux-3.19.0-39-generic-recovery-496e9007-632f-49ee-a0ce-7f2a5eb1a540' {
		recordfail
		load_video
		insmod gzio
		insmod part_msdos
		insmod ext2
		set root='hd0,msdos1'
		if [ x$feature_platform_search_hint = xy ]; then
		  search --no-floppy --fs-uuid --set=root --hint-bios=hd0,msdos1 --hint-efi=hd0,msdos1 --hint-baremetal=ahci0,msdos1  496e9007-632f-49ee-a0ce-7f2a5eb1a540
		else
		  search --no-floppy --fs-uuid --set=root 496e9007-632f-49ee-a0ce-7f2a5eb1a540
		fi
		echo	'Chargement de Linux 3.19.0-39-generic…'
		linux	/boot/vmlinuz-3.19.0-39-generic root=UUID=496e9007-632f-49ee-a0ce-7f2a5eb1a540 ro recovery nomodeset 
		echo	'Chargement du disque mémoire initial…'
		initrd	/boot/initrd.img-3.19.0-39-generic
	}
	menuentry 'Ubuntu, avec Linux 3.19.0-25-generic' --class ubuntu --class gnu-linux --class gnu --class os $menuentry_id_option 'gnulinux-3.19.0-25-generic-advanced-496e9007-632f-49ee-a0ce-7f2a5eb1a540' {
		recordfail
		load_video
		gfxmode $linux_gfx_mode
		insmod gzio
		insmod part_msdos
		insmod ext2
		set root='hd0,msdos1'
		if [ x$feature_platform_search_hint = xy ]; then
		  search --no-floppy --fs-uuid --set=root --hint-bios=hd0,msdos1 --hint-efi=hd0,msdos1 --hint-baremetal=ahci0,msdos1  496e9007-632f-49ee-a0ce-7f2a5eb1a540
		else
		  search --no-floppy --fs-uuid --set=root 496e9007-632f-49ee-a0ce-7f2a5eb1a540
		fi
		echo	'Chargement de Linux 3.19.0-25-generic…'
		linux	/boot/vmlinuz-3.19.0-25-generic root=UUID=496e9007-632f-49ee-a0ce-7f2a5eb1a540 ro  quiet splash $vt_handoff
		echo	'Chargement du disque mémoire initial…'
		initrd	/boot/initrd.img-3.19.0-25-generic
	}
	menuentry 'Ubuntu, with Linux 3.19.0-25-generic (recovery mode)' --class ubuntu --class gnu-linux --class gnu --class os $menuentry_id_option 'gnulinux-3.19.0-25-generic-recovery-496e9007-632f-49ee-a0ce-7f2a5eb1a540' {
		recordfail
		load_video
		insmod gzio
		insmod part_msdos
		insmod ext2
		set root='hd0,msdos1'
		if [ x$feature_platform_search_hint = xy ]; then
		  search --no-floppy --fs-uuid --set=root --hint-bios=hd0,msdos1 --hint-efi=hd0,msdos1 --hint-baremetal=ahci0,msdos1  496e9007-632f-49ee-a0ce-7f2a5eb1a540
		else
		  search --no-floppy --fs-uuid --set=root 496e9007-632f-49ee-a0ce-7f2a5eb1a540
		fi
		echo	'Chargement de Linux 3.19.0-25-generic…'
		linux	/boot/vmlinuz-3.19.0-25-generic root=UUID=496e9007-632f-49ee-a0ce-7f2a5eb1a540 ro recovery nomodeset 
		echo	'Chargement du disque mémoire initial…'
		initrd	/boot/initrd.img-3.19.0-25-generic
	}
}

### END /etc/grub.d/10_linux ###

### BEGIN /etc/grub.d/20_linux_xen ###

### END /etc/grub.d/20_linux_xen ###

### BEGIN /etc/grub.d/20_memtest86+ ###
menuentry 'Memory test (memtest86+)' {
	insmod part_msdos
	insmod ext2
	set root='hd0,msdos1'
	if [ x$feature_platform_search_hint = xy ]; then
	  search --no-floppy --fs-uuid --set=root --hint-bios=hd0,msdos1 --hint-efi=hd0,msdos1 --hint-baremetal=ahci0,msdos1  496e9007-632f-49ee-a0ce-7f2a5eb1a540
	else
	  search --no-floppy --fs-uuid --set=root 496e9007-632f-49ee-a0ce-7f2a5eb1a540
	fi
	knetbsd	/boot/memtest86+.elf
}
menuentry 'Memory test (memtest86+, serial console 115200)' {
	insmod part_msdos
	insmod ext2
	set root='hd0,msdos1'
	if [ x$feature_platform_search_hint = xy ]; then
	  search --no-floppy --fs-uuid --set=root --hint-bios=hd0,msdos1 --hint-efi=hd0,msdos1 --hint-baremetal=ahci0,msdos1  496e9007-632f-49ee-a0ce-7f2a5eb1a540
	else
	  search --no-floppy --fs-uuid --set=root 496e9007-632f-49ee-a0ce-7f2a5eb1a540
	fi
	linux16	/boot/memtest86+.bin console=ttyS0,115200n8
}
### END /etc/grub.d/20_memtest86+ ###

### BEGIN /etc/grub.d/30_os-prober ###
### END /etc/grub.d/30_os-prober ###

### BEGIN /etc/grub.d/30_uefi-firmware ###
### END /etc/grub.d/30_uefi-firmware ###

### BEGIN /etc/grub.d/40_custom ###
# This file provides an easy way to add custom menu entries.  Simply type the
# menu entries you want to add after this comment.  Be careful not to change
# the 'exec tail' line above.
### END /etc/grub.d/40_custom ###

### BEGIN /etc/grub.d/41_custom ###
if [ -f  ${config_directory}/custom.cfg ]; then
  source ${config_directory}/custom.cfg
elif [ -z "${config_directory}" -a -f  $prefix/custom.cfg ]; then
  source $prefix/custom.cfg;
fi
### END /etc/grub.d/41_custom ###
--------------------------------------------------------------------------------

=============================== sda1/etc/fstab: ================================

--------------------------------------------------------------------------------
# /etc/fstab: static file system information.
#
# Use 'blkid' to print the universally unique identifier for a
# device; this may be used with UUID= as a more robust way to name devices
# that works even if disks are added and removed. See fstab(5).
#
# <file system> <mount point>   <type>  <options>       <dump>  <pass>
# / was on /dev/sda1 during installation
UUID=496e9007-632f-49ee-a0ce-7f2a5eb1a540 /               ext4    errors=remount-ro 0       1
# swap was on /dev/sda5 during installation
UUID=4dc511c8-8d84-43fc-83f7-23488cbba3fa none            swap    sw              0       0
--------------------------------------------------------------------------------

=================== sda1: Location of files loaded by Grub: ====================

           GiB - GB             File                                 Fragment(s)

 144.133251190 = 154.761900032  boot/grub/grub.cfg                             1
 100.156139374 = 107.541835776  boot/grub/i386-pc/core.img                     1
   0.897716522 = 0.963915776    boot/vmlinuz-3.19.0-25-generic                 1
   2.772720337 = 2.977185792    boot/vmlinuz-3.19.0-39-generic                 1
 100.796157837 = 108.229050368  boot/vmlinuz-3.19.0-41-generic                 1
   3.952407837 = 4.243865600    boot/vmlinuz-3.19.0-42-generic                 1
   4.284439087 = 4.600381440    boot/vmlinuz-3.19.0-43-generic                 1
   5.698501587 = 6.118719488    boot/vmlinuz-3.19.0-47-generic                 1
   1.300064087 = 1.395933184    boot/vmlinuz-3.19.0-49-generic                 2
   6.546161652 = 7.028887552    boot/vmlinuz-3.19.0-51-generic                 1
 485.764915466 = 521.586106368  boot/vmlinuz-3.19.0-56-generic                 1
   5.995395660 = 6.437507072    boot/vmlinuz-3.19.0-58-generic                 1
   7.464145660 = 8.014565376    boot/vmlinuz-3.19.0-59-generic                 1
 103.624301910 = 111.265746944  boot/vmlinuz-3.19.0-61-generic                 1
 105.522739410 = 113.304178688  boot/vmlinuz-3.19.0-64-generic                 1
 103.073520660 = 110.674350080  boot/vmlinuz-3.19.0-65-generic                 1
 105.120395660 = 112.872165376  boot/vmlinuz-3.19.0-66-generic                 1
 486.534458160 = 522.412396544  boot/vmlinuz-3.19.0-68-generic                 2
 486.726188660 = 522.618265600  boot/vmlinuz-4.4.0-36-generic                  1
 486.726188660 = 522.618265600  vmlinuz                                        1
 486.534458160 = 522.412396544  vmlinuz.old                                    2
   2.336910248 = 2.509238272    boot/initrd.img-3.19.0-25-generic              2
   3.352535248 = 3.599757312    boot/initrd.img-3.19.0-39-generic              2
   3.355346680 = 3.602776064    boot/initrd.img-3.19.0-41-generic              2
   4.274410248 = 4.589613056    boot/initrd.img-3.19.0-42-generic              2
   4.527225494 = 4.861071360    boot/initrd.img-3.19.0-43-generic              1
   6.165035248 = 6.619656192    boot/initrd.img-3.19.0-47-generic              2
   4.041900635 = 4.339957760    boot/initrd.img-3.19.0-49-generic              2
   6.756610870 = 7.254855680    boot/initrd.img-3.19.0-51-generic              2
   6.898880005 = 7.407616000    boot/initrd.img-3.19.0-56-generic              2
 492.878803253 = 529.224585216  boot/initrd.img-3.19.0-58-generic              2
   7.961910248 = 8.549036032    boot/initrd.img-3.19.0-59-generic              2
 104.253807068 = 111.941672960  boot/initrd.img-3.19.0-61-generic              3
 105.917865753 = 113.728442368  boot/initrd.img-3.19.0-64-generic              1
  51.032222748 = 54.795431936   boot/initrd.img-3.19.0-65-generic              2
  51.071285248 = 54.837374976   boot/initrd.img-3.19.0-66-generic              2
 487.089740753 = 523.008626688  boot/initrd.img-3.19.0-68-generic              1
 487.093761444 = 523.012943872  boot/initrd.img-4.4.0-36-generic               2
 487.093761444 = 523.012943872  initrd.img                                     2
 487.089740753 = 523.008626688  initrd.img.old                                 1

========= Devices which don't seem to have a corresponding hard drive: =========

sdc 


ADDITIONAL INFORMATION :
=================== log of boot-info 2016-09-20__22h27 ===================
boot-info version : 4ppa38
boot-sav version : 4ppa38
glade2script version : 3.2.3~ppa1
boot-sav-extra version : 4ppa38
boot-info is executed in installed-session (Ubuntu 14.04.5 LTS, trusty, Ubuntu, x86_64)
CPU op-mode(s):        32-bit, 64-bit
BOOT_IMAGE=/boot/vmlinuz-4.4.0-36-generic root=UUID=496e9007-632f-49ee-a0ce-7f2a5eb1a540 ro quiet splash vt.handoff=7

=================== os-prober:
/dev/sda1:L'OS actuellement utilisé - Ubuntu 14.04.5 LTS CurrentSession:linux

=================== blkid:
/dev/sda1: UUID="496e9007-632f-49ee-a0ce-7f2a5eb1a540" TYPE="ext4"
/dev/sda5: UUID="4dc511c8-8d84-43fc-83f7-23488cbba3fa" TYPE="swap"
/dev/sdb2: UUID="6509CCA977639C2C" TYPE="ntfs"


1 disks with OS, 1 OS : 1 Linux, 0 MacOS, 0 Windows, 0 unknown type OS.

Avertissement : la partition étendue ne commence pas sur une frontière de
cylindres. DOS et Linux interpréteront les contenus différemment.

=================== /etc/grub.d/ :
drwxr-xr-x  2 root root     4096 août   7 07:46 grub.d
total 76
-rwxr-xr-x 1 root root  9791 déc.  17  2015 00_header
-rwxr-xr-x 1 root root  6058 mai   13  2015 05_debian_theme
-rwxr-xr-x 1 root root 11608 juin  26  2015 10_linux
-rwxr-xr-x 1 root root 10412 juin  26  2015 20_linux_xen
-rwxr-xr-x 1 root root  1992 mars  12  2014 20_memtest86+
-rwxr-xr-x 1 root root 11692 juin  26  2015 30_os-prober
-rwxr-xr-x 1 root root  1418 août   2 09:27 30_uefi-firmware
-rwxr-xr-x 1 root root   214 juin  26  2015 40_custom
-rwxr-xr-x 1 root root   216 juin  26  2015 41_custom
-rw-r--r-- 1 root root   483 juin  26  2015 README




=================== /etc/default/grub :

# If you change this file, run 'update-grub' afterwards to update
# /boot/grub/grub.cfg.
# For full documentation of the options in this file, see:
#   info -f grub -n 'Simple configuration'

GRUB_DEFAULT=0
GRUB_HIDDEN_TIMEOUT=0
GRUB_HIDDEN_TIMEOUT_QUIET=true
GRUB_TIMEOUT=10
GRUB_DISTRIBUTOR=`lsb_release -i -s 2> /dev/null || echo Debian`
GRUB_CMDLINE_LINUX_DEFAULT="quiet splash"
GRUB_CMDLINE_LINUX=""

# Uncomment to enable BadRAM filtering, modify to suit your needs
# This works with Linux (no patch required) and with any kernel that obtains
# the memory map information from GRUB (GNU Mach, kernel of FreeBSD ...)
#GRUB_BADRAM="0x01234567,0xfefefefe,0x89abcdef,0xefefefef"

# Uncomment to disable graphical terminal (grub-pc only)
#GRUB_TERMINAL=console

# The resolution used on graphical terminal
# note that you can use only modes which your graphic card supports via VBE
# you can see them in real GRUB with the command `vbeinfo'
#GRUB_GFXMODE=640x480

# Uncomment if you don't want GRUB to pass "root=UUID=xxx" parameter to Linux
#GRUB_DISABLE_LINUX_UUID=true

# Uncomment to disable generation of recovery mode menu entries
#GRUB_DISABLE_RECOVERY="true"

# Uncomment to get a beep at grub start
#GRUB_INIT_TUNE="480 440 1"




=================== UEFI/Legacy mode:
This installed-session is not in EFI-mode.
EFI in dmesg.
[    0.000000] ACPI: UEFI 0x00000000AD1C2520 000042 (v01 ALASKA A M I    01072009      00000000)
SecureBoot disabled.


=================== PARTITIONS & DISKS:
sda1	: sda,	not-sepboot,	grubenv-ok	grub2,	grub-pc ,	update-grub,	64,	with-boot,	is-os,	not--efi--part,	fstab-without-boot,	fstab-without-efi,	no-nt,	no-winload,	no-recov-nor-hid,	no-bmgr,	notwinboot,	apt-get,	grub-install,	with--usr,	fstab-without-usr,	not-sep-usr,	standard,	farbios,	.
sdb2	: sdb,	not-sepboot,	no-grubenv	nogrub,	no-docgrub,	no-update-grub,	32,	no-boot,	no-os,	not--efi--part,	part-has-no-fstab,	part-has-no-fstab,	no-nt,	no-winload,	no-recov-nor-hid,	no-bmgr,	notwinboot,	nopakmgr,	nogrubinstall,	no---usr,	part-has-no-fstab,	not-sep-usr,	standard,	farbios,	/media/vadee/6509CCA977639C2C.

sda	: not-GPT,	BIOSboot-not-needed,	has-no-EFIpart, 	not-usb,	has-os,	2048 sectors * 512 bytes
sdb	: not-GPT,	BIOSboot-not-needed,	has-no-EFIpart, 	not-usb,	no-os,	2048 sectors * 512 bytes


=================== parted -l:

Model: ATA WDC WD10EZEX-00B (scsi)
Disk /dev/sda: 1000GB
Sector size (logical/physical): 512B/4096B
Partition Table: msdos

Number  Start   End     Size    Type      File system     Flags
1      1049kB  984GB   984GB   primary   ext4            boot
2      984GB   1000GB  16.3GB  extended
5      984GB   1000GB  16.3GB  logical   linux-swap(v1)


Model: ATA WDC WD1002FBYS-1 (scsi)
Disk /dev/sdb: 1000GB
Sector size (logical/physical): 512B/512B
Partition Table: msdos

Number  Start   End     Size    Type     File system  Flags
2      1049kB  1000GB  1000GB  primary  ntfs

=================== parted -lm:

BYT;
/dev/sda:1000GB:scsi:512:4096:msdos:ATA WDC WD10EZEX-00B;
1:1049kB:984GB:984GB:ext4::boot;
2:984GB:1000GB:16.3GB:::;
5:984GB:1000GB:16.3GB:linux-swap(v1)::;

BYT;
/dev/sdb:1000GB:scsi:512:512:msdos:ATA WDC WD1002FBYS-1;
2:1049kB:1000GB:1000GB:ntfs::;

=================== lsblk:
KNAME TYPE FSTYPE   SIZE LABEL
sda   disk        931,5G
sda1  part ext4   916,3G
sda2  part            1K
sda5  part swap    15,2G
sdb   disk        931,5G
sdb2  part ntfs   931,5G
sr0   rom          1024M

KNAME ROTA RO RM STATE   MOUNTPOINT
sda      1  0  0 running
sda1     1  0  0         /
sda2     1  0  0
sda5     1  0  0         [SWAP]
sdb      1  0  0 running
sdb2     1  0  0         /media/vadee/6509CCA977639C2C
sr0      1  0  1 running


=================== mount:
/dev/sda1 on / type ext4 (rw,errors=remount-ro)
proc on /proc type proc (rw,noexec,nosuid,nodev)
sysfs on /sys type sysfs (rw,noexec,nosuid,nodev)
none on /sys/fs/cgroup type tmpfs (rw)
none on /sys/fs/fuse/connections type fusectl (rw)
none on /sys/kernel/debug type debugfs (rw)
none on /sys/kernel/security type securityfs (rw)
udev on /dev type devtmpfs (rw,mode=0755)
devpts on /dev/pts type devpts (rw,noexec,nosuid,gid=5,mode=0620)
tmpfs on /run type tmpfs (rw,noexec,nosuid,size=10%,mode=0755)
none on /run/lock type tmpfs (rw,noexec,nosuid,nodev,size=5242880)
none on /run/shm type tmpfs (rw,nosuid,nodev)
none on /run/user type tmpfs (rw,noexec,nosuid,nodev,size=104857600,mode=0755)
none on /sys/fs/pstore type pstore (rw)
systemd on /sys/fs/cgroup/systemd type cgroup (rw,noexec,nosuid,nodev,none,name=systemd)
gvfsd-fuse on /run/user/1000/gvfs type fuse.gvfsd-fuse (rw,nosuid,nodev,user=vadee)
/dev/sdb2 on /media/vadee/6509CCA977639C2C type fuseblk (rw,nosuid,nodev,allow_other,default_permissions,blksize=4096)


=================== ls:
/sys/block/sda (filtered):  alignment_offset bdi capability dev device discard_alignment events events_async events_poll_msecs ext_range holders inflight integrity power queue range removable ro sda1 sda2 sda5 size slaves stat subsystem trace uevent
/sys/block/sdb (filtered):  alignment_offset bdi capability dev device discard_alignment events events_async events_poll_msecs ext_range holders inflight integrity power queue range removable ro sdb2 size slaves stat subsystem trace uevent
/sys/block/sdc (filtered):  alignment_offset bdi capability dev device discard_alignment events events_async events_poll_msecs ext_range holders inflight integrity power queue range removable ro size slaves stat subsystem trace uevent
/sys/block/sr0 (filtered):  alignment_offset bdi capability dev device discard_alignment events events_async events_poll_msecs ext_range holders inflight integrity power queue range removable ro size slaves stat subsystem trace uevent
/dev (filtered):  autofs block bsg btrfs-control bus cdrom char console core cpu cpu_dma_latency cuse disk dri ecryptfs fb0 fd full fuse hidraw0 hidraw1 hpet hwrng i2c-0 i2c-1 i2c-2 i2c-3 i2c-4 i2c-5 i2c-6 i2c-7 i2c-8 input kfd kmsg kvm lightnvm log mapper mcelog mem memory_bandwidth net network_latency network_throughput null port ppp psaux ptmx pts random rfkill rtc rtc0 sda sda1 sda2 sda5 sdb sdb2 sdc sg0 sg1 sg2 sg3 shm snapshot snd sr0 stderr stdin stdout uhid uinput urandom usb userio vfio vga_arbiter vhci vhost-net zero
ls /dev/mapper:  control
ls: impossible d'accéder à : Aucun fichier ou dossier de ce type

=================== hexdump -n512 -C /dev/sdb2
00000000  eb 52 90 4e 54 46 53 20  20 20 20 00 02 08 00 00  |.R.NTFS    .....|
00000010  00 00 00 00 00 f8 00 00  3f 00 ff 00 00 08 00 00  |........?.......|
00000020  00 00 00 00 80 00 80 00  ff 57 70 74 00 00 00 00  |.........Wpt....|
00000030  04 00 00 00 00 00 00 00  7f 05 47 07 00 00 00 00  |..........G.....|
00000040  f6 00 00 00 01 00 00 00  2c 9c 63 77 a9 cc 09 65  |........,.cw...e|
00000050  00 00 00 00 fa 33 c0 8e  d0 bc 00 7c fb 68 c0 07  |.....3.....|.h..|
00000060  1f 1e 68 66 00 cb 88 16  0e 00 66 81 3e 03 00 4e  |..hf......f.>..N|
00000070  54 46 53 75 15 b4 41 bb  aa 55 cd 13 72 0c 81 fb  |TFSu..A..U..r...|
00000080  55 aa 75 06 f7 c1 01 00  75 03 e9 d2 00 1e 83 ec  |U.u.....u.......|
00000090  18 68 1a 00 b4 48 8a 16  0e 00 8b f4 16 1f cd 13  |.h...H..........|
000000a0  9f 83 c4 18 9e 58 1f 72  e1 3b 06 0b 00 75 db a3  |.....X.r.;...u..|
000000b0  0f 00 c1 2e 0f 00 04 1e  5a 33 db b9 00 20 2b c8  |........Z3... +.|
000000c0  66 ff 06 11 00 03 16 0f  00 8e c2 ff 06 16 00 e8  |f...............|
000000d0  40 00 2b c8 77 ef b8 00  bb cd 1a 66 23 c0 75 2d  |@.+.w......f#.u-|
000000e0  66 81 fb 54 43 50 41 75  24 81 f9 02 01 72 1e 16  |f..TCPAu$....r..|
000000f0  68 07 bb 16 68 70 0e 16  68 09 00 66 53 66 53 66  |h...hp..h..fSfSf|
00000100  55 16 16 16 68 b8 01 66  61 0e 07 cd 1a e9 6a 01  |U...h..fa.....j.|
00000110  90 90 66 60 1e 06 66 a1  11 00 66 03 06 1c 00 1e  |..f`..f...f.....|
00000120  66 68 00 00 00 00 66 50  06 53 68 01 00 68 10 00  |fh....fP.Sh..h..|
00000130  b4 42 8a 16 0e 00 16 1f  8b f4 cd 13 66 59 5b 5a  |.B..........fY[Z|
00000140  66 59 66 59 1f 0f 82 16  00 66 ff 06 11 00 03 16  |fYfY.....f......|
00000150  0f 00 8e c2 ff 0e 16 00  75 bc 07 1f 66 61 c3 a0  |........u...fa..|
00000160  f8 01 e8 08 00 a0 fb 01  e8 02 00 eb fe b4 01 8b  |................|
00000170  f0 ac 3c 00 74 09 b4 0e  bb 07 00 cd 10 eb f2 c3  |..<.t...........|
00000180  0d 0a 41 20 64 69 73 6b  20 72 65 61 64 20 65 72  |..A disk read er|
00000190  72 6f 72 20 6f 63 63 75  72 72 65 64 00 0d 0a 42  |ror occurred...B|
000001a0  4f 4f 54 4d 47 52 20 69  73 20 6d 69 73 73 69 6e  |OOTMGR is missin|
000001b0  67 00 0d 0a 42 4f 4f 54  4d 47 52 20 69 73 20 63  |g...BOOTMGR is c|
000001c0  6f 6d 70 72 65 73 73 65  64 00 0d 0a 50 72 65 73  |ompressed...Pres|
000001d0  73 20 43 74 72 6c 2b 41  6c 74 2b 44 65 6c 20 74  |s Ctrl+Alt+Del t|
000001e0  6f 20 72 65 73 74 61 72  74 0d 0a 00 00 00 00 00  |o restart.......|
000001f0  00 00 00 00 00 00 00 00  80 9d b2 ca 00 00 55 aa  |..............U.|
00000200

=================== df -Th:

Filesystem     Type      Size  Used Avail Use% Mounted on
udev           devtmpfs  7.5G   12K  7.5G   1% /dev
tmpfs          tmpfs     1.5G  1.3M  1.5G   1% /run
/dev/sda1      ext4      902G   22G  835G   3% /
none           tmpfs     4.0K     0  4.0K   0% /sys/fs/cgroup
none           tmpfs     5.0M     0  5.0M   0% /run/lock
none           tmpfs     7.5G  316K  7.5G   1% /run/shm
none           tmpfs     100M   48K  100M   1% /run/user
/dev/sdb2      fuseblk   932G   45G  888G   5% /media/vadee/6509CCA977639C2C

=================== fdisk -l:

Disk /dev/sda: 1000.2 GB, 1000204886016 bytes
255 heads, 63 sectors/track, 121601 cylinders, total 1953525168 sectors
Units = sectors of 1 * 512 = 512 bytes
Sector size (logical/physical): 512 bytes / 4096 bytes
I/O size (minimum/optimal): 4096 bytes / 4096 bytes
Disk identifier: 0x000e8201

Device Boot      Start         End      Blocks   Id  System
/dev/sda1   *        2048  1921662975   960830464   83  Linux
/dev/sda2      1921665022  1953523711    15929345    5  Extended
Partition 2 does not start on physical sector boundary.
/dev/sda5      1921665024  1953523711    15929344   82  Linux swap / Solaris

Disk /dev/sdb: 1000.2 GB, 1000204886016 bytes
255 heads, 63 sectors/track, 121601 cylinders, total 1953525168 sectors
Units = sectors of 1 * 512 = 512 bytes
Sector size (logical/physical): 512 bytes / 512 bytes
I/O size (minimum/optimal): 512 bytes / 512 bytes
Disk identifier: 0x000b9b9c

Device Boot      Start         End      Blocks   Id  System
/dev/sdb2            2048  1953521663   976759808    7  HPFS/NTFS/exFAT




=================== Suggested repair
The default repair of the Boot-Repair utility would reinstall the grub2 of sda1 into the MBRs of all disks (except USB without OS).
The boot flag would be placed on sdb2.
Additional repair would be performed: unhide-bootmenu-10s


=================== Final advice in case of suggested repair
N'oubliez pas de régler votre BIOS pour qu'il amorce sur le disque sda (1000GB) !

Les fichiers de démarrage de [L'OS actuellement utilisé - Ubuntu 14.04.5 LTS] sont loin du début du disque. Votre BIOS pourrait ne pas les détecter. Vous voudrez peut-être re-essayer après avoir créé une partition /boot (EXT4, >200MB, en début de disque). Cela peut être réalisé via des outils tels que gParted. Puis sélectionnez cette partition via l'option [Partition /boot séparée :] de [Boot-Repair]. (http://doc.ubuntu-fr.org/tutoriel/partition_boot)


=================== User settings
The settings chosen by the user will not act on the boot.

On y voit que Ubuntu est installé sur le disque dur repéré "sda" dans le Boot-Info qui est un Western Digital Caviar Blue 1 To mais il y a aussi un disque dur "sdb" formaté en NTFS qui doit servir pour le stockage de données. Ce dernier est un Western Digital Caviar RE3 RAID Edition 1 To.
Je te prie de déconnecter le connecteur SATA de ce dernier afin qu'il ne soit plus détecté par ton ordi au démarrage. Ensuite, tu réessaieras de démarrer pour voir si ça prend toujours 25 minutes ou si c'est redevenu normal.

Dernière modification par malbo (Le 21/09/2016, à 05:57)

Hors ligne

#9 Le 21/09/2016, à 07:39

goliath60

Re : un démarrage qui dure 25 min [résolu]

le DD de sauvegarde est rétiré,
le démarrage dure encore 25min

(il st vrai que ça aurait pu venir de lui car j'avais ouvert un sujet car il bloque au démarrage iciici)

Hors ligne

#10 Le 21/09/2016, à 08:03

malbo

Re : un démarrage qui dure 25 min [résolu]

Pourrais-tu forcer l'affichage de Grub au démarrage en faisant la modif décrite ici : http://forum.ubuntu-fr.org/viewtopic.ph … #p16026791 (tu n’oublieras pas de terminer par la commande de mise à jour de Grub "sudo  update-grub" comme indiqué par jeanjd63 dans ce post)
Au redémarrage suivant, note combien il faut de temps pour que le menu Grub apparaisse.

Dernière modification par malbo (Le 21/09/2016, à 08:03)

Hors ligne

#11 Le 21/09/2016, à 12:05

Bougron

Re : un démarrage qui dure 25 min [résolu]

Bonjour
Puisque cela démarre, peux-tu faire les commandes suivantes

dmesg  >TRACE.txt
gedit TRACE.txt

et faire un copier/coller dans cette discussion de ce qui va s'afficher à l'écran en complétant par une mise en forme
Tu donneras aussi le résultat de la commande

systemd-analyze blame 

Ainsi que la sortie de la commande

sudo smartctl  -s on  -a /dev/sda

Il te sera peu-être nécessaire d'installer l'application GSMARTCONTROL qui est dans la logithéque

Dernière modification par Bougron (Le 21/09/2016, à 12:08)

Hors ligne

#12 Le 21/09/2016, à 19:38

goliath60

Re : un démarrage qui dure 25 min [résolu]

malbo:
j'ai suivi le lien que tu m'as donné
après un redémarrage, le grub s'affiche en quelques secondes, reste affiché quelques secondes, puis page violette  (couleur de fond de ubuntu), puis écran noir, l'écran est en mode veille (bouton d'allumage orange). 25 min pour démarrer

Hors ligne

#13 Le 21/09/2016, à 19:40

goliath60

Re : un démarrage qui dure 25 min [résolu]

Bougron:

vadee@vadee:~$ dmesg  >TRACE.txt
vadee@vadee:~$ gedit TRACE.txt

(gedit:2343): Gtk-WARNING **: Attempting to read the recently used resources file at `/home/vadee/.local/share/recently-used.xbel', but the parser failed: L'ouverture du fichier « /home/vadee/.local/share/recently-used.xbel » a échoué : Permission non accordée.
[    0.000000] Initializing cgroup subsys cpuset
[    0.000000] Initializing cgroup subsys cpu
[    0.000000] Initializing cgroup subsys cpuacct
[    0.000000] Linux version 4.4.0-36-generic (buildd@lgw01-20) (gcc version 4.8.4 (Ubuntu 4.8.4-2ubuntu1~14.04.3) ) #55~14.04.1-Ubuntu SMP Fri Aug 12 11:49:30 UTC 2016 (Ubuntu 4.4.0-36.55~14.04.1-generic 4.4.16)
[    0.000000] Command line: BOOT_IMAGE=/boot/vmlinuz-4.4.0-36-generic root=UUID=496e9007-632f-49ee-a0ce-7f2a5eb1a540 ro quiet splash vt.handoff=7
[    0.000000] KERNEL supported cpus:
[    0.000000]   Intel GenuineIntel
[    0.000000]   AMD AuthenticAMD
[    0.000000]   Centaur CentaurHauls
[    0.000000] tseg: 00af000000
[    0.000000] x86/fpu: xstate_offset[2]:  576, xstate_sizes[2]:  256
[    0.000000] x86/fpu: Supporting XSAVE feature 0x01: 'x87 floating point registers'
[    0.000000] x86/fpu: Supporting XSAVE feature 0x02: 'SSE registers'
[    0.000000] x86/fpu: Supporting XSAVE feature 0x04: 'AVX registers'
[    0.000000] x86/fpu: Enabled xstate features 0x7, context size is 832 bytes, using 'standard' format.
[    0.000000] x86/fpu: Using 'lazy' FPU context switches.
[    0.000000] e820: BIOS-provided physical RAM map:
[    0.000000] BIOS-e820: [mem 0x0000000000000000-0x000000000009e7ff] usable
[    0.000000] BIOS-e820: [mem 0x000000000009e800-0x000000000009ffff] reserved
[    0.000000] BIOS-e820: [mem 0x00000000000e0000-0x00000000000fffff] reserved
[    0.000000] BIOS-e820: [mem 0x0000000000100000-0x00000000ad183fff] usable
[    0.000000] BIOS-e820: [mem 0x00000000ad184000-0x00000000ad1b3fff] reserved
[    0.000000] BIOS-e820: [mem 0x00000000ad1b4000-0x00000000ad1c3fff] ACPI data
[    0.000000] BIOS-e820: [mem 0x00000000ad1c4000-0x00000000ad845fff] ACPI NVS
[    0.000000] BIOS-e820: [mem 0x00000000ad846000-0x00000000ae3e7fff] reserved
[    0.000000] BIOS-e820: [mem 0x00000000ae3e8000-0x00000000ae3e8fff] usable
[    0.000000] BIOS-e820: [mem 0x00000000ae3e9000-0x00000000ae5eefff] ACPI NVS
[    0.000000] BIOS-e820: [mem 0x00000000ae5ef000-0x00000000aea32fff] usable
[    0.000000] BIOS-e820: [mem 0x00000000aea33000-0x00000000aeff3fff] reserved
[    0.000000] BIOS-e820: [mem 0x00000000aeff4000-0x00000000aeffffff] usable
[    0.000000] BIOS-e820: [mem 0x00000000f8000000-0x00000000fbffffff] reserved
[    0.000000] BIOS-e820: [mem 0x00000000feb80000-0x00000000fec00fff] reserved
[    0.000000] BIOS-e820: [mem 0x00000000fec10000-0x00000000fec10fff] reserved
[    0.000000] BIOS-e820: [mem 0x00000000fed00000-0x00000000fed00fff] reserved
[    0.000000] BIOS-e820: [mem 0x00000000fed40000-0x00000000fed44fff] reserved
[    0.000000] BIOS-e820: [mem 0x00000000fed80000-0x00000000fed8ffff] reserved
[    0.000000] BIOS-e820: [mem 0x00000000ff000000-0x00000000ffffffff] reserved
[    0.000000] BIOS-e820: [mem 0x0000000100000000-0x000000041effffff] usable
[    0.000000] NX (Execute Disable) protection: active
[    0.000000] SMBIOS 2.7 present.
[    0.000000] DMI: System manufacturer System Product Name/A68HM-PLUS, BIOS 0803 04/09/2015
[    0.000000] e820: update [mem 0x00000000-0x00000fff] usable ==> reserved
[    0.000000] e820: remove [mem 0x000a0000-0x000fffff] usable
[    0.000000] e820: last_pfn = 0x41f000 max_arch_pfn = 0x400000000
[    0.000000] MTRR default type: uncachable
[    0.000000] MTRR fixed ranges enabled:
[    0.000000]   00000-9FFFF write-back
[    0.000000]   A0000-BFFFF write-through
[    0.000000]   C0000-D0FFF write-protect
[    0.000000]   D1000-E7FFF uncachable
[    0.000000]   E8000-FFFFF write-protect
[    0.000000] MTRR variable ranges enabled:
[    0.000000]   0 base 000000000000 mask FFFF80000000 write-back
[    0.000000]   1 base 000080000000 mask FFFFE0000000 write-back
[    0.000000]   2 base 0000A0000000 mask FFFFF0000000 write-back
[    0.000000]   3 disabled
[    0.000000]   4 disabled
[    0.000000]   5 disabled
[    0.000000]   6 disabled
[    0.000000]   7 disabled
[    0.000000] TOM2: 000000041f000000 aka 16880M
[    0.000000] x86/PAT: Configuration [0-7]: WB  WC  UC- UC  WB  WC  UC- WT  
[    0.000000] e820: update [mem 0xb0000000-0xffffffff] usable ==> reserved
[    0.000000] e820: last_pfn = 0xaf000 max_arch_pfn = 0x400000000
[    0.000000] found SMP MP-table at [mem 0x000fd840-0x000fd84f] mapped at [ffff8800000fd840]
[    0.000000] Scanning 1 areas for low memory corruption
[    0.000000] Base memory trampoline at [ffff880000098000] 98000 size 24576
[    0.000000] Using GB pages for direct mapping
[    0.000000] BRK [0x02004000, 0x02004fff] PGTABLE
[    0.000000] BRK [0x02005000, 0x02005fff] PGTABLE
[    0.000000] BRK [0x02006000, 0x02006fff] PGTABLE
[    0.000000] BRK [0x02007000, 0x02007fff] PGTABLE
[    0.000000] BRK [0x02008000, 0x02008fff] PGTABLE
[    0.000000] BRK [0x02009000, 0x02009fff] PGTABLE
[    0.000000] RAMDISK: [mem 0x357b4000-0x36bd1fff]
[    0.000000] ACPI: Early table checksum verification disabled
[    0.000000] ACPI: RSDP 0x00000000000F0490 000024 (v02 ALASKA)
[    0.000000] ACPI: XSDT 0x00000000AD1BB078 00006C (v01 ALASKA A M I    01072009 AMI  00010013)
[    0.000000] ACPI: FACP 0x00000000AD1C2280 00010C (v05 ALASKA A M I    01072009 AMI  00010013)
[    0.000000] ACPI BIOS Warning (bug): Optional FADT field Pm2ControlBlock has zero address or length: 0x0000000000000000/0x1 (20150930/tbfadt-654)
[    0.000000] ACPI: DSDT 0x00000000AD1BB180 007100 (v02 ALASKA A M I    00000000 INTL 20051117)
[    0.000000] ACPI: FACS 0x00000000AD83B080 000040
[    0.000000] ACPI: APIC 0x00000000AD1C2390 000072 (v03 ALASKA A M I    01072009 AMI  00010013)
[    0.000000] ACPI: FPDT 0x00000000AD1C2408 000044 (v01 ALASKA A M I    01072009 AMI  00010013)
[    0.000000] ACPI: MCFG 0x00000000AD1C2450 00003C (v01 ALASKA A M I    01072009 MSFT 00010013)
[    0.000000] ACPI: HPET 0x00000000AD1C2490 000038 (v01 ALASKA A M I    01072009 AMI  00000005)
[    0.000000] ACPI: CRAT 0x00000000AD1C35C0 000578 (v01 AMD    BANTRY   00000001 AMD  00000001)
[    0.000000] ACPI: UEFI 0x00000000AD1C2520 000042 (v01 ALASKA A M I    01072009      00000000)
[    0.000000] ACPI: SSDT 0x00000000AD1C2568 000B9C (v01 AMD    BANTRY   00000001 AMD  00000001)
[    0.000000] ACPI: SSDT 0x00000000AD1C3108 0004B7 (v02 AMD    BANTRY   00000001 MSFT 04000000)
[    0.000000] ACPI: Local APIC address 0xfee00000
[    0.000000] No NUMA configuration found
[    0.000000] Faking a node at [mem 0x0000000000000000-0x000000041effffff]
[    0.000000] NODE_DATA(0) allocated [mem 0x41eff7000-0x41effbfff]
[    0.000000] Zone ranges:
[    0.000000]   DMA      [mem 0x0000000000001000-0x0000000000ffffff]
[    0.000000]   DMA32    [mem 0x0000000001000000-0x00000000ffffffff]
[    0.000000]   Normal   [mem 0x0000000100000000-0x000000041effffff]
[    0.000000]   Device   empty
[    0.000000] Movable zone start for each node
[    0.000000] Early memory node ranges
[    0.000000]   node   0: [mem 0x0000000000001000-0x000000000009dfff]
[    0.000000]   node   0: [mem 0x0000000000100000-0x00000000ad183fff]
[    0.000000]   node   0: [mem 0x00000000ae3e8000-0x00000000ae3e8fff]
[    0.000000]   node   0: [mem 0x00000000ae5ef000-0x00000000aea32fff]
[    0.000000]   node   0: [mem 0x00000000aeff4000-0x00000000aeffffff]
[    0.000000]   node   0: [mem 0x0000000100000000-0x000000041effffff]
[    0.000000] Initmem setup node 0 [mem 0x0000000000001000-0x000000041effffff]
[    0.000000] On node 0 totalpages: 3982706
[    0.000000]   DMA zone: 64 pages used for memmap
[    0.000000]   DMA zone: 21 pages reserved
[    0.000000]   DMA zone: 3997 pages, LIFO batch:0
[    0.000000]   DMA32 zone: 11032 pages used for memmap
[    0.000000]   DMA32 zone: 706005 pages, LIFO batch:31
[    0.000000]   Normal zone: 51136 pages used for memmap
[    0.000000]   Normal zone: 3272704 pages, LIFO batch:31
[    0.000000] ACPI: PM-Timer IO Port: 0x808
[    0.000000] ACPI: Local APIC address 0xfee00000
[    0.000000] ACPI: LAPIC_NMI (acpi_id[0xff] high edge lint[0x1])
[    0.000000] IOAPIC[0]: apic_id 0, version 33, address 0xfec00000, GSI 0-23
[    0.000000] ACPI: INT_SRC_OVR (bus 0 bus_irq 0 global_irq 2 dfl dfl)
[    0.000000] ACPI: INT_SRC_OVR (bus 0 bus_irq 9 global_irq 9 low level)
[    0.000000] ACPI: IRQ0 used by override.
[    0.000000] ACPI: IRQ9 used by override.
[    0.000000] Using ACPI (MADT) for SMP configuration information
[    0.000000] ACPI: HPET id: 0x10228210 base: 0xfed00000
[    0.000000] smpboot: Allowing 4 CPUs, 0 hotplug CPUs
[    0.000000] PM: Registered nosave memory: [mem 0x00000000-0x00000fff]
[    0.000000] PM: Registered nosave memory: [mem 0x0009e000-0x0009efff]
[    0.000000] PM: Registered nosave memory: [mem 0x0009f000-0x0009ffff]
[    0.000000] PM: Registered nosave memory: [mem 0x000a0000-0x000dffff]
[    0.000000] PM: Registered nosave memory: [mem 0x000e0000-0x000fffff]
[    0.000000] PM: Registered nosave memory: [mem 0xad184000-0xad1b3fff]
[    0.000000] PM: Registered nosave memory: [mem 0xad1b4000-0xad1c3fff]
[    0.000000] PM: Registered nosave memory: [mem 0xad1c4000-0xad845fff]
[    0.000000] PM: Registered nosave memory: [mem 0xad846000-0xae3e7fff]
[    0.000000] PM: Registered nosave memory: [mem 0xae3e9000-0xae5eefff]
[    0.000000] PM: Registered nosave memory: [mem 0xaea33000-0xaeff3fff]
[    0.000000] PM: Registered nosave memory: [mem 0xaf000000-0xf7ffffff]
[    0.000000] PM: Registered nosave memory: [mem 0xf8000000-0xfbffffff]
[    0.000000] PM: Registered nosave memory: [mem 0xfc000000-0xfeb7ffff]
[    0.000000] PM: Registered nosave memory: [mem 0xfeb80000-0xfec00fff]
[    0.000000] PM: Registered nosave memory: [mem 0xfec01000-0xfec0ffff]
[    0.000000] PM: Registered nosave memory: [mem 0xfec10000-0xfec10fff]
[    0.000000] PM: Registered nosave memory: [mem 0xfec11000-0xfecfffff]
[    0.000000] PM: Registered nosave memory: [mem 0xfed00000-0xfed00fff]
[    0.000000] PM: Registered nosave memory: [mem 0xfed01000-0xfed3ffff]
[    0.000000] PM: Registered nosave memory: [mem 0xfed40000-0xfed44fff]
[    0.000000] PM: Registered nosave memory: [mem 0xfed45000-0xfed7ffff]
[    0.000000] PM: Registered nosave memory: [mem 0xfed80000-0xfed8ffff]
[    0.000000] PM: Registered nosave memory: [mem 0xfed90000-0xfeffffff]
[    0.000000] PM: Registered nosave memory: [mem 0xff000000-0xffffffff]
[    0.000000] e820: [mem 0xaf000000-0xf7ffffff] available for PCI devices
[    0.000000] Booting paravirtualized kernel on bare hardware
[    0.000000] clocksource: refined-jiffies: mask: 0xffffffff max_cycles: 0xffffffff, max_idle_ns: 7645519600211568 ns
[    0.000000] setup_percpu: NR_CPUS:256 nr_cpumask_bits:256 nr_cpu_ids:4 nr_node_ids:1
[    0.000000] PERCPU: Embedded 33 pages/cpu @ffff88041ec00000 s98008 r8192 d28968 u524288
[    0.000000] pcpu-alloc: s98008 r8192 d28968 u524288 alloc=1*2097152
[    0.000000] pcpu-alloc: [0] 0 1 2 3 
[    0.000000] Built 1 zonelists in Node order, mobility grouping on.  Total pages: 3920453
[    0.000000] Policy zone: Normal
[    0.000000] Kernel command line: BOOT_IMAGE=/boot/vmlinuz-4.4.0-36-generic root=UUID=496e9007-632f-49ee-a0ce-7f2a5eb1a540 ro quiet splash vt.handoff=7
[    0.000000] PID hash table entries: 4096 (order: 3, 32768 bytes)
[    0.000000] Calgary: detecting Calgary via BIOS EBDA area
[    0.000000] Calgary: Unable to locate Rio Grande table in EBDA - bailing!
[    0.000000] Memory: 15576992K/15930824K available (8176K kernel code, 1292K rwdata, 3960K rodata, 1484K init, 1292K bss, 353832K reserved, 0K cma-reserved)
[    0.000000] SLUB: HWalign=64, Order=0-3, MinObjects=0, CPUs=4, Nodes=1
[    0.000000] Hierarchical RCU implementation.
[    0.000000] 	Build-time adjustment of leaf fanout to 64.
[    0.000000] 	RCU restricting CPUs from NR_CPUS=256 to nr_cpu_ids=4.
[    0.000000] RCU: Adjusting geometry for rcu_fanout_leaf=64, nr_cpu_ids=4
[    0.000000] NR_IRQS:16640 nr_irqs:456 16
[    0.000000] spurious 8259A interrupt: IRQ7.
[    0.000000] vt handoff: transparent VT on vt#7
[    0.000000] Console: colour dummy device 80x25
[    0.000000] console [tty0] enabled
[    0.000000] clocksource: hpet: mask: 0xffffffff max_cycles: 0xffffffff, max_idle_ns: 133484873504 ns
[    0.000000] hpet clockevent registered
[    0.000000] tsc: Fast TSC calibration using PIT
[    0.000000] tsc: Detected 3889.875 MHz processor
[    0.000020] Calibrating delay loop (skipped), value calculated using timer frequency.. 7779.75 BogoMIPS (lpj=15559500)
[    0.000023] pid_max: default: 32768 minimum: 301
[    0.000028] ACPI: Core revision 20150930
[    0.003026] ACPI: 3 ACPI AML tables successfully acquired and loaded
[    0.003043] Security Framework initialized
[    0.003045] Yama: becoming mindful.
[    0.003058] AppArmor: AppArmor initialized
[    0.003932] Dentry cache hash table entries: 2097152 (order: 12, 16777216 bytes)
[    0.007689] Inode-cache hash table entries: 1048576 (order: 11, 8388608 bytes)
[    0.009346] Mount-cache hash table entries: 32768 (order: 6, 262144 bytes)
[    0.009362] Mountpoint-cache hash table entries: 32768 (order: 6, 262144 bytes)
[    0.009619] Initializing cgroup subsys io
[    0.009623] Initializing cgroup subsys memory
[    0.009629] Initializing cgroup subsys devices
[    0.009632] Initializing cgroup subsys freezer
[    0.009634] Initializing cgroup subsys net_cls
[    0.009636] Initializing cgroup subsys perf_event
[    0.009638] Initializing cgroup subsys net_prio
[    0.009640] Initializing cgroup subsys hugetlb
[    0.009643] Initializing cgroup subsys pids
[    0.009661] CPU: Physical Processor ID: 0
[    0.009662] CPU: Processor Core ID: 0
[    0.009663] mce: CPU supports 7 MCE banks
[    0.009673] Last level iTLB entries: 4KB 512, 2MB 1024, 4MB 512
[    0.009674] Last level dTLB entries: 4KB 1024, 2MB 1024, 4MB 512, 1GB 0
[    0.009911] Freeing SMP alternatives memory: 28K (ffffffff81eb8000 - ffffffff81ebf000)
[    0.020496] ftrace: allocating 32030 entries in 126 pages
[    0.029273] smpboot: Max logical packages: 2
[    0.029276] smpboot: APIC(10) Converting physical 1 to logical package 0
[    0.029584] ..TIMER: vector=0x30 apic1=0 pin1=2 apic2=-1 pin2=-1
[    0.173761] smpboot: CPU0: AMD A8-6600K APU with Radeon(tm) HD Graphics (family: 0x15, model: 0x13, stepping: 0x1)
[    0.173776] Performance Events: Fam15h core perfctr, AMD PMU driver.
[    0.173780] ... version:                0
[    0.173781] ... bit width:              48
[    0.173781] ... generic registers:      6
[    0.173782] ... value mask:             0000ffffffffffff
[    0.173783] ... max period:             00007fffffffffff
[    0.173784] ... fixed-purpose events:   0
[    0.173784] ... event mask:             000000000000003f
[    0.174417] NMI watchdog: enabled on all CPUs, permanently consumes one hw-PMU counter.
[    0.174476] x86: Booting SMP configuration:
[    0.174477] .... node  #0, CPUs:      #1 #2 #3
[    0.181080] x86: Booted up 1 node, 4 CPUs
[    0.181083] smpboot: Total of 4 processors activated (31119.00 BogoMIPS)
[    0.181713] devtmpfs: initialized
[    0.184875] evm: security.selinux
[    0.184876] evm: security.SMACK64
[    0.184876] evm: security.SMACK64EXEC
[    0.184877] evm: security.SMACK64TRANSMUTE
[    0.184878] evm: security.SMACK64MMAP
[    0.184879] evm: security.ima
[    0.184880] evm: security.capability
[    0.185010] PM: Registering ACPI NVS region [mem 0xad1c4000-0xad845fff] (6823936 bytes)
[    0.185090] PM: Registering ACPI NVS region [mem 0xae3e9000-0xae5eefff] (2121728 bytes)
[    0.185179] clocksource: jiffies: mask: 0xffffffff max_cycles: 0xffffffff, max_idle_ns: 7645041785100000 ns
[    0.185252] pinctrl core: initialized pinctrl subsystem
[    0.185344] RTC time: 16:40:14, date: 09/21/16
[    0.185437] NET: Registered protocol family 16
[    0.193831] cpuidle: using governor ladder
[    0.205830] cpuidle: using governor menu
[    0.205842] PCCT header not found.
[    0.205993] ACPI: bus type PCI registered
[    0.205995] acpiphp: ACPI Hot Plug PCI Controller Driver version: 0.5
[    0.206070] PCI: MMCONFIG for domain 0000 [bus 00-3f] at [mem 0xf8000000-0xfbffffff] (base 0xf8000000)
[    0.206072] PCI: MMCONFIG at [mem 0xf8000000-0xfbffffff] reserved in E820
[    0.206081] PCI: Using configuration type 1 for base access
[    0.206445] mtrr: your CPUs had inconsistent variable MTRR settings
[    0.206446] mtrr: probably your BIOS does not setup all CPUs.
[    0.206447] mtrr: corrected configuration.
[    0.218325] ACPI: Added _OSI(Module Device)
[    0.218327] ACPI: Added _OSI(Processor Device)
[    0.218328] ACPI: Added _OSI(3.0 _SCP Extensions)
[    0.218329] ACPI: Added _OSI(Processor Aggregator Device)
[    0.219608] ACPI: Executed 1 blocks of module-level executable AML code
[    0.221376] [Firmware Bug]: ACPI: BIOS _OSI(Linux) query ignored
[    0.221680] ACPI: Interpreter enabled
[    0.221684] ACPI Exception: AE_NOT_FOUND, While evaluating Sleep State [\_S1_] (20150930/hwxface-580)
[    0.221688] ACPI Exception: AE_NOT_FOUND, While evaluating Sleep State [\_S2_] (20150930/hwxface-580)
[    0.221699] ACPI: (supports S0 S3 S4 S5)
[    0.221700] ACPI: Using IOAPIC for interrupt routing
[    0.221830] PCI: Using host bridge windows from ACPI; if necessary, use "pci=nocrs" and report a bug
[    0.222231] [Firmware Bug]: ACPI: No _BQC method, cannot determine initial brightness
[    0.226244] ACPI: PCI Root Bridge [PCI0] (domain 0000 [bus 00-ff])
[    0.226249] acpi PNP0A08:00: _OSC: OS supports [ExtendedConfig ASPM ClockPM Segments MSI]
[    0.226372] acpi PNP0A08:00: _OSC: platform does not support [PCIeHotplug PME]
[    0.226486] acpi PNP0A08:00: _OSC: OS now controls [AER PCIeCapability]
[    0.226495] acpi PNP0A08:00: [Firmware Info]: MMCONFIG for domain 0000 [bus 00-3f] only partially covers this bridge
[    0.226724] PCI host bridge to bus 0000:00
[    0.226726] pci_bus 0000:00: root bus resource [io  0x0000-0x03af window]
[    0.226728] pci_bus 0000:00: root bus resource [io  0x03e0-0x0cf7 window]
[    0.226729] pci_bus 0000:00: root bus resource [io  0x03b0-0x03df window]
[    0.226730] pci_bus 0000:00: root bus resource [io  0x0d00-0xffff window]
[    0.226732] pci_bus 0000:00: root bus resource [mem 0x000a0000-0x000bffff window]
[    0.226733] pci_bus 0000:00: root bus resource [mem 0x000c0000-0x000dffff window]
[    0.226735] pci_bus 0000:00: root bus resource [mem 0xe0000000-0xffffffff window]
[    0.226736] pci_bus 0000:00: root bus resource [bus 00-ff]
[    0.226742] pci 0000:00:00.0: [1022:1410] type 00 class 0x060000
[    0.226836] pci 0000:00:01.0: [1002:990e] type 00 class 0x030000
[    0.226852] pci 0000:00:01.0: reg 0x10: [mem 0xe0000000-0xefffffff pref]
[    0.226858] pci 0000:00:01.0: reg 0x14: [io  0xf000-0xf0ff]
[    0.226864] pci 0000:00:01.0: reg 0x18: [mem 0xfeb00000-0xfeb3ffff]
[    0.226898] pci 0000:00:01.0: supports D1 D2
[    0.226970] pci 0000:00:01.1: [1002:9902] type 00 class 0x040300
[    0.226986] pci 0000:00:01.1: reg 0x10: [mem 0xfeb44000-0xfeb47fff]
[    0.227027] pci 0000:00:01.1: supports D1 D2
[    0.227115] pci 0000:00:10.0: [1022:7814] type 00 class 0x0c0330
[    0.227154] pci 0000:00:10.0: reg 0x10: [mem 0xfeb48000-0xfeb49fff 64bit]
[    0.227236] pci 0000:00:10.0: PME# supported from D0 D3hot D3cold
[    0.227281] pci 0000:00:10.0: System wakeup disabled by ACPI
[    0.227325] pci 0000:00:11.0: [1022:7801] type 00 class 0x010601
[    0.227350] pci 0000:00:11.0: reg 0x10: [io  0xf140-0xf147]
[    0.227359] pci 0000:00:11.0: reg 0x14: [io  0xf130-0xf133]
[    0.227368] pci 0000:00:11.0: reg 0x18: [io  0xf120-0xf127]
[    0.227376] pci 0000:00:11.0: reg 0x1c: [io  0xf110-0xf113]
[    0.227385] pci 0000:00:11.0: reg 0x20: [io  0xf100-0xf10f]
[    0.227393] pci 0000:00:11.0: reg 0x24: [mem 0xfeb4e000-0xfeb4e7ff]
[    0.227484] pci 0000:00:12.0: [1022:7807] type 00 class 0x0c0310
[    0.227498] pci 0000:00:12.0: reg 0x10: [mem 0xfeb4d000-0xfeb4dfff]
[    0.227581] pci 0000:00:12.0: System wakeup disabled by ACPI
[    0.227619] pci 0000:00:12.2: [1022:7808] type 00 class 0x0c0320
[    0.227643] pci 0000:00:12.2: reg 0x10: [mem 0xfeb4c000-0xfeb4c0ff]
[    0.227709] pci 0000:00:12.2: supports D1 D2
[    0.227710] pci 0000:00:12.2: PME# supported from D0 D1 D2 D3hot D3cold
[    0.227746] pci 0000:00:12.2: System wakeup disabled by ACPI
[    0.227784] pci 0000:00:13.0: [1022:7807] type 00 class 0x0c0310
[    0.227799] pci 0000:00:13.0: reg 0x10: [mem 0xfeb4b000-0xfeb4bfff]
[    0.227881] pci 0000:00:13.0: System wakeup disabled by ACPI
[    0.227927] pci 0000:00:13.2: [1022:7808] type 00 class 0x0c0320
[    0.227951] pci 0000:00:13.2: reg 0x10: [mem 0xfeb4a000-0xfeb4a0ff]
[    0.228017] pci 0000:00:13.2: supports D1 D2
[    0.228018] pci 0000:00:13.2: PME# supported from D0 D1 D2 D3hot D3cold
[    0.228053] pci 0000:00:13.2: System wakeup disabled by ACPI
[    0.228091] pci 0000:00:14.0: [1022:780b] type 00 class 0x0c0500
[    0.228207] pci 0000:00:14.2: [1022:780d] type 00 class 0x040300
[    0.228231] pci 0000:00:14.2: reg 0x10: [mem 0xfeb40000-0xfeb43fff 64bit]
[    0.228287] pci 0000:00:14.2: PME# supported from D0 D3hot D3cold
[    0.228321] pci 0000:00:14.2: System wakeup disabled by ACPI
[    0.228354] pci 0000:00:14.3: [1022:780e] type 00 class 0x060100
[    0.228474] pci 0000:00:14.4: [1022:780f] type 01 class 0x060401
[    0.228536] pci 0000:00:14.4: System wakeup disabled by ACPI
[    0.228577] pci 0000:00:15.0: [1022:43a0] type 01 class 0x060400
[    0.228643] pci 0000:00:15.0: supports D1 D2
[    0.228645] pci 0000:00:15.0: PME# supported from D0 D3hot D3cold
[    0.228683] pci 0000:00:15.0: System wakeup disabled by ACPI
[    0.228722] pci 0000:00:15.1: [1022:43a1] type 01 class 0x060400
[    0.228788] pci 0000:00:15.1: supports D1 D2
[    0.228789] pci 0000:00:15.1: PME# supported from D0 D3hot D3cold
[    0.228828] pci 0000:00:15.1: System wakeup disabled by ACPI
[    0.228865] pci 0000:00:15.2: [1022:43a2] type 01 class 0x060400
[    0.228931] pci 0000:00:15.2: supports D1 D2
[    0.228932] pci 0000:00:15.2: PME# supported from D0 D3hot D3cold
[    0.228970] pci 0000:00:15.2: System wakeup disabled by ACPI
[    0.229008] pci 0000:00:18.0: [1022:1400] type 00 class 0x060000
[    0.229077] pci 0000:00:18.1: [1022:1401] type 00 class 0x060000
[    0.229155] pci 0000:00:18.2: [1022:1402] type 00 class 0x060000
[    0.229222] pci 0000:00:18.3: [1022:1403] type 00 class 0x060000
[    0.229294] pci 0000:00:18.4: [1022:1404] type 00 class 0x060000
[    0.229361] pci 0000:00:18.5: [1022:1405] type 00 class 0x060000
[    0.229501] pci 0000:00:14.4: PCI bridge to [bus 01] (subtractive decode)
[    0.229509] pci 0000:00:14.4:   bridge window [io  0x0000-0x03af window] (subtractive decode)
[    0.229511] pci 0000:00:14.4:   bridge window [io  0x03e0-0x0cf7 window] (subtractive decode)
[    0.229512] pci 0000:00:14.4:   bridge window [io  0x03b0-0x03df window] (subtractive decode)
[    0.229513] pci 0000:00:14.4:   bridge window [io  0x0d00-0xffff window] (subtractive decode)
[    0.229515] pci 0000:00:14.4:   bridge window [mem 0x000a0000-0x000bffff window] (subtractive decode)
[    0.229516] pci 0000:00:14.4:   bridge window [mem 0x000c0000-0x000dffff window] (subtractive decode)
[    0.229517] pci 0000:00:14.4:   bridge window [mem 0xe0000000-0xffffffff window] (subtractive decode)
[    0.229561] pci 0000:00:15.0: PCI bridge to [bus 02]
[    0.229631] pci 0000:03:00.0: [1b21:1142] type 00 class 0x0c0330
[    0.229677] pci 0000:03:00.0: reg 0x10: [mem 0xfea00000-0xfea07fff 64bit]
[    0.229794] pci 0000:03:00.0: PME# supported from D3cold
[    0.237850] pci 0000:00:15.1: PCI bridge to [bus 03]
[    0.237873] pci 0000:00:15.1:   bridge window [mem 0xfea00000-0xfeafffff]
[    0.237974] pci 0000:04:00.0: [10ec:8168] type 00 class 0x020000
[    0.238017] pci 0000:04:00.0: reg 0x10: [io  0xe000-0xe0ff]
[    0.238046] pci 0000:04:00.0: reg 0x18: [mem 0xfe900000-0xfe900fff 64bit]
[    0.238064] pci 0000:04:00.0: reg 0x20: [mem 0xf0000000-0xf0003fff 64bit pref]
[    0.238142] pci 0000:04:00.0: supports D1 D2
[    0.238144] pci 0000:04:00.0: PME# supported from D0 D1 D2 D3hot D3cold
[    0.238188] pci 0000:04:00.0: System wakeup disabled by ACPI
[    0.245852] pci 0000:00:15.2: PCI bridge to [bus 04]
[    0.245864] pci 0000:00:15.2:   bridge window [io  0xe000-0xefff]
[    0.245876] pci 0000:00:15.2:   bridge window [mem 0xfe900000-0xfe9fffff]
[    0.245881] pci 0000:00:15.2:   bridge window [mem 0xf0000000-0xf00fffff 64bit pref]
[    0.246336] ACPI: PCI Interrupt Link [LNKA] (IRQs 4 5 6 7 10 11 14 15) *0
[    0.246387] ACPI: PCI Interrupt Link [LNKB] (IRQs 4 5 6 7 10 11 14 15) *0
[    0.246438] ACPI: PCI Interrupt Link [LNKC] (IRQs 4 5 6 7 10 11 14 15) *0
[    0.246489] ACPI: PCI Interrupt Link [LNKD] (IRQs 4 10 11 14 15) *0
[    0.246530] ACPI: PCI Interrupt Link [LNKE] (IRQs 4 5 6 7 10 11 14 15) *0
[    0.246562] ACPI: PCI Interrupt Link [LNKF] (IRQs 4 5 6 7 10 11 14 15) *0
[    0.246594] ACPI: PCI Interrupt Link [LNKG] (IRQs 4 5 6 7 10 11 14 15) *0
[    0.246626] ACPI: PCI Interrupt Link [LNKH] (IRQs 4 5 6 7 10 11 14 15) *0
[    0.246856] vgaarb: setting as boot device: PCI:0000:00:01.0
[    0.246858] vgaarb: device added: PCI:0000:00:01.0,decodes=io+mem,owns=io+mem,locks=none
[    0.246862] vgaarb: loaded
[    0.246863] vgaarb: bridge control possible 0000:00:01.0
[    0.247091] SCSI subsystem initialized
[    0.247144] libata version 3.00 loaded.
[    0.247164] ACPI: bus type USB registered
[    0.247179] usbcore: registered new interface driver usbfs
[    0.247187] usbcore: registered new interface driver hub
[    0.247202] usbcore: registered new device driver usb
[    0.247327] PCI: Using ACPI for IRQ routing
[    0.249245] PCI: pci_cache_line_size set to 64 bytes
[    0.249299] e820: reserve RAM buffer [mem 0x0009e800-0x0009ffff]
[    0.249301] e820: reserve RAM buffer [mem 0xad184000-0xafffffff]
[    0.249302] e820: reserve RAM buffer [mem 0xae3e9000-0xafffffff]
[    0.249303] e820: reserve RAM buffer [mem 0xaea33000-0xafffffff]
[    0.249304] e820: reserve RAM buffer [mem 0xaf000000-0xafffffff]
[    0.249305] e820: reserve RAM buffer [mem 0x41f000000-0x41fffffff]
[    0.249405] NetLabel: Initializing
[    0.249407] NetLabel:  domain hash size = 128
[    0.249407] NetLabel:  protocols = UNLABELED CIPSOv4
[    0.249416] NetLabel:  unlabeled traffic allowed by default
[    0.249476] hpet0: at MMIO 0xfed00000, IRQs 2, 8, 0
[    0.249479] hpet0: 3 comparators, 32-bit 14.318180 MHz counter
[    0.251563] clocksource: Switched to clocksource hpet
[    0.257468] AppArmor: AppArmor Filesystem Enabled
[    0.257533] pnp: PnP ACPI init
[    0.257649] system 00:00: [mem 0xf8000000-0xfbffffff] has been reserved
[    0.257653] system 00:00: Plug and Play ACPI device, IDs PNP0c01 (active)
[    0.257719] system 00:01: [mem 0xb0000000-0xdfffffff] has been reserved
[    0.257721] system 00:01: Plug and Play ACPI device, IDs PNP0c02 (active)
[    0.257785] system 00:02: Plug and Play ACPI device, IDs PNP0c02 (active)
[    0.257923] system 00:03: [io  0x0300-0x031f] has been reserved
[    0.257924] system 00:03: [io  0x0290-0x029f] has been reserved
[    0.257926] system 00:03: [io  0x0230-0x023f] has been reserved
[    0.257928] system 00:03: Plug and Play ACPI device, IDs PNP0c02 (active)
[    0.257977] pnp 00:04: Plug and Play ACPI device, IDs PNP0b00 (active)
[    0.258073] system 00:05: [io  0x04d0-0x04d1] has been reserved
[    0.258076] system 00:05: Plug and Play ACPI device, IDs PNP0c02 (active)
[    0.258110] system 00:06: Plug and Play ACPI device, IDs PNP0c02 (active)
[    0.258307] pnp 00:07: [dma 0 disabled]
[    0.258345] pnp 00:07: Plug and Play ACPI device, IDs PNP0501 (active)
[    0.258548] system 00:08: [io  0x04d0-0x04d1] has been reserved
[    0.258549] system 00:08: [io  0x040b] has been reserved
[    0.258552] system 00:08: [io  0x04d6] has been reserved
[    0.258554] system 00:08: [io  0x0c00-0x0c01] has been reserved
[    0.258555] system 00:08: [io  0x0c14] has been reserved
[    0.258557] system 00:08: [io  0x0c50-0x0c51] has been reserved
[    0.258558] system 00:08: [io  0x0c52] has been reserved
[    0.258559] system 00:08: [io  0x0c6c] has been reserved
[    0.258561] system 00:08: [io  0x0c6f] has been reserved
[    0.258562] system 00:08: [io  0x0cd0-0x0cd1] has been reserved
[    0.258563] system 00:08: [io  0x0cd2-0x0cd3] has been reserved
[    0.258565] system 00:08: [io  0x0cd4-0x0cd5] has been reserved
[    0.258566] system 00:08: [io  0x0cd6-0x0cd7] has been reserved
[    0.258567] system 00:08: [io  0x0cd8-0x0cdf] has been reserved
[    0.258569] system 00:08: [io  0x0800-0x089f] could not be reserved
[    0.258570] system 00:08: [io  0x0b20-0x0b3f] has been reserved
[    0.258572] system 00:08: [io  0x0900-0x090f] has been reserved
[    0.258573] system 00:08: [io  0x0910-0x091f] has been reserved
[    0.258575] system 00:08: [io  0xfe00-0xfefe] has been reserved
[    0.258577] system 00:08: [mem 0xfec00000-0xfec00fff] could not be reserved
[    0.258578] system 00:08: [mem 0xfee00000-0xfee00fff] has been reserved
[    0.258580] system 00:08: [mem 0xfed80000-0xfed8ffff] has been reserved
[    0.258581] system 00:08: [mem 0xfed61000-0xfed70fff] has been reserved
[    0.258583] system 00:08: [mem 0xfec10000-0xfec10fff] has been reserved
[    0.258585] system 00:08: [mem 0xff000000-0xffffffff] has been reserved
[    0.258586] system 00:08: Plug and Play ACPI device, IDs PNP0c02 (active)
[    0.258727] pnp: PnP ACPI: found 9 devices
[    0.265009] clocksource: acpi_pm: mask: 0xffffff max_cycles: 0xffffff, max_idle_ns: 2085701024 ns
[    0.265038] pci 0000:00:14.4: PCI bridge to [bus 01]
[    0.265050] pci 0000:00:15.0: PCI bridge to [bus 02]
[    0.265059] pci 0000:00:15.1: PCI bridge to [bus 03]
[    0.265063] pci 0000:00:15.1:   bridge window [mem 0xfea00000-0xfeafffff]
[    0.265070] pci 0000:00:15.2: PCI bridge to [bus 04]
[    0.265072] pci 0000:00:15.2:   bridge window [io  0xe000-0xefff]
[    0.265076] pci 0000:00:15.2:   bridge window [mem 0xfe900000-0xfe9fffff]
[    0.265079] pci 0000:00:15.2:   bridge window [mem 0xf0000000-0xf00fffff 64bit pref]
[    0.265085] pci_bus 0000:00: resource 4 [io  0x0000-0x03af window]
[    0.265086] pci_bus 0000:00: resource 5 [io  0x03e0-0x0cf7 window]
[    0.265088] pci_bus 0000:00: resource 6 [io  0x03b0-0x03df window]
[    0.265089] pci_bus 0000:00: resource 7 [io  0x0d00-0xffff window]
[    0.265090] pci_bus 0000:00: resource 8 [mem 0x000a0000-0x000bffff window]
[    0.265092] pci_bus 0000:00: resource 9 [mem 0x000c0000-0x000dffff window]
[    0.265093] pci_bus 0000:00: resource 10 [mem 0xe0000000-0xffffffff window]
[    0.265095] pci_bus 0000:01: resource 4 [io  0x0000-0x03af window]
[    0.265096] pci_bus 0000:01: resource 5 [io  0x03e0-0x0cf7 window]
[    0.265097] pci_bus 0000:01: resource 6 [io  0x03b0-0x03df window]
[    0.265098] pci_bus 0000:01: resource 7 [io  0x0d00-0xffff window]
[    0.265100] pci_bus 0000:01: resource 8 [mem 0x000a0000-0x000bffff window]
[    0.265101] pci_bus 0000:01: resource 9 [mem 0x000c0000-0x000dffff window]
[    0.265102] pci_bus 0000:01: resource 10 [mem 0xe0000000-0xffffffff window]
[    0.265104] pci_bus 0000:03: resource 1 [mem 0xfea00000-0xfeafffff]
[    0.265105] pci_bus 0000:04: resource 0 [io  0xe000-0xefff]
[    0.265106] pci_bus 0000:04: resource 1 [mem 0xfe900000-0xfe9fffff]
[    0.265107] pci_bus 0000:04: resource 2 [mem 0xf0000000-0xf00fffff 64bit pref]
[    0.265133] NET: Registered protocol family 2
[    0.265318] TCP established hash table entries: 131072 (order: 8, 1048576 bytes)
[    0.265569] TCP bind hash table entries: 65536 (order: 8, 1048576 bytes)
[    0.265768] TCP: Hash tables configured (established 131072 bind 65536)
[    0.265811] UDP hash table entries: 8192 (order: 6, 262144 bytes)
[    0.265873] UDP-Lite hash table entries: 8192 (order: 6, 262144 bytes)
[    0.265981] NET: Registered protocol family 1
[    0.265998] pci 0000:00:01.0: Video device with shadowed ROM
[    0.459922] PCI: CLS 64 bytes, default 64
[    0.459973] Trying to unpack rootfs image as initramfs...
[    0.702072] Freeing initrd memory: 20600K (ffff8800357b4000 - ffff880036bd2000)
[    0.702094] PCI-DMA: Using software bounce buffering for IO (SWIOTLB)
[    0.702096] software IO TLB [mem 0xa9184000-0xad184000] (64MB) mapped at [ffff8800a9184000-ffff8800ad183fff]
[    0.702137] perf: AMD NB counters detected
[    0.702208] LVT offset 0 assigned for vector 0x400
[    0.702220] perf: AMD IBS detected (0x000000ff)
[    0.702298] Scanning for low memory corruption every 60 seconds
[    0.702631] futex hash table entries: 1024 (order: 4, 65536 bytes)
[    0.702662] audit: initializing netlink subsys (disabled)
[    0.702678] audit: type=2000 audit(1474476014.592:1): initialized
[    0.702953] Initialise system trusted keyring
[    0.703102] HugeTLB registered 1 GB page size, pre-allocated 0 pages
[    0.703104] HugeTLB registered 2 MB page size, pre-allocated 0 pages
[    0.704581] zbud: loaded
[    0.704804] VFS: Disk quotas dquot_6.6.0
[    0.704833] VFS: Dquot-cache hash table entries: 512 (order 0, 4096 bytes)
[    0.705083] squashfs: version 4.0 (2009/01/31) Phillip Lougher
[    0.705362] fuse init (API version 7.23)
[    0.705478] Key type big_key registered
[    0.705488] Allocating IMA MOK and blacklist keyrings.
[    0.705919] Key type asymmetric registered
[    0.705921] Asymmetric key parser 'x509' registered
[    0.705952] Block layer SCSI generic (bsg) driver version 0.4 loaded (major 249)
[    0.705982] io scheduler noop registered
[    0.705984] io scheduler deadline registered (default)
[    0.706010] io scheduler cfq registered
[    0.706368] pci_hotplug: PCI Hot Plug PCI Core version: 0.5
[    0.706373] pciehp: PCI Express Hot Plug Controller Driver version: 0.4
[    0.706400] vesafb: mode is 1280x1024x32, linelength=5120, pages=0
[    0.706401] vesafb: scrolling: redraw
[    0.706402] vesafb: Truecolor: size=0:8:8:8, shift=0:16:8:0
[    0.706412] vesafb: framebuffer at 0xe0000000, mapped to 0xffffc90002000000, using 5120k, total 5120k
[    0.706495] Console: switching to colour frame buffer device 160x64
[    0.706512] fb0: VESA VGA frame buffer device
[    0.706582] input: Power Button as /devices/LNXSYSTM:00/LNXSYBUS:00/PNP0C0C:00/input/input0
[    0.706585] ACPI: Power Button [PWRB]
[    0.706617] input: Power Button as /devices/LNXSYSTM:00/LNXPWRBN:00/input/input1
[    0.706619] ACPI: Power Button [PWRF]
[    0.706648] ACPI: acpi_idle registered with cpuidle
[    0.707067] GHES: HEST is not enabled!
[    0.707164] Serial: 8250/16550 driver, 32 ports, IRQ sharing enabled
[    0.727537] 00:07: ttyS0 at I/O 0x3f8 (irq = 4, base_baud = 115200) is a 16550A
[    0.729093] Linux agpgart interface v0.103
[    0.731939] brd: module loaded
[    0.733063] loop: module loaded
[    0.733260] libphy: Fixed MDIO Bus: probed
[    0.733262] tun: Universal TUN/TAP device driver, 1.6
[    0.733264] tun: (C) 1999-2004 Max Krasnyansky <maxk@qualcomm.com>
[    0.733300] PPP generic driver version 2.4.2
[    0.733353] ehci_hcd: USB 2.0 'Enhanced' Host Controller (EHCI) Driver
[    0.733357] ehci-pci: EHCI PCI platform driver
[    0.733466] QUIRK: Enable AMD PLL fix
[    0.733488] ehci-pci 0000:00:12.2: EHCI Host Controller
[    0.733493] ehci-pci 0000:00:12.2: new USB bus registered, assigned bus number 1
[    0.733496] ehci-pci 0000:00:12.2: applying AMD SB700/SB800/Hudson-2/3 EHCI dummy qh workaround
[    0.733505] ehci-pci 0000:00:12.2: debug port 1
[    0.733543] ehci-pci 0000:00:12.2: irq 17, io mem 0xfeb4c000
[    0.743693] ehci-pci 0000:00:12.2: USB 2.0 started, EHCI 1.00
[    0.743754] usb usb1: New USB device found, idVendor=1d6b, idProduct=0002
[    0.743756] usb usb1: New USB device strings: Mfr=3, Product=2, SerialNumber=1
[    0.743757] usb usb1: Product: EHCI Host Controller
[    0.743759] usb usb1: Manufacturer: Linux 4.4.0-36-generic ehci_hcd
[    0.743760] usb usb1: SerialNumber: 0000:00:12.2
[    0.743891] hub 1-0:1.0: USB hub found
[    0.743895] hub 1-0:1.0: 5 ports detected
[    0.744128] ehci-pci 0000:00:13.2: EHCI Host Controller
[    0.744133] ehci-pci 0000:00:13.2: new USB bus registered, assigned bus number 2
[    0.744136] ehci-pci 0000:00:13.2: applying AMD SB700/SB800/Hudson-2/3 EHCI dummy qh workaround
[    0.744144] ehci-pci 0000:00:13.2: debug port 1
[    0.744169] ehci-pci 0000:00:13.2: irq 17, io mem 0xfeb4a000
[    0.755702] ehci-pci 0000:00:13.2: USB 2.0 started, EHCI 1.00
[    0.755767] usb usb2: New USB device found, idVendor=1d6b, idProduct=0002
[    0.755769] usb usb2: New USB device strings: Mfr=3, Product=2, SerialNumber=1
[    0.755770] usb usb2: Product: EHCI Host Controller
[    0.755772] usb usb2: Manufacturer: Linux 4.4.0-36-generic ehci_hcd
[    0.755773] usb usb2: SerialNumber: 0000:00:13.2
[    0.755900] hub 2-0:1.0: USB hub found
[    0.755905] hub 2-0:1.0: 5 ports detected
[    0.756027] ehci-platform: EHCI generic platform driver
[    0.756041] ohci_hcd: USB 1.1 'Open' Host Controller (OHCI) Driver
[    0.756044] ohci-pci: OHCI PCI platform driver
[    0.756168] ohci-pci 0000:00:12.0: OHCI PCI host controller
[    0.756172] ohci-pci 0000:00:12.0: new USB bus registered, assigned bus number 3
[    0.756192] ohci-pci 0000:00:12.0: irq 18, io mem 0xfeb4d000
[    0.815724] usb usb3: New USB device found, idVendor=1d6b, idProduct=0001
[    0.815728] usb usb3: New USB device strings: Mfr=3, Product=2, SerialNumber=1
[    0.815729] usb usb3: Product: OHCI PCI host controller
[    0.815731] usb usb3: Manufacturer: Linux 4.4.0-36-generic ohci_hcd
[    0.815732] usb usb3: SerialNumber: 0000:00:12.0
[    0.815927] hub 3-0:1.0: USB hub found
[    0.815933] hub 3-0:1.0: 5 ports detected
[    0.816159] ohci-pci 0000:00:13.0: OHCI PCI host controller
[    0.816163] ohci-pci 0000:00:13.0: new USB bus registered, assigned bus number 4
[    0.816184] ohci-pci 0000:00:13.0: irq 18, io mem 0xfeb4b000
[    0.875732] usb usb4: New USB device found, idVendor=1d6b, idProduct=0001
[    0.875735] usb usb4: New USB device strings: Mfr=3, Product=2, SerialNumber=1
[    0.875737] usb usb4: Product: OHCI PCI host controller
[    0.875738] usb usb4: Manufacturer: Linux 4.4.0-36-generic ohci_hcd
[    0.875739] usb usb4: SerialNumber: 0000:00:13.0
[    0.875934] hub 4-0:1.0: USB hub found
[    0.875941] hub 4-0:1.0: 5 ports detected
[    0.876069] ohci-platform: OHCI generic platform driver
[    0.876082] uhci_hcd: USB Universal Host Controller Interface driver
[    0.876226] xhci_hcd 0000:00:10.0: xHCI Host Controller
[    0.876230] xhci_hcd 0000:00:10.0: new USB bus registered, assigned bus number 5
[    0.876409] xhci_hcd 0000:00:10.0: hcc params 0x014040c3 hci version 0x100 quirks 0x00000418
[    0.876531] usb usb5: New USB device found, idVendor=1d6b, idProduct=0002
[    0.876533] usb usb5: New USB device strings: Mfr=3, Product=2, SerialNumber=1
[    0.876534] usb usb5: Product: xHCI Host Controller
[    0.876535] usb usb5: Manufacturer: Linux 4.4.0-36-generic xhci-hcd
[    0.876537] usb usb5: SerialNumber: 0000:00:10.0
[    0.876690] hub 5-0:1.0: USB hub found
[    0.876696] hub 5-0:1.0: 2 ports detected
[    0.876761] xhci_hcd 0000:00:10.0: xHCI Host Controller
[    0.876763] xhci_hcd 0000:00:10.0: new USB bus registered, assigned bus number 6
[    0.879826] usb usb6: We don't know the algorithms for LPM for this host, disabling LPM.
[    0.879838] usb usb6: New USB device found, idVendor=1d6b, idProduct=0003
[    0.879840] usb usb6: New USB device strings: Mfr=3, Product=2, SerialNumber=1
[    0.879841] usb usb6: Product: xHCI Host Controller
[    0.879842] usb usb6: Manufacturer: Linux 4.4.0-36-generic xhci-hcd
[    0.879843] usb usb6: SerialNumber: 0000:00:10.0
[    0.879947] hub 6-0:1.0: USB hub found
[    0.879953] hub 6-0:1.0: 2 ports detected
[    0.880055] xhci_hcd 0000:03:00.0: xHCI Host Controller
[    0.880059] xhci_hcd 0000:03:00.0: new USB bus registered, assigned bus number 7
[    0.940427] xhci_hcd 0000:03:00.0: hcc params 0x0200e081 hci version 0x100 quirks 0x00000010
[    0.940535] usb usb7: New USB device found, idVendor=1d6b, idProduct=0002
[    0.940536] usb usb7: New USB device strings: Mfr=3, Product=2, SerialNumber=1
[    0.940538] usb usb7: Product: xHCI Host Controller
[    0.940539] usb usb7: Manufacturer: Linux 4.4.0-36-generic xhci-hcd
[    0.940540] usb usb7: SerialNumber: 0000:03:00.0
[    0.940692] hub 7-0:1.0: USB hub found
[    0.940698] hub 7-0:1.0: 2 ports detected
[    0.940765] xhci_hcd 0000:03:00.0: xHCI Host Controller
[    0.940767] xhci_hcd 0000:03:00.0: new USB bus registered, assigned bus number 8
[    0.940802] usb usb8: We don't know the algorithms for LPM for this host, disabling LPM.
[    0.940815] usb usb8: New USB device found, idVendor=1d6b, idProduct=0003
[    0.940816] usb usb8: New USB device strings: Mfr=3, Product=2, SerialNumber=1
[    0.940817] usb usb8: Product: xHCI Host Controller
[    0.940818] usb usb8: Manufacturer: Linux 4.4.0-36-generic xhci-hcd
[    0.940819] usb usb8: SerialNumber: 0000:03:00.0
[    0.940900] hub 8-0:1.0: USB hub found
[    0.940906] hub 8-0:1.0: 2 ports detected
[    0.941000] i8042: PNP: No PS/2 controller found. Probing ports directly.
[    0.941419] serio: i8042 KBD port at 0x60,0x64 irq 1
[    0.941422] serio: i8042 AUX port at 0x60,0x64 irq 12
[    0.941569] mousedev: PS/2 mouse device common for all mice
[    0.941699] rtc_cmos 00:04: RTC can wake from S4
[    0.941803] rtc_cmos 00:04: rtc core: registered rtc_cmos as rtc0
[    0.941822] rtc_cmos 00:04: alarms up to one month, y3k, 114 bytes nvram, hpet irqs
[    0.941829] i2c /dev entries driver
[    0.941876] device-mapper: uevent: version 1.0.3
[    0.941932] device-mapper: ioctl: 4.34.0-ioctl (2015-10-28) initialised: dm-devel@redhat.com
[    0.941949] ledtrig-cpu: registered to indicate activity on CPUs
[    0.942295] NET: Registered protocol family 10
[    0.942457] NET: Registered protocol family 17
[    0.942468] Key type dns_resolver registered
[    0.942730] microcode: CPU0: patch_level=0x06001119
[    0.942736] microcode: CPU1: patch_level=0x06001119
[    0.942746] microcode: CPU2: patch_level=0x06001119
[    0.942753] microcode: CPU3: patch_level=0x06001119
[    0.942787] microcode: Microcode Update Driver: v2.01 <tigran@aivazian.fsnet.co.uk>, Peter Oruba
[    0.942934] registered taskstats version 1
[    0.942944] Loading compiled-in X.509 certificates
[    0.943663] Loaded X.509 cert 'Build time autogenerated kernel key: 6d1a628313c57c2b040732ed4f3ff204c35686cf'
[    0.943690] zswap: loaded using pool lzo/zbud
[    0.945460] Key type trusted registered
[    0.948282] Key type encrypted registered
[    0.948290] AppArmor: AppArmor sha1 policy hashing enabled
[    0.948293] ima: No TPM chip found, activating TPM-bypass!
[    0.948315] evm: HMAC attrs: 0x1
[    0.948607]   Magic number: 0:427:692
[    0.948700] rtc_cmos 00:04: setting system clock to 2016-09-21 16:40:15 UTC (1474476015)
[    0.948842] acpi-cpufreq: overriding BIOS provided _PSD data
[    0.948969] BIOS EDD facility v0.16 2004-Jun-25, 0 devices found
[    0.948970] EDD information not available.
[    0.949036] PM: Hibernation image not present or could not be loaded.
[    0.949894] Freeing unused kernel memory: 1484K (ffffffff81d45000 - ffffffff81eb8000)
[    0.949896] Write protecting the kernel read-only data: 12288k
[    0.950074] Freeing unused kernel memory: 4K (ffff8800017ff000 - ffff880001800000)
[    0.950346] Freeing unused kernel memory: 136K (ffff880001bde000 - ffff880001c00000)
[    0.961233] systemd-udevd[134]: starting version 204
[    0.970473] FUJITSU Extended Socket Network Device Driver - version 1.0 - Copyright (c) 2015 FUJITSU LIMITED
[    0.979168] ahci 0000:00:11.0: version 3.0
[    0.979394] ahci 0000:00:11.0: AHCI 0001.0300 32 slots 6 ports 6 Gbps 0x3f impl SATA mode
[    0.979397] ahci 0000:00:11.0: flags: 64bit ncq sntf ilck pm led clo pmp pio 
[    0.980325] scsi host0: ahci
[    0.980484] scsi host1: ahci
[    0.980641] scsi host2: ahci
[    0.980735] scsi host3: ahci
[    0.980921] r8169 Gigabit Ethernet driver 2.3LK-NAPI loaded
[    0.981267] scsi host4: ahci
[    0.981363] scsi host5: ahci
[    0.981409] ata1: SATA max UDMA/133 abar m2048@0xfeb4e000 port 0xfeb4e100 irq 34
[    0.981412] ata2: SATA max UDMA/133 abar m2048@0xfeb4e000 port 0xfeb4e180 irq 34
[    0.981414] ata3: SATA max UDMA/133 abar m2048@0xfeb4e000 port 0xfeb4e200 irq 34
[    0.981415] ata4: SATA max UDMA/133 abar m2048@0xfeb4e000 port 0xfeb4e280 irq 34
[    0.981417] ata5: SATA max UDMA/133 abar m2048@0xfeb4e000 port 0xfeb4e300 irq 34
[    0.981419] ata6: SATA max UDMA/133 abar m2048@0xfeb4e000 port 0xfeb4e380 irq 34
[    0.988668] r8169 0000:04:00.0 eth0: RTL8168g/8111g at 0xffffc90001b52000, 14:dd:a9:79:64:8d, XID 0c000800 IRQ 35
[    0.988672] r8169 0000:04:00.0 eth0: jumbo features [frames: 9200 bytes, tx checksumming: ko]
[    1.187739] usb 5-2: new high-speed USB device number 2 using xhci_hcd
[    1.299789] ata4: SATA link down (SStatus 0 SControl 300)
[    1.299817] ata3: SATA link down (SStatus 0 SControl 300)
[    1.299842] ata6: SATA link down (SStatus 0 SControl 300)
[    1.299886] ata5: SATA link down (SStatus 0 SControl 300)
[    1.307765] usb 7-1: new full-speed USB device number 2 using xhci_hcd
[    1.329081] usb 5-2: New USB device found, idVendor=05e3, idProduct=0745
[    1.329085] usb 5-2: New USB device strings: Mfr=0, Product=1, SerialNumber=2
[    1.329087] usb 5-2: Product: USB Storage
[    1.329088] usb 5-2: SerialNumber: 000000000903
[    1.332594] usb-storage 5-2:1.0: USB Mass Storage device detected
[    1.332730] scsi host6: usb-storage 5-2:1.0
[    1.332796] usbcore: registered new interface driver usb-storage
[    1.333891] usbcore: registered new interface driver uas
[    1.471778] ata2: SATA link up 1.5 Gbps (SStatus 113 SControl 300)
[    1.471814] ata1: SATA link up 6.0 Gbps (SStatus 133 SControl 300)
[    1.472477] ata1.00: ATA-9: WDC WD10EZEX-00BN5A0, 01.01A01, max UDMA/133
[    1.472481] ata1.00: 1953525168 sectors, multi 16: LBA48 NCQ (depth 31/32), AA
[    1.473200] ata1.00: configured for UDMA/133
[    1.473384] scsi 0:0:0:0: Direct-Access     ATA      WDC WD10EZEX-00B 1A01 PQ: 0 ANSI: 5
[    1.473590] sd 0:0:0:0: [sda] 1953525168 512-byte logical blocks: (1.00 TB/932 GiB)
[    1.473592] sd 0:0:0:0: [sda] 4096-byte physical blocks
[    1.473631] sd 0:0:0:0: Attached scsi generic sg0 type 0
[    1.473651] sd 0:0:0:0: [sda] Write Protect is off
[    1.473655] sd 0:0:0:0: [sda] Mode Sense: 00 3a 00 00
[    1.473686] sd 0:0:0:0: [sda] Write cache: enabled, read cache: enabled, doesn't support DPO or FUA
[    1.473962] ata2.00: ATAPI: ASUS     DRW-24F1ST   a, 1.00, max UDMA/100
[    1.475082] ata2.00: configured for UDMA/100
[    1.476668] scsi 1:0:0:0: CD-ROM            ASUS     DRW-24F1ST   a   1.00 PQ: 0 ANSI: 5
[    1.495832] sr 1:0:0:0: [sr0] scsi3-mmc drive: 48x/48x writer dvd-ram cd/rw xa/form2 cdda tray
[    1.495836] cdrom: Uniform CD-ROM driver Revision: 3.20
[    1.496009] sr 1:0:0:0: Attached scsi CD-ROM sr0
[    1.496097] sr 1:0:0:0: Attached scsi generic sg1 type 5
[    1.520975] usb 7-1: New USB device found, idVendor=058f, idProduct=9254
[    1.520978] usb 7-1: New USB device strings: Mfr=1, Product=2, SerialNumber=0
[    1.520980] usb 7-1: Product: Generic USB Hub
[    1.520981] usb 7-1: Manufacturer: ALCOR
[    1.521107] usb 7-1: ep 0x81 - rounding interval to 1024 microframes, ep desc says 2040 microframes
[    1.522521] hub 7-1:1.0: USB hub found
[    1.522836] hub 7-1:1.0: 4 ports detected
[    1.540454]  sda: sda1 sda2 < sda5 >
[    1.540940] sd 0:0:0:0: [sda] Attached SCSI disk
[    1.699788] tsc: Refined TSC clocksource calibration: 3890.257 MHz
[    1.699793] clocksource: tsc: mask: 0xffffffffffffffff max_cycles: 0x7026cd4cb37, max_idle_ns: 881590669116 ns
[    1.795746] usb 7-1.1: new low-speed USB device number 3 using xhci_hcd
[    1.916222] usb 7-1.1: New USB device found, idVendor=413c, idProduct=2005
[    1.916226] usb 7-1.1: New USB device strings: Mfr=1, Product=2, SerialNumber=0
[    1.916228] usb 7-1.1: Product: DELL USB Keyboard
[    1.916229] usb 7-1.1: Manufacturer: DELL
[    1.916356] usb 7-1.1: ep 0x81 - rounding interval to 64 microframes, ep desc says 80 microframes
[    1.920505] hidraw: raw HID events driver (C) Jiri Kosina
[    1.925107] usbcore: registered new interface driver usbhid
[    1.925117] usbhid: USB HID core driver
[    1.926573] input: DELL DELL USB Keyboard as /devices/pci0000:00/0000:00:15.1/0000:03:00.0/usb7/7-1/7-1.1/7-1.1:1.0/0003:413C:2005.0001/input/input5
[    1.979951] hid-generic 0003:413C:2005.0001: input,hidraw0: USB HID v1.10 Keyboard [DELL DELL USB Keyboard] on usb-0000:03:00.0-1.1/input0
[    1.987761] usb 7-1.3: new full-speed USB device number 4 using xhci_hcd
[    1.989414] EXT4-fs (sda1): mounted filesystem with ordered data mode. Opts: (null)
[    2.102867] usb 7-1.3: not running at top speed; connect to a high speed hub
[    2.104214] usb 7-1.3: New USB device found, idVendor=04f9, idProduct=003a
[    2.104217] usb 7-1.3: New USB device strings: Mfr=1, Product=2, SerialNumber=3
[    2.104219] usb 7-1.3: Product: HL-5350DN series
[    2.104220] usb 7-1.3: Manufacturer: Brother
[    2.104221] usb 7-1.3: SerialNumber: K0J807812
[    2.179821] usb 7-1.4: new low-speed USB device number 5 using xhci_hcd
[    2.299868] usb 7-1.4: New USB device found, idVendor=093a, idProduct=2510
[    2.299871] usb 7-1.4: New USB device strings: Mfr=1, Product=2, SerialNumber=0
[    2.299873] usb 7-1.4: Product: USB OPTICAL MOUSE
[    2.299874] usb 7-1.4: Manufacturer: PIXART
[    2.299981] usb 7-1.4: ep 0x81 - rounding interval to 64 microframes, ep desc says 80 microframes
[    2.305562] input: PIXART USB OPTICAL MOUSE as /devices/pci0000:00/0000:00:15.1/0000:03:00.0/usb7/7-1/7-1.4/7-1.4:1.0/0003:093A:2510.0002/input/input6
[    2.305696] hid-generic 0003:093A:2510.0002: input,hidraw1: USB HID v1.11 Mouse [PIXART USB OPTICAL MOUSE] on usb-0000:03:00.0-1.4/input0
[    2.333664] scsi 6:0:0:0: Direct-Access     Generic  STORAGE DEVICE   0903 PQ: 0 ANSI: 6
[    2.333971] sd 6:0:0:0: Attached scsi generic sg2 type 0
[    2.336417] sd 6:0:0:0: [sdb] Attached SCSI removable disk
[    2.452021] random: init urandom read with 96 bits of entropy available
[    2.699923] clocksource: Switched to clocksource tsc
[    2.899159] random: nonblocking pool is initialized
[    2.952910] init: plymouth-upstart-bridge main process (218) terminated with status 1
[    2.952921] init: plymouth-upstart-bridge main process ended, respawning
[    2.954809] init: plymouth-upstart-bridge main process (232) terminated with status 1
[    2.954819] init: plymouth-upstart-bridge main process ended, respawning
[    2.956210] init: plymouth-upstart-bridge main process (234) terminated with status 1
[    2.956220] init: plymouth-upstart-bridge main process ended, respawning
[    2.958298] init: plymouth-upstart-bridge main process (235) terminated with status 1
[    2.958308] init: plymouth-upstart-bridge main process ended, respawning
[    2.959614] init: plymouth-upstart-bridge main process (237) terminated with status 1
[    2.959623] init: plymouth-upstart-bridge main process ended, respawning
[    2.961257] init: plymouth-upstart-bridge main process (238) terminated with status 1
[    2.961265] init: plymouth-upstart-bridge main process ended, respawning
[    2.963153] init: plymouth-upstart-bridge main process (240) terminated with status 1
[    2.963164] init: plymouth-upstart-bridge main process ended, respawning
[    2.965046] init: plymouth-upstart-bridge main process (241) terminated with status 1
[    2.965055] init: plymouth-upstart-bridge main process ended, respawning
[    2.966401] init: plymouth-upstart-bridge main process (243) terminated with status 1
[    2.966409] init: plymouth-upstart-bridge main process ended, respawning
[    2.968233] init: plymouth-upstart-bridge main process (244) terminated with status 1
[    2.968241] init: plymouth-upstart-bridge main process ended, respawning
[    2.969738] init: plymouth-upstart-bridge main process (246) terminated with status 1
[    2.969746] init: plymouth-upstart-bridge respawning too fast, stopped
[    8.583716] Adding 15929340k swap on /dev/sda5.  Priority:-1 extents:1 across:15929340k FS
[    8.925974] systemd-udevd[351]: starting version 204
[    9.262428] lp: driver loaded but no devices found
[    9.269077] ppdev: user-space parallel port driver
[    9.271071] ACPI: Video Device [VGA] (multi-head: yes  rom: no  post: no)
[    9.271102] [Firmware Bug]: ACPI: No _BQC method, cannot determine initial brightness
[    9.271168] ACPI Error: [AFN7] Namespace lookup failure, AE_NOT_FOUND (20150930/psargs-359)
[    9.271173] ACPI Error: Method parse/execution failed [\_SB.PCI0.VGA.LCD._BCM] (Node ffff88040eca5c30), AE_NOT_FOUND (20150930/psparse-542)
[    9.271179] ACPI Error: Evaluating _BCM failed (20150930/video-367)
[    9.271253] input: Video Bus as /devices/LNXSYSTM:00/LNXSYBUS:00/PNP0A08:00/LNXVIDEO:00/input/input7
[    9.312998] piix4_smbus 0000:00:14.0: SMBus Host Controller at 0xb00, revision 0
[    9.313530] piix4_smbus 0000:00:14.0: Auxiliary SMBus Host Controller at 0xb20
[    9.323675] shpchp: Standard Hot Plug PCI Controller Driver version: 0.4
[    9.324863] wmi: Mapper loaded
[    9.394662] AVX version of gcm_enc/dec engaged.
[    9.394665] AES CTR mode by8 optimization enabled
[    9.403681] [drm] Initialized drm 1.1.0 20060810
[    9.489302] asus_wmi: ASUS WMI generic driver loaded
[    9.490552] asus_wmi: Initialization: 0x0
[    9.490575] asus_wmi: BIOS WMI version: 0.9
[    9.490620] asus_wmi: SFUN value: 0x0
[    9.490938] input: Eee PC WMI hotkeys as /devices/platform/eeepc-wmi/input/input8
[    9.491188] asus_wmi: Number of fans: 1
[    9.543371] kvm: Nested Virtualization enabled
[    9.543375] kvm: Nested Paging enabled
[    9.547806] snd_hda_intel 0000:00:01.1: Force to non-snoop mode
[    9.557468] input: HDA ATI HDMI HDMI/DP,pcm=3 as /devices/pci0000:00/0000:00:01.1/sound/card0/input9
[    9.561406] snd_hda_codec_realtek hdaudioC1D0: autoconfig for ALC887-VD: line_outs=1 (0x14/0x0/0x0/0x0/0x0) type:line
[    9.561409] snd_hda_codec_realtek hdaudioC1D0:    speaker_outs=0 (0x0/0x0/0x0/0x0/0x0)
[    9.561412] snd_hda_codec_realtek hdaudioC1D0:    hp_outs=1 (0x1b/0x0/0x0/0x0/0x0)
[    9.561413] snd_hda_codec_realtek hdaudioC1D0:    mono: mono_out=0x0
[    9.561414] snd_hda_codec_realtek hdaudioC1D0:    dig-out=0x11/0x0
[    9.561415] snd_hda_codec_realtek hdaudioC1D0:    inputs:
[    9.561420] snd_hda_codec_realtek hdaudioC1D0:      Front Mic=0x19
[    9.561422] snd_hda_codec_realtek hdaudioC1D0:      Rear Mic=0x18
[    9.561423] snd_hda_codec_realtek hdaudioC1D0:      Line=0x1a
[    9.574737] input: HD-Audio Generic Front Mic as /devices/pci0000:00/0000:00:14.2/sound/card1/input10
[    9.574831] input: HD-Audio Generic Rear Mic as /devices/pci0000:00/0000:00:14.2/sound/card1/input11
[    9.574905] input: HD-Audio Generic Line as /devices/pci0000:00/0000:00:14.2/sound/card1/input12
[    9.575002] input: HD-Audio Generic Line Out as /devices/pci0000:00/0000:00:14.2/sound/card1/input13
[    9.575150] input: HD-Audio Generic Front Headphone as /devices/pci0000:00/0000:00:14.2/sound/card1/input14
[   10.128077] audit: type=1400 audit(1474476024.677:2): apparmor="STATUS" operation="profile_load" profile="unconfined" name="/sbin/dhclient" pid=463 comm="apparmor_parser"
[   10.128083] audit: type=1400 audit(1474476024.677:3): apparmor="STATUS" operation="profile_load" profile="unconfined" name="/usr/lib/NetworkManager/nm-dhcp-client.action" pid=463 comm="apparmor_parser"
[   10.128086] audit: type=1400 audit(1474476024.677:4): apparmor="STATUS" operation="profile_load" profile="unconfined" name="/usr/lib/connman/scripts/dhclient-script" pid=463 comm="apparmor_parser"
[   10.128098] audit: type=1400 audit(1474476024.677:5): apparmor="STATUS" operation="profile_replace" profile="unconfined" name="/sbin/dhclient" pid=456 comm="apparmor_parser"
[   10.128106] audit: type=1400 audit(1474476024.677:6): apparmor="STATUS" operation="profile_replace" profile="unconfined" name="/usr/lib/NetworkManager/nm-dhcp-client.action" pid=456 comm="apparmor_parser"
[   10.128109] audit: type=1400 audit(1474476024.677:7): apparmor="STATUS" operation="profile_replace" profile="unconfined" name="/usr/lib/connman/scripts/dhclient-script" pid=456 comm="apparmor_parser"
[   10.128401] audit: type=1400 audit(1474476024.677:8): apparmor="STATUS" operation="profile_replace" profile="unconfined" name="/usr/lib/NetworkManager/nm-dhcp-client.action" pid=463 comm="apparmor_parser"
[   10.128406] audit: type=1400 audit(1474476024.677:9): apparmor="STATUS" operation="profile_replace" profile="unconfined" name="/usr/lib/connman/scripts/dhclient-script" pid=463 comm="apparmor_parser"
[   10.128424] audit: type=1400 audit(1474476024.677:10): apparmor="STATUS" operation="profile_replace" profile="unconfined" name="/usr/lib/NetworkManager/nm-dhcp-client.action" pid=456 comm="apparmor_parser"
[   10.128428] audit: type=1400 audit(1474476024.677:11): apparmor="STATUS" operation="profile_replace" profile="unconfined" name="/usr/lib/connman/scripts/dhclient-script" pid=456 comm="apparmor_parser"
[   10.403928] [drm] radeon kernel modesetting enabled.
[   10.406508] AMD IOMMUv2 driver by Joerg Roedel <jroedel@suse.de>
[   10.406511] AMD IOMMUv2 functionality not available on this system
[   10.468278] Found CRAT image with size=1400
[   10.468281] Parsing CRAT table with 1 nodes
[   10.468283] Found CU entry in CRAT table with proximity_domain=0 caps=0
[   10.468284] CU CPU: cores=4 id_base=16
[   10.468286] Found memory entry in CRAT table with proximity_domain=0
[   10.468287] Found memory entry in CRAT table with proximity_domain=0
[   10.468288] Found memory entry in CRAT table with proximity_domain=0
[   10.468289] Found memory entry in CRAT table with proximity_domain=0
[   10.468290] Found cache entry in CRAT table with processor_id=16
[   10.468291] Found cache entry in CRAT table with processor_id=16
[   10.468292] Found cache entry in CRAT table with processor_id=16
[   10.468293] Found cache entry in CRAT table with processor_id=17
[   10.468293] Found cache entry in CRAT table with processor_id=18
[   10.468294] Found cache entry in CRAT table with processor_id=18
[   10.468295] Found cache entry in CRAT table with processor_id=18
[   10.468296] Found cache entry in CRAT table with processor_id=19
[   10.468297] Found TLB entry in CRAT table (not processing)
[   10.468298] Found TLB entry in CRAT table (not processing)
[   10.468299] Found TLB entry in CRAT table (not processing)
[   10.468299] Found TLB entry in CRAT table (not processing)
[   10.468300] Found TLB entry in CRAT table (not processing)
[   10.468301] Found TLB entry in CRAT table (not processing)
[   10.468301] Found TLB entry in CRAT table (not processing)
[   10.468302] Found TLB entry in CRAT table (not processing)
[   10.468303] Found TLB entry in CRAT table (not processing)
[   10.468304] Found TLB entry in CRAT table (not processing)
[   10.468305] Creating topology SYSFS entries
[   10.468331] Finished initializing topology ret=0
[   10.468523] kfd kfd: Initialized module
[   10.468892] checking generic (e0000000 500000) vs hw (e0000000 10000000)
[   10.468896] fb: switching to radeondrmfb from VESA VGA
[   10.468951] Console: switching to colour dummy device 80x25
[   10.469320] [drm] initializing kernel modesetting (ARUBA 0x1002:0x990E 0x1043:0x8526).
[   10.469334] [drm] register mmio base: 0xFEB00000
[   10.469335] [drm] register mmio size: 262144
[   10.469377] ATOM BIOS: 113
[   10.469433] radeon 0000:00:01.0: VRAM: 768M 0x0000000000000000 - 0x000000002FFFFFFF (768M used)
[   10.469435] radeon 0000:00:01.0: GTT: 1024M 0x0000000030000000 - 0x000000006FFFFFFF
[   10.469437] [drm] Detected VRAM RAM=768M, BAR=256M
[   10.469438] [drm] RAM width 64bits DDR
[   10.469517] [TTM] Zone  kernel: Available graphics memory: 7799622 kiB
[   10.469518] [TTM] Zone   dma32: Available graphics memory: 2097152 kiB
[   10.469519] [TTM] Initializing pool allocator
[   10.469523] [TTM] Initializing DMA pool allocator
[   10.469539] [drm] radeon: 768M of VRAM memory ready
[   10.469541] [drm] radeon: 1024M of GTT memory ready.
[   10.469553] [drm] Loading ARUBA Microcode
[   11.055515] [drm] Internal thermal controller without fan control
[   11.055695] [drm] radeon: dpm initialized
[   11.363669] [drm] Found VCE firmware/feedback version 50.0.1 / 17!
[   11.363691] [drm] GART: num cpu pages 262144, num gpu pages 262144
[   11.390611] [drm] PCIE GART of 1024M enabled (table at 0x00000000002E8000).
[   11.390732] radeon 0000:00:01.0: WB enabled
[   11.390735] radeon 0000:00:01.0: fence driver on ring 0 use gpu addr 0x0000000030000c00 and cpu addr 0xffff8800a8709c00
[   11.391465] radeon 0000:00:01.0: fence driver on ring 5 use gpu addr 0x0000000000075a18 and cpu addr 0xffffc90002035a18
[   11.411490] radeon 0000:00:01.0: fence driver on ring 6 use gpu addr 0x0000000030000c18 and cpu addr 0xffff8800a8709c18
[   11.411492] radeon 0000:00:01.0: fence driver on ring 7 use gpu addr 0x0000000030000c1c and cpu addr 0xffff8800a8709c1c
[   11.411494] radeon 0000:00:01.0: fence driver on ring 1 use gpu addr 0x0000000030000c04 and cpu addr 0xffff8800a8709c04
[   11.411495] radeon 0000:00:01.0: fence driver on ring 2 use gpu addr 0x0000000030000c08 and cpu addr 0xffff8800a8709c08
[   11.411497] radeon 0000:00:01.0: fence driver on ring 3 use gpu addr 0x0000000030000c0c and cpu addr 0xffff8800a8709c0c
[   11.411498] radeon 0000:00:01.0: fence driver on ring 4 use gpu addr 0x0000000030000c10 and cpu addr 0xffff8800a8709c10
[   11.411500] [drm] Supports vblank timestamp caching Rev 2 (21.10.2013).
[   11.411501] [drm] Driver supports precise vblank timestamp query.
[   11.411502] radeon 0000:00:01.0: radeon: MSI limited to 32-bit
[   11.411526] radeon 0000:00:01.0: radeon: using MSI.
[   11.411821] [drm] radeon: irq initialized.
[   11.430803] [drm] ring test on 0 succeeded in 3 usecs
[   11.430810] [drm] ring test on 3 succeeded in 3 usecs
[   11.430815] [drm] ring test on 4 succeeded in 3 usecs
[   11.476489] [drm] ring test on 5 succeeded in 1 usecs
[   11.496343] [drm] UVD initialized successfully.
[   11.605538] [drm] ring test on 6 succeeded in 17 usecs
[   11.605550] [drm] ring test on 7 succeeded in 3 usecs
[   11.605551] [drm] VCE initialized successfully.
[   11.606354] [drm] ib test on ring 0 succeeded in 0 usecs
[   11.606884] [drm] ib test on ring 3 succeeded in 0 usecs
[   11.607413] [drm] ib test on ring 4 succeeded in 0 usecs
[   11.627799] [drm] ib test on ring 5 succeeded
[   11.635586] EXT4-fs (sda1): re-mounted. Opts: errors=remount-ro
[   11.814729] init: failsafe main process (669) killed by TERM signal
[   12.145329] [drm] ib test on ring 6 succeeded
[   12.145860] [drm] ib test on ring 7 succeeded
[   12.146652] [drm] Radeon Display Connectors
[   12.146653] [drm] Connector 0:
[   12.146654] [drm]   HDMI-A-1
[   12.146655] [drm]   HPD1
[   12.146656] [drm]   DDC: 0x6530 0x6530 0x6534 0x6534 0x6538 0x6538 0x653c 0x653c
[   12.146657] [drm]   Encoders:
[   12.146658] [drm]     DFP1: INTERNAL_UNIPHY2
[   12.146659] [drm] Connector 1:
[   12.146659] [drm]   VGA-1
[   12.146660] [drm]   HPD2
[   12.146661] [drm]   DDC: 0x6540 0x6540 0x6544 0x6544 0x6548 0x6548 0x654c 0x654c
[   12.146662] [drm]   Encoders:
[   12.146662] [drm]     CRT1: INTERNAL_UNIPHY2
[   12.146663] [drm]     CRT1: NUTMEG
[   12.146664] [drm] Connector 2:
[   12.146664] [drm]   DVI-D-1
[   12.146665] [drm]   HPD3
[   12.146666] [drm]   DDC: 0x6550 0x6550 0x6554 0x6554 0x6558 0x6558 0x655c 0x655c
[   12.146667] [drm]   Encoders:
[   12.146667] [drm]     DFP2: INTERNAL_UNIPHY
[   12.250349] Bluetooth: Core ver 2.21
[   12.250368] NET: Registered protocol family 31
[   12.250370] Bluetooth: HCI device and connection manager initialized
[   12.250374] Bluetooth: HCI socket layer initialized
[   12.250377] Bluetooth: L2CAP socket layer initialized
[   12.250383] Bluetooth: SCO socket layer initialized
[   12.266318] Bluetooth: RFCOMM TTY layer initialized
[   12.266327] Bluetooth: RFCOMM socket layer initialized
[   12.266332] Bluetooth: RFCOMM ver 1.11
[   12.319808] Bluetooth: BNEP (Ethernet Emulation) ver 1.3
[   12.319812] Bluetooth: BNEP filters: protocol multicast
[   12.319817] Bluetooth: BNEP socket layer initialized
[   12.327106] [drm] fb mappable at 0xE04EC000
[   12.327109] [drm] vram apper at 0xE0000000
[   12.327110] [drm] size 5242880
[   12.327111] [drm] fb depth is 24
[   12.327112] [drm]    pitch is 5120
[   12.327245] fbcon: radeondrmfb (fb0) is primary device
[   12.327362] Console: switching to colour frame buffer device 160x64
[   12.327382] radeon 0000:00:01.0: fb0: radeondrmfb frame buffer device
[   12.345071] [drm] Initialized radeon 2.43.0 20080528 for 0000:00:01.0 on minor 0
[   12.380332] init: cups main process (741) killed by HUP signal
[   12.380342] init: cups main process ended, respawning
[   12.662652] usblp 7-1.3:1.0: usblp0: USB Bidirectional printer dev 4 if 0 alt 0 proto 2 vid 0x04F9 pid 0x003A
[   12.662681] usbcore: registered new interface driver usblp
[   13.287481] r8169 0000:04:00.0 eth0: link down
[   13.287486] r8169 0000:04:00.0 eth0: link down
[   13.287546] IPv6: ADDRCONF(NETDEV_UP): eth0: link is not ready
[   17.013791] r8169 0000:04:00.0 eth0: link up
[   17.013802] IPv6: ADDRCONF(NETDEV_CHANGE): eth0: link becomes ready
[   17.481298] [drm:atom_op_jump [radeon]] *ERROR* atombios stuck in loop for more than 5secs aborting
[   17.481314] [drm:atom_execute_table_locked [radeon]] *ERROR* atombios stuck executing E4DE (len 2650, WS 0, PS 8) @ 0xEAE6
[   17.497444] audit_printk_skb: 138 callbacks suppressed
[   17.497447] audit: type=1400 audit(1474476032.045:58): apparmor="STATUS" operation="profile_replace" profile="unconfined" name="/usr/bin/evince-previewer//sanitized_helper" pid=798 comm="apparmor_parser"
[   17.497454] audit: type=1400 audit(1474476032.045:59): apparmor="STATUS" operation="profile_replace" profile="unconfined" name="/usr/bin/evince-thumbnailer" pid=798 comm="apparmor_parser"
[   17.513452] audit: type=1400 audit(1474476032.061:60): apparmor="STATUS" operation="profile_replace" profile="unconfined" name="/usr/bin/evince-thumbnailer//sanitized_helper" pid=798 comm="apparmor_parser"
[   17.514572] audit: type=1400 audit(1474476032.061:61): apparmor="STATUS" operation="profile_replace" profile="unconfined" name="/usr/bin/evince-previewer//sanitized_helper" pid=798 comm="apparmor_parser"
[   17.514576] audit: type=1400 audit(1474476032.061:62): apparmor="STATUS" operation="profile_replace" profile="unconfined" name="/usr/bin/evince-thumbnailer" pid=798 comm="apparmor_parser"
[   17.529508] audit: type=1400 audit(1474476032.077:63): apparmor="STATUS" operation="profile_replace" profile="unconfined" name="/usr/bin/evince-thumbnailer//sanitized_helper" pid=798 comm="apparmor_parser"
[   17.529966] audit: type=1400 audit(1474476032.077:64): apparmor="STATUS" operation="profile_replace" profile="unconfined" name="/usr/bin/evince-thumbnailer" pid=798 comm="apparmor_parser"
[   17.553450] audit: type=1400 audit(1474476032.101:65): apparmor="STATUS" operation="profile_replace" profile="unconfined" name="/usr/bin/evince-thumbnailer//sanitized_helper" pid=798 comm="apparmor_parser"
[   17.553980] audit: type=1400 audit(1474476032.101:66): apparmor="STATUS" operation="profile_replace" profile="unconfined" name="/usr/bin/evince-thumbnailer//sanitized_helper" pid=798 comm="apparmor_parser"
[   17.576501] [drm:radeon_dp_link_train [radeon]] *ERROR* clock recovery reached max voltage
[   17.576521] [drm:radeon_dp_link_train [radeon]] *ERROR* clock recovery failed
[   18.224225] audit: type=1400 audit(1474476032.769:67): apparmor="STATUS" operation="profile_replace" profile="unconfined" name="/usr/sbin/cups-browsed" pid=1059 comm="apparmor_parser"
[   42.111866] audit: type=1400 audit(1474476056.659:68): apparmor="STATUS" operation="profile_replace" profile="unconfined" name="/usr/lib/cups/backend/cups-pdf" pid=2109 comm="apparmor_parser"
[   42.111874] audit: type=1400 audit(1474476056.659:69): apparmor="STATUS" operation="profile_replace" profile="unconfined" name="/usr/sbin/cupsd" pid=2109 comm="apparmor_parser"
[   42.112198] audit: type=1400 audit(1474476056.659:70): apparmor="STATUS" operation="profile_replace" profile="unconfined" name="/usr/sbin/cupsd" pid=2109 comm="apparmor_parser"

Hors ligne

#14 Le 21/09/2016, à 19:42

goliath60

Re : un démarrage qui dure 25 min [résolu]

vadee@vadee:~$ systemd-analyze blame 
systemd-analyze : commande introuvable
vadee@vadee:~$ 

Hors ligne

#15 Le 21/09/2016, à 19:43

goliath60

Re : un démarrage qui dure 25 min [résolu]

vadee@vadee:~$ sudo smartctl  -s on  -a /dev/sda
[sudo] password for vadee: 
sudo: smartctl: command not found
vadee@vadee:~$ 

Hors ligne

#16 Le 21/09/2016, à 19:43

goliath60

Re : un démarrage qui dure 25 min [résolu]

au vu de ces réponses, je suppose que je dois installer l'application GSMARTCONTROL
c'est parti

Hors ligne

#17 Le 21/09/2016, à 19:49

goliath60

Re : un démarrage qui dure 25 min [résolu]

après installation:

[    0.000000] Initializing cgroup subsys cpuset
[    0.000000] Initializing cgroup subsys cpu
[    0.000000] Initializing cgroup subsys cpuacct
[    0.000000] Linux version 4.4.0-36-generic (buildd@lgw01-20) (gcc version 4.8.4 (Ubuntu 4.8.4-2ubuntu1~14.04.3) ) #55~14.04.1-Ubuntu SMP Fri Aug 12 11:49:30 UTC 2016 (Ubuntu 4.4.0-36.55~14.04.1-generic 4.4.16)
[    0.000000] Command line: BOOT_IMAGE=/boot/vmlinuz-4.4.0-36-generic root=UUID=496e9007-632f-49ee-a0ce-7f2a5eb1a540 ro quiet splash vt.handoff=7
[    0.000000] KERNEL supported cpus:
[    0.000000]   Intel GenuineIntel
[    0.000000]   AMD AuthenticAMD
[    0.000000]   Centaur CentaurHauls
[    0.000000] tseg: 00af000000
[    0.000000] x86/fpu: xstate_offset[2]:  576, xstate_sizes[2]:  256
[    0.000000] x86/fpu: Supporting XSAVE feature 0x01: 'x87 floating point registers'
[    0.000000] x86/fpu: Supporting XSAVE feature 0x02: 'SSE registers'
[    0.000000] x86/fpu: Supporting XSAVE feature 0x04: 'AVX registers'
[    0.000000] x86/fpu: Enabled xstate features 0x7, context size is 832 bytes, using 'standard' format.
[    0.000000] x86/fpu: Using 'lazy' FPU context switches.
[    0.000000] e820: BIOS-provided physical RAM map:
[    0.000000] BIOS-e820: [mem 0x0000000000000000-0x000000000009e7ff] usable
[    0.000000] BIOS-e820: [mem 0x000000000009e800-0x000000000009ffff] reserved
[    0.000000] BIOS-e820: [mem 0x00000000000e0000-0x00000000000fffff] reserved
[    0.000000] BIOS-e820: [mem 0x0000000000100000-0x00000000ad183fff] usable
[    0.000000] BIOS-e820: [mem 0x00000000ad184000-0x00000000ad1b3fff] reserved
[    0.000000] BIOS-e820: [mem 0x00000000ad1b4000-0x00000000ad1c3fff] ACPI data
[    0.000000] BIOS-e820: [mem 0x00000000ad1c4000-0x00000000ad845fff] ACPI NVS
[    0.000000] BIOS-e820: [mem 0x00000000ad846000-0x00000000ae3e7fff] reserved
[    0.000000] BIOS-e820: [mem 0x00000000ae3e8000-0x00000000ae3e8fff] usable
[    0.000000] BIOS-e820: [mem 0x00000000ae3e9000-0x00000000ae5eefff] ACPI NVS
[    0.000000] BIOS-e820: [mem 0x00000000ae5ef000-0x00000000aea32fff] usable
[    0.000000] BIOS-e820: [mem 0x00000000aea33000-0x00000000aeff3fff] reserved
[    0.000000] BIOS-e820: [mem 0x00000000aeff4000-0x00000000aeffffff] usable
[    0.000000] BIOS-e820: [mem 0x00000000f8000000-0x00000000fbffffff] reserved
[    0.000000] BIOS-e820: [mem 0x00000000feb80000-0x00000000fec00fff] reserved
[    0.000000] BIOS-e820: [mem 0x00000000fec10000-0x00000000fec10fff] reserved
[    0.000000] BIOS-e820: [mem 0x00000000fed00000-0x00000000fed00fff] reserved
[    0.000000] BIOS-e820: [mem 0x00000000fed40000-0x00000000fed44fff] reserved
[    0.000000] BIOS-e820: [mem 0x00000000fed80000-0x00000000fed8ffff] reserved
[    0.000000] BIOS-e820: [mem 0x00000000ff000000-0x00000000ffffffff] reserved
[    0.000000] BIOS-e820: [mem 0x0000000100000000-0x000000041effffff] usable
[    0.000000] NX (Execute Disable) protection: active
[    0.000000] SMBIOS 2.7 present.
[    0.000000] DMI: System manufacturer System Product Name/A68HM-PLUS, BIOS 0803 04/09/2015
[    0.000000] e820: update [mem 0x00000000-0x00000fff] usable ==> reserved
[    0.000000] e820: remove [mem 0x000a0000-0x000fffff] usable
[    0.000000] e820: last_pfn = 0x41f000 max_arch_pfn = 0x400000000
[    0.000000] MTRR default type: uncachable
[    0.000000] MTRR fixed ranges enabled:
[    0.000000]   00000-9FFFF write-back
[    0.000000]   A0000-BFFFF write-through
[    0.000000]   C0000-D0FFF write-protect
[    0.000000]   D1000-E7FFF uncachable
[    0.000000]   E8000-FFFFF write-protect
[    0.000000] MTRR variable ranges enabled:
[    0.000000]   0 base 000000000000 mask FFFF80000000 write-back
[    0.000000]   1 base 000080000000 mask FFFFE0000000 write-back
[    0.000000]   2 base 0000A0000000 mask FFFFF0000000 write-back
[    0.000000]   3 disabled
[    0.000000]   4 disabled
[    0.000000]   5 disabled
[    0.000000]   6 disabled
[    0.000000]   7 disabled
[    0.000000] TOM2: 000000041f000000 aka 16880M
[    0.000000] x86/PAT: Configuration [0-7]: WB  WC  UC- UC  WB  WC  UC- WT  
[    0.000000] e820: update [mem 0xb0000000-0xffffffff] usable ==> reserved
[    0.000000] e820: last_pfn = 0xaf000 max_arch_pfn = 0x400000000
[    0.000000] found SMP MP-table at [mem 0x000fd840-0x000fd84f] mapped at [ffff8800000fd840]
[    0.000000] Scanning 1 areas for low memory corruption
[    0.000000] Base memory trampoline at [ffff880000098000] 98000 size 24576
[    0.000000] Using GB pages for direct mapping
[    0.000000] BRK [0x02004000, 0x02004fff] PGTABLE
[    0.000000] BRK [0x02005000, 0x02005fff] PGTABLE
[    0.000000] BRK [0x02006000, 0x02006fff] PGTABLE
[    0.000000] BRK [0x02007000, 0x02007fff] PGTABLE
[    0.000000] BRK [0x02008000, 0x02008fff] PGTABLE
[    0.000000] BRK [0x02009000, 0x02009fff] PGTABLE
[    0.000000] RAMDISK: [mem 0x357b4000-0x36bd1fff]
[    0.000000] ACPI: Early table checksum verification disabled
[    0.000000] ACPI: RSDP 0x00000000000F0490 000024 (v02 ALASKA)
[    0.000000] ACPI: XSDT 0x00000000AD1BB078 00006C (v01 ALASKA A M I    01072009 AMI  00010013)
[    0.000000] ACPI: FACP 0x00000000AD1C2280 00010C (v05 ALASKA A M I    01072009 AMI  00010013)
[    0.000000] ACPI BIOS Warning (bug): Optional FADT field Pm2ControlBlock has zero address or length: 0x0000000000000000/0x1 (20150930/tbfadt-654)
[    0.000000] ACPI: DSDT 0x00000000AD1BB180 007100 (v02 ALASKA A M I    00000000 INTL 20051117)
[    0.000000] ACPI: FACS 0x00000000AD83B080 000040
[    0.000000] ACPI: APIC 0x00000000AD1C2390 000072 (v03 ALASKA A M I    01072009 AMI  00010013)
[    0.000000] ACPI: FPDT 0x00000000AD1C2408 000044 (v01 ALASKA A M I    01072009 AMI  00010013)
[    0.000000] ACPI: MCFG 0x00000000AD1C2450 00003C (v01 ALASKA A M I    01072009 MSFT 00010013)
[    0.000000] ACPI: HPET 0x00000000AD1C2490 000038 (v01 ALASKA A M I    01072009 AMI  00000005)
[    0.000000] ACPI: CRAT 0x00000000AD1C35C0 000578 (v01 AMD    BANTRY   00000001 AMD  00000001)
[    0.000000] ACPI: UEFI 0x00000000AD1C2520 000042 (v01 ALASKA A M I    01072009      00000000)
[    0.000000] ACPI: SSDT 0x00000000AD1C2568 000B9C (v01 AMD    BANTRY   00000001 AMD  00000001)
[    0.000000] ACPI: SSDT 0x00000000AD1C3108 0004B7 (v02 AMD    BANTRY   00000001 MSFT 04000000)
[    0.000000] ACPI: Local APIC address 0xfee00000
[    0.000000] No NUMA configuration found
[    0.000000] Faking a node at [mem 0x0000000000000000-0x000000041effffff]
[    0.000000] NODE_DATA(0) allocated [mem 0x41eff7000-0x41effbfff]
[    0.000000] Zone ranges:
[    0.000000]   DMA      [mem 0x0000000000001000-0x0000000000ffffff]
[    0.000000]   DMA32    [mem 0x0000000001000000-0x00000000ffffffff]
[    0.000000]   Normal   [mem 0x0000000100000000-0x000000041effffff]
[    0.000000]   Device   empty
[    0.000000] Movable zone start for each node
[    0.000000] Early memory node ranges
[    0.000000]   node   0: [mem 0x0000000000001000-0x000000000009dfff]
[    0.000000]   node   0: [mem 0x0000000000100000-0x00000000ad183fff]
[    0.000000]   node   0: [mem 0x00000000ae3e8000-0x00000000ae3e8fff]
[    0.000000]   node   0: [mem 0x00000000ae5ef000-0x00000000aea32fff]
[    0.000000]   node   0: [mem 0x00000000aeff4000-0x00000000aeffffff]
[    0.000000]   node   0: [mem 0x0000000100000000-0x000000041effffff]
[    0.000000] Initmem setup node 0 [mem 0x0000000000001000-0x000000041effffff]
[    0.000000] On node 0 totalpages: 3982706
[    0.000000]   DMA zone: 64 pages used for memmap
[    0.000000]   DMA zone: 21 pages reserved
[    0.000000]   DMA zone: 3997 pages, LIFO batch:0
[    0.000000]   DMA32 zone: 11032 pages used for memmap
[    0.000000]   DMA32 zone: 706005 pages, LIFO batch:31
[    0.000000]   Normal zone: 51136 pages used for memmap
[    0.000000]   Normal zone: 3272704 pages, LIFO batch:31
[    0.000000] ACPI: PM-Timer IO Port: 0x808
[    0.000000] ACPI: Local APIC address 0xfee00000
[    0.000000] ACPI: LAPIC_NMI (acpi_id[0xff] high edge lint[0x1])
[    0.000000] IOAPIC[0]: apic_id 0, version 33, address 0xfec00000, GSI 0-23
[    0.000000] ACPI: INT_SRC_OVR (bus 0 bus_irq 0 global_irq 2 dfl dfl)
[    0.000000] ACPI: INT_SRC_OVR (bus 0 bus_irq 9 global_irq 9 low level)
[    0.000000] ACPI: IRQ0 used by override.
[    0.000000] ACPI: IRQ9 used by override.
[    0.000000] Using ACPI (MADT) for SMP configuration information
[    0.000000] ACPI: HPET id: 0x10228210 base: 0xfed00000
[    0.000000] smpboot: Allowing 4 CPUs, 0 hotplug CPUs
[    0.000000] PM: Registered nosave memory: [mem 0x00000000-0x00000fff]
[    0.000000] PM: Registered nosave memory: [mem 0x0009e000-0x0009efff]
[    0.000000] PM: Registered nosave memory: [mem 0x0009f000-0x0009ffff]
[    0.000000] PM: Registered nosave memory: [mem 0x000a0000-0x000dffff]
[    0.000000] PM: Registered nosave memory: [mem 0x000e0000-0x000fffff]
[    0.000000] PM: Registered nosave memory: [mem 0xad184000-0xad1b3fff]
[    0.000000] PM: Registered nosave memory: [mem 0xad1b4000-0xad1c3fff]
[    0.000000] PM: Registered nosave memory: [mem 0xad1c4000-0xad845fff]
[    0.000000] PM: Registered nosave memory: [mem 0xad846000-0xae3e7fff]
[    0.000000] PM: Registered nosave memory: [mem 0xae3e9000-0xae5eefff]
[    0.000000] PM: Registered nosave memory: [mem 0xaea33000-0xaeff3fff]
[    0.000000] PM: Registered nosave memory: [mem 0xaf000000-0xf7ffffff]
[    0.000000] PM: Registered nosave memory: [mem 0xf8000000-0xfbffffff]
[    0.000000] PM: Registered nosave memory: [mem 0xfc000000-0xfeb7ffff]
[    0.000000] PM: Registered nosave memory: [mem 0xfeb80000-0xfec00fff]
[    0.000000] PM: Registered nosave memory: [mem 0xfec01000-0xfec0ffff]
[    0.000000] PM: Registered nosave memory: [mem 0xfec10000-0xfec10fff]
[    0.000000] PM: Registered nosave memory: [mem 0xfec11000-0xfecfffff]
[    0.000000] PM: Registered nosave memory: [mem 0xfed00000-0xfed00fff]
[    0.000000] PM: Registered nosave memory: [mem 0xfed01000-0xfed3ffff]
[    0.000000] PM: Registered nosave memory: [mem 0xfed40000-0xfed44fff]
[    0.000000] PM: Registered nosave memory: [mem 0xfed45000-0xfed7ffff]
[    0.000000] PM: Registered nosave memory: [mem 0xfed80000-0xfed8ffff]
[    0.000000] PM: Registered nosave memory: [mem 0xfed90000-0xfeffffff]
[    0.000000] PM: Registered nosave memory: [mem 0xff000000-0xffffffff]
[    0.000000] e820: [mem 0xaf000000-0xf7ffffff] available for PCI devices
[    0.000000] Booting paravirtualized kernel on bare hardware
[    0.000000] clocksource: refined-jiffies: mask: 0xffffffff max_cycles: 0xffffffff, max_idle_ns: 7645519600211568 ns
[    0.000000] setup_percpu: NR_CPUS:256 nr_cpumask_bits:256 nr_cpu_ids:4 nr_node_ids:1
[    0.000000] PERCPU: Embedded 33 pages/cpu @ffff88041ec00000 s98008 r8192 d28968 u524288
[    0.000000] pcpu-alloc: s98008 r8192 d28968 u524288 alloc=1*2097152
[    0.000000] pcpu-alloc: [0] 0 1 2 3 
[    0.000000] Built 1 zonelists in Node order, mobility grouping on.  Total pages: 3920453
[    0.000000] Policy zone: Normal
[    0.000000] Kernel command line: BOOT_IMAGE=/boot/vmlinuz-4.4.0-36-generic root=UUID=496e9007-632f-49ee-a0ce-7f2a5eb1a540 ro quiet splash vt.handoff=7
[    0.000000] PID hash table entries: 4096 (order: 3, 32768 bytes)
[    0.000000] Calgary: detecting Calgary via BIOS EBDA area
[    0.000000] Calgary: Unable to locate Rio Grande table in EBDA - bailing!
[    0.000000] Memory: 15576992K/15930824K available (8176K kernel code, 1292K rwdata, 3960K rodata, 1484K init, 1292K bss, 353832K reserved, 0K cma-reserved)
[    0.000000] SLUB: HWalign=64, Order=0-3, MinObjects=0, CPUs=4, Nodes=1
[    0.000000] Hierarchical RCU implementation.
[    0.000000] 	Build-time adjustment of leaf fanout to 64.
[    0.000000] 	RCU restricting CPUs from NR_CPUS=256 to nr_cpu_ids=4.
[    0.000000] RCU: Adjusting geometry for rcu_fanout_leaf=64, nr_cpu_ids=4
[    0.000000] NR_IRQS:16640 nr_irqs:456 16
[    0.000000] spurious 8259A interrupt: IRQ7.
[    0.000000] vt handoff: transparent VT on vt#7
[    0.000000] Console: colour dummy device 80x25
[    0.000000] console [tty0] enabled
[    0.000000] clocksource: hpet: mask: 0xffffffff max_cycles: 0xffffffff, max_idle_ns: 133484873504 ns
[    0.000000] hpet clockevent registered
[    0.000000] tsc: Fast TSC calibration using PIT
[    0.000000] tsc: Detected 3889.875 MHz processor
[    0.000020] Calibrating delay loop (skipped), value calculated using timer frequency.. 7779.75 BogoMIPS (lpj=15559500)
[    0.000023] pid_max: default: 32768 minimum: 301
[    0.000028] ACPI: Core revision 20150930
[    0.003026] ACPI: 3 ACPI AML tables successfully acquired and loaded
[    0.003043] Security Framework initialized
[    0.003045] Yama: becoming mindful.
[    0.003058] AppArmor: AppArmor initialized
[    0.003932] Dentry cache hash table entries: 2097152 (order: 12, 16777216 bytes)
[    0.007689] Inode-cache hash table entries: 1048576 (order: 11, 8388608 bytes)
[    0.009346] Mount-cache hash table entries: 32768 (order: 6, 262144 bytes)
[    0.009362] Mountpoint-cache hash table entries: 32768 (order: 6, 262144 bytes)
[    0.009619] Initializing cgroup subsys io
[    0.009623] Initializing cgroup subsys memory
[    0.009629] Initializing cgroup subsys devices
[    0.009632] Initializing cgroup subsys freezer
[    0.009634] Initializing cgroup subsys net_cls
[    0.009636] Initializing cgroup subsys perf_event
[    0.009638] Initializing cgroup subsys net_prio
[    0.009640] Initializing cgroup subsys hugetlb
[    0.009643] Initializing cgroup subsys pids
[    0.009661] CPU: Physical Processor ID: 0
[    0.009662] CPU: Processor Core ID: 0
[    0.009663] mce: CPU supports 7 MCE banks
[    0.009673] Last level iTLB entries: 4KB 512, 2MB 1024, 4MB 512
[    0.009674] Last level dTLB entries: 4KB 1024, 2MB 1024, 4MB 512, 1GB 0
[    0.009911] Freeing SMP alternatives memory: 28K (ffffffff81eb8000 - ffffffff81ebf000)
[    0.020496] ftrace: allocating 32030 entries in 126 pages
[    0.029273] smpboot: Max logical packages: 2
[    0.029276] smpboot: APIC(10) Converting physical 1 to logical package 0
[    0.029584] ..TIMER: vector=0x30 apic1=0 pin1=2 apic2=-1 pin2=-1
[    0.173761] smpboot: CPU0: AMD A8-6600K APU with Radeon(tm) HD Graphics (family: 0x15, model: 0x13, stepping: 0x1)
[    0.173776] Performance Events: Fam15h core perfctr, AMD PMU driver.
[    0.173780] ... version:                0
[    0.173781] ... bit width:              48
[    0.173781] ... generic registers:      6
[    0.173782] ... value mask:             0000ffffffffffff
[    0.173783] ... max period:             00007fffffffffff
[    0.173784] ... fixed-purpose events:   0
[    0.173784] ... event mask:             000000000000003f
[    0.174417] NMI watchdog: enabled on all CPUs, permanently consumes one hw-PMU counter.
[    0.174476] x86: Booting SMP configuration:
[    0.174477] .... node  #0, CPUs:      #1 #2 #3
[    0.181080] x86: Booted up 1 node, 4 CPUs
[    0.181083] smpboot: Total of 4 processors activated (31119.00 BogoMIPS)
[    0.181713] devtmpfs: initialized
[    0.184875] evm: security.selinux
[    0.184876] evm: security.SMACK64
[    0.184876] evm: security.SMACK64EXEC
[    0.184877] evm: security.SMACK64TRANSMUTE
[    0.184878] evm: security.SMACK64MMAP
[    0.184879] evm: security.ima
[    0.184880] evm: security.capability
[    0.185010] PM: Registering ACPI NVS region [mem 0xad1c4000-0xad845fff] (6823936 bytes)
[    0.185090] PM: Registering ACPI NVS region [mem 0xae3e9000-0xae5eefff] (2121728 bytes)
[    0.185179] clocksource: jiffies: mask: 0xffffffff max_cycles: 0xffffffff, max_idle_ns: 7645041785100000 ns
[    0.185252] pinctrl core: initialized pinctrl subsystem
[    0.185344] RTC time: 16:40:14, date: 09/21/16
[    0.185437] NET: Registered protocol family 16
[    0.193831] cpuidle: using governor ladder
[    0.205830] cpuidle: using governor menu
[    0.205842] PCCT header not found.
[    0.205993] ACPI: bus type PCI registered
[    0.205995] acpiphp: ACPI Hot Plug PCI Controller Driver version: 0.5
[    0.206070] PCI: MMCONFIG for domain 0000 [bus 00-3f] at [mem 0xf8000000-0xfbffffff] (base 0xf8000000)
[    0.206072] PCI: MMCONFIG at [mem 0xf8000000-0xfbffffff] reserved in E820
[    0.206081] PCI: Using configuration type 1 for base access
[    0.206445] mtrr: your CPUs had inconsistent variable MTRR settings
[    0.206446] mtrr: probably your BIOS does not setup all CPUs.
[    0.206447] mtrr: corrected configuration.
[    0.218325] ACPI: Added _OSI(Module Device)
[    0.218327] ACPI: Added _OSI(Processor Device)
[    0.218328] ACPI: Added _OSI(3.0 _SCP Extensions)
[    0.218329] ACPI: Added _OSI(Processor Aggregator Device)
[    0.219608] ACPI: Executed 1 blocks of module-level executable AML code
[    0.221376] [Firmware Bug]: ACPI: BIOS _OSI(Linux) query ignored
[    0.221680] ACPI: Interpreter enabled
[    0.221684] ACPI Exception: AE_NOT_FOUND, While evaluating Sleep State [\_S1_] (20150930/hwxface-580)
[    0.221688] ACPI Exception: AE_NOT_FOUND, While evaluating Sleep State [\_S2_] (20150930/hwxface-580)
[    0.221699] ACPI: (supports S0 S3 S4 S5)
[    0.221700] ACPI: Using IOAPIC for interrupt routing
[    0.221830] PCI: Using host bridge windows from ACPI; if necessary, use "pci=nocrs" and report a bug
[    0.222231] [Firmware Bug]: ACPI: No _BQC method, cannot determine initial brightness
[    0.226244] ACPI: PCI Root Bridge [PCI0] (domain 0000 [bus 00-ff])
[    0.226249] acpi PNP0A08:00: _OSC: OS supports [ExtendedConfig ASPM ClockPM Segments MSI]
[    0.226372] acpi PNP0A08:00: _OSC: platform does not support [PCIeHotplug PME]
[    0.226486] acpi PNP0A08:00: _OSC: OS now controls [AER PCIeCapability]
[    0.226495] acpi PNP0A08:00: [Firmware Info]: MMCONFIG for domain 0000 [bus 00-3f] only partially covers this bridge
[    0.226724] PCI host bridge to bus 0000:00
[    0.226726] pci_bus 0000:00: root bus resource [io  0x0000-0x03af window]
[    0.226728] pci_bus 0000:00: root bus resource [io  0x03e0-0x0cf7 window]
[    0.226729] pci_bus 0000:00: root bus resource [io  0x03b0-0x03df window]
[    0.226730] pci_bus 0000:00: root bus resource [io  0x0d00-0xffff window]
[    0.226732] pci_bus 0000:00: root bus resource [mem 0x000a0000-0x000bffff window]
[    0.226733] pci_bus 0000:00: root bus resource [mem 0x000c0000-0x000dffff window]
[    0.226735] pci_bus 0000:00: root bus resource [mem 0xe0000000-0xffffffff window]
[    0.226736] pci_bus 0000:00: root bus resource [bus 00-ff]
[    0.226742] pci 0000:00:00.0: [1022:1410] type 00 class 0x060000
[    0.226836] pci 0000:00:01.0: [1002:990e] type 00 class 0x030000
[    0.226852] pci 0000:00:01.0: reg 0x10: [mem 0xe0000000-0xefffffff pref]
[    0.226858] pci 0000:00:01.0: reg 0x14: [io  0xf000-0xf0ff]
[    0.226864] pci 0000:00:01.0: reg 0x18: [mem 0xfeb00000-0xfeb3ffff]
[    0.226898] pci 0000:00:01.0: supports D1 D2
[    0.226970] pci 0000:00:01.1: [1002:9902] type 00 class 0x040300
[    0.226986] pci 0000:00:01.1: reg 0x10: [mem 0xfeb44000-0xfeb47fff]
[    0.227027] pci 0000:00:01.1: supports D1 D2
[    0.227115] pci 0000:00:10.0: [1022:7814] type 00 class 0x0c0330
[    0.227154] pci 0000:00:10.0: reg 0x10: [mem 0xfeb48000-0xfeb49fff 64bit]
[    0.227236] pci 0000:00:10.0: PME# supported from D0 D3hot D3cold
[    0.227281] pci 0000:00:10.0: System wakeup disabled by ACPI
[    0.227325] pci 0000:00:11.0: [1022:7801] type 00 class 0x010601
[    0.227350] pci 0000:00:11.0: reg 0x10: [io  0xf140-0xf147]
[    0.227359] pci 0000:00:11.0: reg 0x14: [io  0xf130-0xf133]
[    0.227368] pci 0000:00:11.0: reg 0x18: [io  0xf120-0xf127]
[    0.227376] pci 0000:00:11.0: reg 0x1c: [io  0xf110-0xf113]
[    0.227385] pci 0000:00:11.0: reg 0x20: [io  0xf100-0xf10f]
[    0.227393] pci 0000:00:11.0: reg 0x24: [mem 0xfeb4e000-0xfeb4e7ff]
[    0.227484] pci 0000:00:12.0: [1022:7807] type 00 class 0x0c0310
[    0.227498] pci 0000:00:12.0: reg 0x10: [mem 0xfeb4d000-0xfeb4dfff]
[    0.227581] pci 0000:00:12.0: System wakeup disabled by ACPI
[    0.227619] pci 0000:00:12.2: [1022:7808] type 00 class 0x0c0320
[    0.227643] pci 0000:00:12.2: reg 0x10: [mem 0xfeb4c000-0xfeb4c0ff]
[    0.227709] pci 0000:00:12.2: supports D1 D2
[    0.227710] pci 0000:00:12.2: PME# supported from D0 D1 D2 D3hot D3cold
[    0.227746] pci 0000:00:12.2: System wakeup disabled by ACPI
[    0.227784] pci 0000:00:13.0: [1022:7807] type 00 class 0x0c0310
[    0.227799] pci 0000:00:13.0: reg 0x10: [mem 0xfeb4b000-0xfeb4bfff]
[    0.227881] pci 0000:00:13.0: System wakeup disabled by ACPI
[    0.227927] pci 0000:00:13.2: [1022:7808] type 00 class 0x0c0320
[    0.227951] pci 0000:00:13.2: reg 0x10: [mem 0xfeb4a000-0xfeb4a0ff]
[    0.228017] pci 0000:00:13.2: supports D1 D2
[    0.228018] pci 0000:00:13.2: PME# supported from D0 D1 D2 D3hot D3cold
[    0.228053] pci 0000:00:13.2: System wakeup disabled by ACPI
[    0.228091] pci 0000:00:14.0: [1022:780b] type 00 class 0x0c0500
[    0.228207] pci 0000:00:14.2: [1022:780d] type 00 class 0x040300
[    0.228231] pci 0000:00:14.2: reg 0x10: [mem 0xfeb40000-0xfeb43fff 64bit]
[    0.228287] pci 0000:00:14.2: PME# supported from D0 D3hot D3cold
[    0.228321] pci 0000:00:14.2: System wakeup disabled by ACPI
[    0.228354] pci 0000:00:14.3: [1022:780e] type 00 class 0x060100
[    0.228474] pci 0000:00:14.4: [1022:780f] type 01 class 0x060401
[    0.228536] pci 0000:00:14.4: System wakeup disabled by ACPI
[    0.228577] pci 0000:00:15.0: [1022:43a0] type 01 class 0x060400
[    0.228643] pci 0000:00:15.0: supports D1 D2
[    0.228645] pci 0000:00:15.0: PME# supported from D0 D3hot D3cold
[    0.228683] pci 0000:00:15.0: System wakeup disabled by ACPI
[    0.228722] pci 0000:00:15.1: [1022:43a1] type 01 class 0x060400
[    0.228788] pci 0000:00:15.1: supports D1 D2
[    0.228789] pci 0000:00:15.1: PME# supported from D0 D3hot D3cold
[    0.228828] pci 0000:00:15.1: System wakeup disabled by ACPI
[    0.228865] pci 0000:00:15.2: [1022:43a2] type 01 class 0x060400
[    0.228931] pci 0000:00:15.2: supports D1 D2
[    0.228932] pci 0000:00:15.2: PME# supported from D0 D3hot D3cold
[    0.228970] pci 0000:00:15.2: System wakeup disabled by ACPI
[    0.229008] pci 0000:00:18.0: [1022:1400] type 00 class 0x060000
[    0.229077] pci 0000:00:18.1: [1022:1401] type 00 class 0x060000
[    0.229155] pci 0000:00:18.2: [1022:1402] type 00 class 0x060000
[    0.229222] pci 0000:00:18.3: [1022:1403] type 00 class 0x060000
[    0.229294] pci 0000:00:18.4: [1022:1404] type 00 class 0x060000
[    0.229361] pci 0000:00:18.5: [1022:1405] type 00 class 0x060000
[    0.229501] pci 0000:00:14.4: PCI bridge to [bus 01] (subtractive decode)
[    0.229509] pci 0000:00:14.4:   bridge window [io  0x0000-0x03af window] (subtractive decode)
[    0.229511] pci 0000:00:14.4:   bridge window [io  0x03e0-0x0cf7 window] (subtractive decode)
[    0.229512] pci 0000:00:14.4:   bridge window [io  0x03b0-0x03df window] (subtractive decode)
[    0.229513] pci 0000:00:14.4:   bridge window [io  0x0d00-0xffff window] (subtractive decode)
[    0.229515] pci 0000:00:14.4:   bridge window [mem 0x000a0000-0x000bffff window] (subtractive decode)
[    0.229516] pci 0000:00:14.4:   bridge window [mem 0x000c0000-0x000dffff window] (subtractive decode)
[    0.229517] pci 0000:00:14.4:   bridge window [mem 0xe0000000-0xffffffff window] (subtractive decode)
[    0.229561] pci 0000:00:15.0: PCI bridge to [bus 02]
[    0.229631] pci 0000:03:00.0: [1b21:1142] type 00 class 0x0c0330
[    0.229677] pci 0000:03:00.0: reg 0x10: [mem 0xfea00000-0xfea07fff 64bit]
[    0.229794] pci 0000:03:00.0: PME# supported from D3cold
[    0.237850] pci 0000:00:15.1: PCI bridge to [bus 03]
[    0.237873] pci 0000:00:15.1:   bridge window [mem 0xfea00000-0xfeafffff]
[    0.237974] pci 0000:04:00.0: [10ec:8168] type 00 class 0x020000
[    0.238017] pci 0000:04:00.0: reg 0x10: [io  0xe000-0xe0ff]
[    0.238046] pci 0000:04:00.0: reg 0x18: [mem 0xfe900000-0xfe900fff 64bit]
[    0.238064] pci 0000:04:00.0: reg 0x20: [mem 0xf0000000-0xf0003fff 64bit pref]
[    0.238142] pci 0000:04:00.0: supports D1 D2
[    0.238144] pci 0000:04:00.0: PME# supported from D0 D1 D2 D3hot D3cold
[    0.238188] pci 0000:04:00.0: System wakeup disabled by ACPI
[    0.245852] pci 0000:00:15.2: PCI bridge to [bus 04]
[    0.245864] pci 0000:00:15.2:   bridge window [io  0xe000-0xefff]
[    0.245876] pci 0000:00:15.2:   bridge window [mem 0xfe900000-0xfe9fffff]
[    0.245881] pci 0000:00:15.2:   bridge window [mem 0xf0000000-0xf00fffff 64bit pref]
[    0.246336] ACPI: PCI Interrupt Link [LNKA] (IRQs 4 5 6 7 10 11 14 15) *0
[    0.246387] ACPI: PCI Interrupt Link [LNKB] (IRQs 4 5 6 7 10 11 14 15) *0
[    0.246438] ACPI: PCI Interrupt Link [LNKC] (IRQs 4 5 6 7 10 11 14 15) *0
[    0.246489] ACPI: PCI Interrupt Link [LNKD] (IRQs 4 10 11 14 15) *0
[    0.246530] ACPI: PCI Interrupt Link [LNKE] (IRQs 4 5 6 7 10 11 14 15) *0
[    0.246562] ACPI: PCI Interrupt Link [LNKF] (IRQs 4 5 6 7 10 11 14 15) *0
[    0.246594] ACPI: PCI Interrupt Link [LNKG] (IRQs 4 5 6 7 10 11 14 15) *0
[    0.246626] ACPI: PCI Interrupt Link [LNKH] (IRQs 4 5 6 7 10 11 14 15) *0
[    0.246856] vgaarb: setting as boot device: PCI:0000:00:01.0
[    0.246858] vgaarb: device added: PCI:0000:00:01.0,decodes=io+mem,owns=io+mem,locks=none
[    0.246862] vgaarb: loaded
[    0.246863] vgaarb: bridge control possible 0000:00:01.0
[    0.247091] SCSI subsystem initialized
[    0.247144] libata version 3.00 loaded.
[    0.247164] ACPI: bus type USB registered
[    0.247179] usbcore: registered new interface driver usbfs
[    0.247187] usbcore: registered new interface driver hub
[    0.247202] usbcore: registered new device driver usb
[    0.247327] PCI: Using ACPI for IRQ routing
[    0.249245] PCI: pci_cache_line_size set to 64 bytes
[    0.249299] e820: reserve RAM buffer [mem 0x0009e800-0x0009ffff]
[    0.249301] e820: reserve RAM buffer [mem 0xad184000-0xafffffff]
[    0.249302] e820: reserve RAM buffer [mem 0xae3e9000-0xafffffff]
[    0.249303] e820: reserve RAM buffer [mem 0xaea33000-0xafffffff]
[    0.249304] e820: reserve RAM buffer [mem 0xaf000000-0xafffffff]
[    0.249305] e820: reserve RAM buffer [mem 0x41f000000-0x41fffffff]
[    0.249405] NetLabel: Initializing
[    0.249407] NetLabel:  domain hash size = 128
[    0.249407] NetLabel:  protocols = UNLABELED CIPSOv4
[    0.249416] NetLabel:  unlabeled traffic allowed by default
[    0.249476] hpet0: at MMIO 0xfed00000, IRQs 2, 8, 0
[    0.249479] hpet0: 3 comparators, 32-bit 14.318180 MHz counter
[    0.251563] clocksource: Switched to clocksource hpet
[    0.257468] AppArmor: AppArmor Filesystem Enabled
[    0.257533] pnp: PnP ACPI init
[    0.257649] system 00:00: [mem 0xf8000000-0xfbffffff] has been reserved
[    0.257653] system 00:00: Plug and Play ACPI device, IDs PNP0c01 (active)
[    0.257719] system 00:01: [mem 0xb0000000-0xdfffffff] has been reserved
[    0.257721] system 00:01: Plug and Play ACPI device, IDs PNP0c02 (active)
[    0.257785] system 00:02: Plug and Play ACPI device, IDs PNP0c02 (active)
[    0.257923] system 00:03: [io  0x0300-0x031f] has been reserved
[    0.257924] system 00:03: [io  0x0290-0x029f] has been reserved
[    0.257926] system 00:03: [io  0x0230-0x023f] has been reserved
[    0.257928] system 00:03: Plug and Play ACPI device, IDs PNP0c02 (active)
[    0.257977] pnp 00:04: Plug and Play ACPI device, IDs PNP0b00 (active)
[    0.258073] system 00:05: [io  0x04d0-0x04d1] has been reserved
[    0.258076] system 00:05: Plug and Play ACPI device, IDs PNP0c02 (active)
[    0.258110] system 00:06: Plug and Play ACPI device, IDs PNP0c02 (active)
[    0.258307] pnp 00:07: [dma 0 disabled]
[    0.258345] pnp 00:07: Plug and Play ACPI device, IDs PNP0501 (active)
[    0.258548] system 00:08: [io  0x04d0-0x04d1] has been reserved
[    0.258549] system 00:08: [io  0x040b] has been reserved
[    0.258552] system 00:08: [io  0x04d6] has been reserved
[    0.258554] system 00:08: [io  0x0c00-0x0c01] has been reserved
[    0.258555] system 00:08: [io  0x0c14] has been reserved
[    0.258557] system 00:08: [io  0x0c50-0x0c51] has been reserved
[    0.258558] system 00:08: [io  0x0c52] has been reserved
[    0.258559] system 00:08: [io  0x0c6c] has been reserved
[    0.258561] system 00:08: [io  0x0c6f] has been reserved
[    0.258562] system 00:08: [io  0x0cd0-0x0cd1] has been reserved
[    0.258563] system 00:08: [io  0x0cd2-0x0cd3] has been reserved
[    0.258565] system 00:08: [io  0x0cd4-0x0cd5] has been reserved
[    0.258566] system 00:08: [io  0x0cd6-0x0cd7] has been reserved
[    0.258567] system 00:08: [io  0x0cd8-0x0cdf] has been reserved
[    0.258569] system 00:08: [io  0x0800-0x089f] could not be reserved
[    0.258570] system 00:08: [io  0x0b20-0x0b3f] has been reserved
[    0.258572] system 00:08: [io  0x0900-0x090f] has been reserved
[    0.258573] system 00:08: [io  0x0910-0x091f] has been reserved
[    0.258575] system 00:08: [io  0xfe00-0xfefe] has been reserved
[    0.258577] system 00:08: [mem 0xfec00000-0xfec00fff] could not be reserved
[    0.258578] system 00:08: [mem 0xfee00000-0xfee00fff] has been reserved
[    0.258580] system 00:08: [mem 0xfed80000-0xfed8ffff] has been reserved
[    0.258581] system 00:08: [mem 0xfed61000-0xfed70fff] has been reserved
[    0.258583] system 00:08: [mem 0xfec10000-0xfec10fff] has been reserved
[    0.258585] system 00:08: [mem 0xff000000-0xffffffff] has been reserved
[    0.258586] system 00:08: Plug and Play ACPI device, IDs PNP0c02 (active)
[    0.258727] pnp: PnP ACPI: found 9 devices
[    0.265009] clocksource: acpi_pm: mask: 0xffffff max_cycles: 0xffffff, max_idle_ns: 2085701024 ns
[    0.265038] pci 0000:00:14.4: PCI bridge to [bus 01]
[    0.265050] pci 0000:00:15.0: PCI bridge to [bus 02]
[    0.265059] pci 0000:00:15.1: PCI bridge to [bus 03]
[    0.265063] pci 0000:00:15.1:   bridge window [mem 0xfea00000-0xfeafffff]
[    0.265070] pci 0000:00:15.2: PCI bridge to [bus 04]
[    0.265072] pci 0000:00:15.2:   bridge window [io  0xe000-0xefff]
[    0.265076] pci 0000:00:15.2:   bridge window [mem 0xfe900000-0xfe9fffff]
[    0.265079] pci 0000:00:15.2:   bridge window [mem 0xf0000000-0xf00fffff 64bit pref]
[    0.265085] pci_bus 0000:00: resource 4 [io  0x0000-0x03af window]
[    0.265086] pci_bus 0000:00: resource 5 [io  0x03e0-0x0cf7 window]
[    0.265088] pci_bus 0000:00: resource 6 [io  0x03b0-0x03df window]
[    0.265089] pci_bus 0000:00: resource 7 [io  0x0d00-0xffff window]
[    0.265090] pci_bus 0000:00: resource 8 [mem 0x000a0000-0x000bffff window]
[    0.265092] pci_bus 0000:00: resource 9 [mem 0x000c0000-0x000dffff window]
[    0.265093] pci_bus 0000:00: resource 10 [mem 0xe0000000-0xffffffff window]
[    0.265095] pci_bus 0000:01: resource 4 [io  0x0000-0x03af window]
[    0.265096] pci_bus 0000:01: resource 5 [io  0x03e0-0x0cf7 window]
[    0.265097] pci_bus 0000:01: resource 6 [io  0x03b0-0x03df window]
[    0.265098] pci_bus 0000:01: resource 7 [io  0x0d00-0xffff window]
[    0.265100] pci_bus 0000:01: resource 8 [mem 0x000a0000-0x000bffff window]
[    0.265101] pci_bus 0000:01: resource 9 [mem 0x000c0000-0x000dffff window]
[    0.265102] pci_bus 0000:01: resource 10 [mem 0xe0000000-0xffffffff window]
[    0.265104] pci_bus 0000:03: resource 1 [mem 0xfea00000-0xfeafffff]
[    0.265105] pci_bus 0000:04: resource 0 [io  0xe000-0xefff]
[    0.265106] pci_bus 0000:04: resource 1 [mem 0xfe900000-0xfe9fffff]
[    0.265107] pci_bus 0000:04: resource 2 [mem 0xf0000000-0xf00fffff 64bit pref]
[    0.265133] NET: Registered protocol family 2
[    0.265318] TCP established hash table entries: 131072 (order: 8, 1048576 bytes)
[    0.265569] TCP bind hash table entries: 65536 (order: 8, 1048576 bytes)
[    0.265768] TCP: Hash tables configured (established 131072 bind 65536)
[    0.265811] UDP hash table entries: 8192 (order: 6, 262144 bytes)
[    0.265873] UDP-Lite hash table entries: 8192 (order: 6, 262144 bytes)
[    0.265981] NET: Registered protocol family 1
[    0.265998] pci 0000:00:01.0: Video device with shadowed ROM
[    0.459922] PCI: CLS 64 bytes, default 64
[    0.459973] Trying to unpack rootfs image as initramfs...
[    0.702072] Freeing initrd memory: 20600K (ffff8800357b4000 - ffff880036bd2000)
[    0.702094] PCI-DMA: Using software bounce buffering for IO (SWIOTLB)
[    0.702096] software IO TLB [mem 0xa9184000-0xad184000] (64MB) mapped at [ffff8800a9184000-ffff8800ad183fff]
[    0.702137] perf: AMD NB counters detected
[    0.702208] LVT offset 0 assigned for vector 0x400
[    0.702220] perf: AMD IBS detected (0x000000ff)
[    0.702298] Scanning for low memory corruption every 60 seconds
[    0.702631] futex hash table entries: 1024 (order: 4, 65536 bytes)
[    0.702662] audit: initializing netlink subsys (disabled)
[    0.702678] audit: type=2000 audit(1474476014.592:1): initialized
[    0.702953] Initialise system trusted keyring
[    0.703102] HugeTLB registered 1 GB page size, pre-allocated 0 pages
[    0.703104] HugeTLB registered 2 MB page size, pre-allocated 0 pages
[    0.704581] zbud: loaded
[    0.704804] VFS: Disk quotas dquot_6.6.0
[    0.704833] VFS: Dquot-cache hash table entries: 512 (order 0, 4096 bytes)
[    0.705083] squashfs: version 4.0 (2009/01/31) Phillip Lougher
[    0.705362] fuse init (API version 7.23)
[    0.705478] Key type big_key registered
[    0.705488] Allocating IMA MOK and blacklist keyrings.
[    0.705919] Key type asymmetric registered
[    0.705921] Asymmetric key parser 'x509' registered
[    0.705952] Block layer SCSI generic (bsg) driver version 0.4 loaded (major 249)
[    0.705982] io scheduler noop registered
[    0.705984] io scheduler deadline registered (default)
[    0.706010] io scheduler cfq registered
[    0.706368] pci_hotplug: PCI Hot Plug PCI Core version: 0.5
[    0.706373] pciehp: PCI Express Hot Plug Controller Driver version: 0.4
[    0.706400] vesafb: mode is 1280x1024x32, linelength=5120, pages=0
[    0.706401] vesafb: scrolling: redraw
[    0.706402] vesafb: Truecolor: size=0:8:8:8, shift=0:16:8:0
[    0.706412] vesafb: framebuffer at 0xe0000000, mapped to 0xffffc90002000000, using 5120k, total 5120k
[    0.706495] Console: switching to colour frame buffer device 160x64
[    0.706512] fb0: VESA VGA frame buffer device
[    0.706582] input: Power Button as /devices/LNXSYSTM:00/LNXSYBUS:00/PNP0C0C:00/input/input0
[    0.706585] ACPI: Power Button [PWRB]
[    0.706617] input: Power Button as /devices/LNXSYSTM:00/LNXPWRBN:00/input/input1
[    0.706619] ACPI: Power Button [PWRF]
[    0.706648] ACPI: acpi_idle registered with cpuidle
[    0.707067] GHES: HEST is not enabled!
[    0.707164] Serial: 8250/16550 driver, 32 ports, IRQ sharing enabled
[    0.727537] 00:07: ttyS0 at I/O 0x3f8 (irq = 4, base_baud = 115200) is a 16550A
[    0.729093] Linux agpgart interface v0.103
[    0.731939] brd: module loaded
[    0.733063] loop: module loaded
[    0.733260] libphy: Fixed MDIO Bus: probed
[    0.733262] tun: Universal TUN/TAP device driver, 1.6
[    0.733264] tun: (C) 1999-2004 Max Krasnyansky <maxk@qualcomm.com>
[    0.733300] PPP generic driver version 2.4.2
[    0.733353] ehci_hcd: USB 2.0 'Enhanced' Host Controller (EHCI) Driver
[    0.733357] ehci-pci: EHCI PCI platform driver
[    0.733466] QUIRK: Enable AMD PLL fix
[    0.733488] ehci-pci 0000:00:12.2: EHCI Host Controller
[    0.733493] ehci-pci 0000:00:12.2: new USB bus registered, assigned bus number 1
[    0.733496] ehci-pci 0000:00:12.2: applying AMD SB700/SB800/Hudson-2/3 EHCI dummy qh workaround
[    0.733505] ehci-pci 0000:00:12.2: debug port 1
[    0.733543] ehci-pci 0000:00:12.2: irq 17, io mem 0xfeb4c000
[    0.743693] ehci-pci 0000:00:12.2: USB 2.0 started, EHCI 1.00
[    0.743754] usb usb1: New USB device found, idVendor=1d6b, idProduct=0002
[    0.743756] usb usb1: New USB device strings: Mfr=3, Product=2, SerialNumber=1
[    0.743757] usb usb1: Product: EHCI Host Controller
[    0.743759] usb usb1: Manufacturer: Linux 4.4.0-36-generic ehci_hcd
[    0.743760] usb usb1: SerialNumber: 0000:00:12.2
[    0.743891] hub 1-0:1.0: USB hub found
[    0.743895] hub 1-0:1.0: 5 ports detected
[    0.744128] ehci-pci 0000:00:13.2: EHCI Host Controller
[    0.744133] ehci-pci 0000:00:13.2: new USB bus registered, assigned bus number 2
[    0.744136] ehci-pci 0000:00:13.2: applying AMD SB700/SB800/Hudson-2/3 EHCI dummy qh workaround
[    0.744144] ehci-pci 0000:00:13.2: debug port 1
[    0.744169] ehci-pci 0000:00:13.2: irq 17, io mem 0xfeb4a000
[    0.755702] ehci-pci 0000:00:13.2: USB 2.0 started, EHCI 1.00
[    0.755767] usb usb2: New USB device found, idVendor=1d6b, idProduct=0002
[    0.755769] usb usb2: New USB device strings: Mfr=3, Product=2, SerialNumber=1
[    0.755770] usb usb2: Product: EHCI Host Controller
[    0.755772] usb usb2: Manufacturer: Linux 4.4.0-36-generic ehci_hcd
[    0.755773] usb usb2: SerialNumber: 0000:00:13.2
[    0.755900] hub 2-0:1.0: USB hub found
[    0.755905] hub 2-0:1.0: 5 ports detected
[    0.756027] ehci-platform: EHCI generic platform driver
[    0.756041] ohci_hcd: USB 1.1 'Open' Host Controller (OHCI) Driver
[    0.756044] ohci-pci: OHCI PCI platform driver
[    0.756168] ohci-pci 0000:00:12.0: OHCI PCI host controller
[    0.756172] ohci-pci 0000:00:12.0: new USB bus registered, assigned bus number 3
[    0.756192] ohci-pci 0000:00:12.0: irq 18, io mem 0xfeb4d000
[    0.815724] usb usb3: New USB device found, idVendor=1d6b, idProduct=0001
[    0.815728] usb usb3: New USB device strings: Mfr=3, Product=2, SerialNumber=1
[    0.815729] usb usb3: Product: OHCI PCI host controller
[    0.815731] usb usb3: Manufacturer: Linux 4.4.0-36-generic ohci_hcd
[    0.815732] usb usb3: SerialNumber: 0000:00:12.0
[    0.815927] hub 3-0:1.0: USB hub found
[    0.815933] hub 3-0:1.0: 5 ports detected
[    0.816159] ohci-pci 0000:00:13.0: OHCI PCI host controller
[    0.816163] ohci-pci 0000:00:13.0: new USB bus registered, assigned bus number 4
[    0.816184] ohci-pci 0000:00:13.0: irq 18, io mem 0xfeb4b000
[    0.875732] usb usb4: New USB device found, idVendor=1d6b, idProduct=0001
[    0.875735] usb usb4: New USB device strings: Mfr=3, Product=2, SerialNumber=1
[    0.875737] usb usb4: Product: OHCI PCI host controller
[    0.875738] usb usb4: Manufacturer: Linux 4.4.0-36-generic ohci_hcd
[    0.875739] usb usb4: SerialNumber: 0000:00:13.0
[    0.875934] hub 4-0:1.0: USB hub found
[    0.875941] hub 4-0:1.0: 5 ports detected
[    0.876069] ohci-platform: OHCI generic platform driver
[    0.876082] uhci_hcd: USB Universal Host Controller Interface driver
[    0.876226] xhci_hcd 0000:00:10.0: xHCI Host Controller
[    0.876230] xhci_hcd 0000:00:10.0: new USB bus registered, assigned bus number 5
[    0.876409] xhci_hcd 0000:00:10.0: hcc params 0x014040c3 hci version 0x100 quirks 0x00000418
[    0.876531] usb usb5: New USB device found, idVendor=1d6b, idProduct=0002
[    0.876533] usb usb5: New USB device strings: Mfr=3, Product=2, SerialNumber=1
[    0.876534] usb usb5: Product: xHCI Host Controller
[    0.876535] usb usb5: Manufacturer: Linux 4.4.0-36-generic xhci-hcd
[    0.876537] usb usb5: SerialNumber: 0000:00:10.0
[    0.876690] hub 5-0:1.0: USB hub found
[    0.876696] hub 5-0:1.0: 2 ports detected
[    0.876761] xhci_hcd 0000:00:10.0: xHCI Host Controller
[    0.876763] xhci_hcd 0000:00:10.0: new USB bus registered, assigned bus number 6
[    0.879826] usb usb6: We don't know the algorithms for LPM for this host, disabling LPM.
[    0.879838] usb usb6: New USB device found, idVendor=1d6b, idProduct=0003
[    0.879840] usb usb6: New USB device strings: Mfr=3, Product=2, SerialNumber=1
[    0.879841] usb usb6: Product: xHCI Host Controller
[    0.879842] usb usb6: Manufacturer: Linux 4.4.0-36-generic xhci-hcd
[    0.879843] usb usb6: SerialNumber: 0000:00:10.0
[    0.879947] hub 6-0:1.0: USB hub found
[    0.879953] hub 6-0:1.0: 2 ports detected
[    0.880055] xhci_hcd 0000:03:00.0: xHCI Host Controller
[    0.880059] xhci_hcd 0000:03:00.0: new USB bus registered, assigned bus number 7
[    0.940427] xhci_hcd 0000:03:00.0: hcc params 0x0200e081 hci version 0x100 quirks 0x00000010
[    0.940535] usb usb7: New USB device found, idVendor=1d6b, idProduct=0002
[    0.940536] usb usb7: New USB device strings: Mfr=3, Product=2, SerialNumber=1
[    0.940538] usb usb7: Product: xHCI Host Controller
[    0.940539] usb usb7: Manufacturer: Linux 4.4.0-36-generic xhci-hcd
[    0.940540] usb usb7: SerialNumber: 0000:03:00.0
[    0.940692] hub 7-0:1.0: USB hub found
[    0.940698] hub 7-0:1.0: 2 ports detected
[    0.940765] xhci_hcd 0000:03:00.0: xHCI Host Controller
[    0.940767] xhci_hcd 0000:03:00.0: new USB bus registered, assigned bus number 8
[    0.940802] usb usb8: We don't know the algorithms for LPM for this host, disabling LPM.
[    0.940815] usb usb8: New USB device found, idVendor=1d6b, idProduct=0003
[    0.940816] usb usb8: New USB device strings: Mfr=3, Product=2, SerialNumber=1
[    0.940817] usb usb8: Product: xHCI Host Controller
[    0.940818] usb usb8: Manufacturer: Linux 4.4.0-36-generic xhci-hcd
[    0.940819] usb usb8: SerialNumber: 0000:03:00.0
[    0.940900] hub 8-0:1.0: USB hub found
[    0.940906] hub 8-0:1.0: 2 ports detected
[    0.941000] i8042: PNP: No PS/2 controller found. Probing ports directly.
[    0.941419] serio: i8042 KBD port at 0x60,0x64 irq 1
[    0.941422] serio: i8042 AUX port at 0x60,0x64 irq 12
[    0.941569] mousedev: PS/2 mouse device common for all mice
[    0.941699] rtc_cmos 00:04: RTC can wake from S4
[    0.941803] rtc_cmos 00:04: rtc core: registered rtc_cmos as rtc0
[    0.941822] rtc_cmos 00:04: alarms up to one month, y3k, 114 bytes nvram, hpet irqs
[    0.941829] i2c /dev entries driver
[    0.941876] device-mapper: uevent: version 1.0.3
[    0.941932] device-mapper: ioctl: 4.34.0-ioctl (2015-10-28) initialised: dm-devel@redhat.com
[    0.941949] ledtrig-cpu: registered to indicate activity on CPUs
[    0.942295] NET: Registered protocol family 10
[    0.942457] NET: Registered protocol family 17
[    0.942468] Key type dns_resolver registered
[    0.942730] microcode: CPU0: patch_level=0x06001119
[    0.942736] microcode: CPU1: patch_level=0x06001119
[    0.942746] microcode: CPU2: patch_level=0x06001119
[    0.942753] microcode: CPU3: patch_level=0x06001119
[    0.942787] microcode: Microcode Update Driver: v2.01 <tigran@aivazian.fsnet.co.uk>, Peter Oruba
[    0.942934] registered taskstats version 1
[    0.942944] Loading compiled-in X.509 certificates
[    0.943663] Loaded X.509 cert 'Build time autogenerated kernel key: 6d1a628313c57c2b040732ed4f3ff204c35686cf'
[    0.943690] zswap: loaded using pool lzo/zbud
[    0.945460] Key type trusted registered
[    0.948282] Key type encrypted registered
[    0.948290] AppArmor: AppArmor sha1 policy hashing enabled
[    0.948293] ima: No TPM chip found, activating TPM-bypass!
[    0.948315] evm: HMAC attrs: 0x1
[    0.948607]   Magic number: 0:427:692
[    0.948700] rtc_cmos 00:04: setting system clock to 2016-09-21 16:40:15 UTC (1474476015)
[    0.948842] acpi-cpufreq: overriding BIOS provided _PSD data
[    0.948969] BIOS EDD facility v0.16 2004-Jun-25, 0 devices found
[    0.948970] EDD information not available.
[    0.949036] PM: Hibernation image not present or could not be loaded.
[    0.949894] Freeing unused kernel memory: 1484K (ffffffff81d45000 - ffffffff81eb8000)
[    0.949896] Write protecting the kernel read-only data: 12288k
[    0.950074] Freeing unused kernel memory: 4K (ffff8800017ff000 - ffff880001800000)
[    0.950346] Freeing unused kernel memory: 136K (ffff880001bde000 - ffff880001c00000)
[    0.961233] systemd-udevd[134]: starting version 204
[    0.970473] FUJITSU Extended Socket Network Device Driver - version 1.0 - Copyright (c) 2015 FUJITSU LIMITED
[    0.979168] ahci 0000:00:11.0: version 3.0
[    0.979394] ahci 0000:00:11.0: AHCI 0001.0300 32 slots 6 ports 6 Gbps 0x3f impl SATA mode
[    0.979397] ahci 0000:00:11.0: flags: 64bit ncq sntf ilck pm led clo pmp pio 
[    0.980325] scsi host0: ahci
[    0.980484] scsi host1: ahci
[    0.980641] scsi host2: ahci
[    0.980735] scsi host3: ahci
[    0.980921] r8169 Gigabit Ethernet driver 2.3LK-NAPI loaded
[    0.981267] scsi host4: ahci
[    0.981363] scsi host5: ahci
[    0.981409] ata1: SATA max UDMA/133 abar m2048@0xfeb4e000 port 0xfeb4e100 irq 34
[    0.981412] ata2: SATA max UDMA/133 abar m2048@0xfeb4e000 port 0xfeb4e180 irq 34
[    0.981414] ata3: SATA max UDMA/133 abar m2048@0xfeb4e000 port 0xfeb4e200 irq 34
[    0.981415] ata4: SATA max UDMA/133 abar m2048@0xfeb4e000 port 0xfeb4e280 irq 34
[    0.981417] ata5: SATA max UDMA/133 abar m2048@0xfeb4e000 port 0xfeb4e300 irq 34
[    0.981419] ata6: SATA max UDMA/133 abar m2048@0xfeb4e000 port 0xfeb4e380 irq 34
[    0.988668] r8169 0000:04:00.0 eth0: RTL8168g/8111g at 0xffffc90001b52000, 14:dd:a9:79:64:8d, XID 0c000800 IRQ 35
[    0.988672] r8169 0000:04:00.0 eth0: jumbo features [frames: 9200 bytes, tx checksumming: ko]
[    1.187739] usb 5-2: new high-speed USB device number 2 using xhci_hcd
[    1.299789] ata4: SATA link down (SStatus 0 SControl 300)
[    1.299817] ata3: SATA link down (SStatus 0 SControl 300)
[    1.299842] ata6: SATA link down (SStatus 0 SControl 300)
[    1.299886] ata5: SATA link down (SStatus 0 SControl 300)
[    1.307765] usb 7-1: new full-speed USB device number 2 using xhci_hcd
[    1.329081] usb 5-2: New USB device found, idVendor=05e3, idProduct=0745
[    1.329085] usb 5-2: New USB device strings: Mfr=0, Product=1, SerialNumber=2
[    1.329087] usb 5-2: Product: USB Storage
[    1.329088] usb 5-2: SerialNumber: 000000000903
[    1.332594] usb-storage 5-2:1.0: USB Mass Storage device detected
[    1.332730] scsi host6: usb-storage 5-2:1.0
[    1.332796] usbcore: registered new interface driver usb-storage
[    1.333891] usbcore: registered new interface driver uas
[    1.471778] ata2: SATA link up 1.5 Gbps (SStatus 113 SControl 300)
[    1.471814] ata1: SATA link up 6.0 Gbps (SStatus 133 SControl 300)
[    1.472477] ata1.00: ATA-9: WDC WD10EZEX-00BN5A0, 01.01A01, max UDMA/133
[    1.472481] ata1.00: 1953525168 sectors, multi 16: LBA48 NCQ (depth 31/32), AA
[    1.473200] ata1.00: configured for UDMA/133
[    1.473384] scsi 0:0:0:0: Direct-Access     ATA      WDC WD10EZEX-00B 1A01 PQ: 0 ANSI: 5
[    1.473590] sd 0:0:0:0: [sda] 1953525168 512-byte logical blocks: (1.00 TB/932 GiB)
[    1.473592] sd 0:0:0:0: [sda] 4096-byte physical blocks
[    1.473631] sd 0:0:0:0: Attached scsi generic sg0 type 0
[    1.473651] sd 0:0:0:0: [sda] Write Protect is off
[    1.473655] sd 0:0:0:0: [sda] Mode Sense: 00 3a 00 00
[    1.473686] sd 0:0:0:0: [sda] Write cache: enabled, read cache: enabled, doesn't support DPO or FUA
[    1.473962] ata2.00: ATAPI: ASUS     DRW-24F1ST   a, 1.00, max UDMA/100
[    1.475082] ata2.00: configured for UDMA/100
[    1.476668] scsi 1:0:0:0: CD-ROM            ASUS     DRW-24F1ST   a   1.00 PQ: 0 ANSI: 5
[    1.495832] sr 1:0:0:0: [sr0] scsi3-mmc drive: 48x/48x writer dvd-ram cd/rw xa/form2 cdda tray
[    1.495836] cdrom: Uniform CD-ROM driver Revision: 3.20
[    1.496009] sr 1:0:0:0: Attached scsi CD-ROM sr0
[    1.496097] sr 1:0:0:0: Attached scsi generic sg1 type 5
[    1.520975] usb 7-1: New USB device found, idVendor=058f, idProduct=9254
[    1.520978] usb 7-1: New USB device strings: Mfr=1, Product=2, SerialNumber=0
[    1.520980] usb 7-1: Product: Generic USB Hub
[    1.520981] usb 7-1: Manufacturer: ALCOR
[    1.521107] usb 7-1: ep 0x81 - rounding interval to 1024 microframes, ep desc says 2040 microframes
[    1.522521] hub 7-1:1.0: USB hub found
[    1.522836] hub 7-1:1.0: 4 ports detected
[    1.540454]  sda: sda1 sda2 < sda5 >
[    1.540940] sd 0:0:0:0: [sda] Attached SCSI disk
[    1.699788] tsc: Refined TSC clocksource calibration: 3890.257 MHz
[    1.699793] clocksource: tsc: mask: 0xffffffffffffffff max_cycles: 0x7026cd4cb37, max_idle_ns: 881590669116 ns
[    1.795746] usb 7-1.1: new low-speed USB device number 3 using xhci_hcd
[    1.916222] usb 7-1.1: New USB device found, idVendor=413c, idProduct=2005
[    1.916226] usb 7-1.1: New USB device strings: Mfr=1, Product=2, SerialNumber=0
[    1.916228] usb 7-1.1: Product: DELL USB Keyboard
[    1.916229] usb 7-1.1: Manufacturer: DELL
[    1.916356] usb 7-1.1: ep 0x81 - rounding interval to 64 microframes, ep desc says 80 microframes
[    1.920505] hidraw: raw HID events driver (C) Jiri Kosina
[    1.925107] usbcore: registered new interface driver usbhid
[    1.925117] usbhid: USB HID core driver
[    1.926573] input: DELL DELL USB Keyboard as /devices/pci0000:00/0000:00:15.1/0000:03:00.0/usb7/7-1/7-1.1/7-1.1:1.0/0003:413C:2005.0001/input/input5
[    1.979951] hid-generic 0003:413C:2005.0001: input,hidraw0: USB HID v1.10 Keyboard [DELL DELL USB Keyboard] on usb-0000:03:00.0-1.1/input0
[    1.987761] usb 7-1.3: new full-speed USB device number 4 using xhci_hcd
[    1.989414] EXT4-fs (sda1): mounted filesystem with ordered data mode. Opts: (null)
[    2.102867] usb 7-1.3: not running at top speed; connect to a high speed hub
[    2.104214] usb 7-1.3: New USB device found, idVendor=04f9, idProduct=003a
[    2.104217] usb 7-1.3: New USB device strings: Mfr=1, Product=2, SerialNumber=3
[    2.104219] usb 7-1.3: Product: HL-5350DN series
[    2.104220] usb 7-1.3: Manufacturer: Brother
[    2.104221] usb 7-1.3: SerialNumber: K0J807812
[    2.179821] usb 7-1.4: new low-speed USB device number 5 using xhci_hcd
[    2.299868] usb 7-1.4: New USB device found, idVendor=093a, idProduct=2510
[    2.299871] usb 7-1.4: New USB device strings: Mfr=1, Product=2, SerialNumber=0
[    2.299873] usb 7-1.4: Product: USB OPTICAL MOUSE
[    2.299874] usb 7-1.4: Manufacturer: PIXART
[    2.299981] usb 7-1.4: ep 0x81 - rounding interval to 64 microframes, ep desc says 80 microframes
[    2.305562] input: PIXART USB OPTICAL MOUSE as /devices/pci0000:00/0000:00:15.1/0000:03:00.0/usb7/7-1/7-1.4/7-1.4:1.0/0003:093A:2510.0002/input/input6
[    2.305696] hid-generic 0003:093A:2510.0002: input,hidraw1: USB HID v1.11 Mouse [PIXART USB OPTICAL MOUSE] on usb-0000:03:00.0-1.4/input0
[    2.333664] scsi 6:0:0:0: Direct-Access     Generic  STORAGE DEVICE   0903 PQ: 0 ANSI: 6
[    2.333971] sd 6:0:0:0: Attached scsi generic sg2 type 0
[    2.336417] sd 6:0:0:0: [sdb] Attached SCSI removable disk
[    2.452021] random: init urandom read with 96 bits of entropy available
[    2.699923] clocksource: Switched to clocksource tsc
[    2.899159] random: nonblocking pool is initialized
[    2.952910] init: plymouth-upstart-bridge main process (218) terminated with status 1
[    2.952921] init: plymouth-upstart-bridge main process ended, respawning
[    2.954809] init: plymouth-upstart-bridge main process (232) terminated with status 1
[    2.954819] init: plymouth-upstart-bridge main process ended, respawning
[    2.956210] init: plymouth-upstart-bridge main process (234) terminated with status 1
[    2.956220] init: plymouth-upstart-bridge main process ended, respawning
[    2.958298] init: plymouth-upstart-bridge main process (235) terminated with status 1
[    2.958308] init: plymouth-upstart-bridge main process ended, respawning
[    2.959614] init: plymouth-upstart-bridge main process (237) terminated with status 1
[    2.959623] init: plymouth-upstart-bridge main process ended, respawning
[    2.961257] init: plymouth-upstart-bridge main process (238) terminated with status 1
[    2.961265] init: plymouth-upstart-bridge main process ended, respawning
[    2.963153] init: plymouth-upstart-bridge main process (240) terminated with status 1
[    2.963164] init: plymouth-upstart-bridge main process ended, respawning
[    2.965046] init: plymouth-upstart-bridge main process (241) terminated with status 1
[    2.965055] init: plymouth-upstart-bridge main process ended, respawning
[    2.966401] init: plymouth-upstart-bridge main process (243) terminated with status 1
[    2.966409] init: plymouth-upstart-bridge main process ended, respawning
[    2.968233] init: plymouth-upstart-bridge main process (244) terminated with status 1
[    2.968241] init: plymouth-upstart-bridge main process ended, respawning
[    2.969738] init: plymouth-upstart-bridge main process (246) terminated with status 1
[    2.969746] init: plymouth-upstart-bridge respawning too fast, stopped
[    8.583716] Adding 15929340k swap on /dev/sda5.  Priority:-1 extents:1 across:15929340k FS
[    8.925974] systemd-udevd[351]: starting version 204
[    9.262428] lp: driver loaded but no devices found
[    9.269077] ppdev: user-space parallel port driver
[    9.271071] ACPI: Video Device [VGA] (multi-head: yes  rom: no  post: no)
[    9.271102] [Firmware Bug]: ACPI: No _BQC method, cannot determine initial brightness
[    9.271168] ACPI Error: [AFN7] Namespace lookup failure, AE_NOT_FOUND (20150930/psargs-359)
[    9.271173] ACPI Error: Method parse/execution failed [\_SB.PCI0.VGA.LCD._BCM] (Node ffff88040eca5c30), AE_NOT_FOUND (20150930/psparse-542)
[    9.271179] ACPI Error: Evaluating _BCM failed (20150930/video-367)
[    9.271253] input: Video Bus as /devices/LNXSYSTM:00/LNXSYBUS:00/PNP0A08:00/LNXVIDEO:00/input/input7
[    9.312998] piix4_smbus 0000:00:14.0: SMBus Host Controller at 0xb00, revision 0
[    9.313530] piix4_smbus 0000:00:14.0: Auxiliary SMBus Host Controller at 0xb20
[    9.323675] shpchp: Standard Hot Plug PCI Controller Driver version: 0.4
[    9.324863] wmi: Mapper loaded
[    9.394662] AVX version of gcm_enc/dec engaged.
[    9.394665] AES CTR mode by8 optimization enabled
[    9.403681] [drm] Initialized drm 1.1.0 20060810
[    9.489302] asus_wmi: ASUS WMI generic driver loaded
[    9.490552] asus_wmi: Initialization: 0x0
[    9.490575] asus_wmi: BIOS WMI version: 0.9
[    9.490620] asus_wmi: SFUN value: 0x0
[    9.490938] input: Eee PC WMI hotkeys as /devices/platform/eeepc-wmi/input/input8
[    9.491188] asus_wmi: Number of fans: 1
[    9.543371] kvm: Nested Virtualization enabled
[    9.543375] kvm: Nested Paging enabled
[    9.547806] snd_hda_intel 0000:00:01.1: Force to non-snoop mode
[    9.557468] input: HDA ATI HDMI HDMI/DP,pcm=3 as /devices/pci0000:00/0000:00:01.1/sound/card0/input9
[    9.561406] snd_hda_codec_realtek hdaudioC1D0: autoconfig for ALC887-VD: line_outs=1 (0x14/0x0/0x0/0x0/0x0) type:line
[    9.561409] snd_hda_codec_realtek hdaudioC1D0:    speaker_outs=0 (0x0/0x0/0x0/0x0/0x0)
[    9.561412] snd_hda_codec_realtek hdaudioC1D0:    hp_outs=1 (0x1b/0x0/0x0/0x0/0x0)
[    9.561413] snd_hda_codec_realtek hdaudioC1D0:    mono: mono_out=0x0
[    9.561414] snd_hda_codec_realtek hdaudioC1D0:    dig-out=0x11/0x0
[    9.561415] snd_hda_codec_realtek hdaudioC1D0:    inputs:
[    9.561420] snd_hda_codec_realtek hdaudioC1D0:      Front Mic=0x19
[    9.561422] snd_hda_codec_realtek hdaudioC1D0:      Rear Mic=0x18
[    9.561423] snd_hda_codec_realtek hdaudioC1D0:      Line=0x1a
[    9.574737] input: HD-Audio Generic Front Mic as /devices/pci0000:00/0000:00:14.2/sound/card1/input10
[    9.574831] input: HD-Audio Generic Rear Mic as /devices/pci0000:00/0000:00:14.2/sound/card1/input11
[    9.574905] input: HD-Audio Generic Line as /devices/pci0000:00/0000:00:14.2/sound/card1/input12
[    9.575002] input: HD-Audio Generic Line Out as /devices/pci0000:00/0000:00:14.2/sound/card1/input13
[    9.575150] input: HD-Audio Generic Front Headphone as /devices/pci0000:00/0000:00:14.2/sound/card1/input14
[   10.128077] audit: type=1400 audit(1474476024.677:2): apparmor="STATUS" operation="profile_load" profile="unconfined" name="/sbin/dhclient" pid=463 comm="apparmor_parser"
[   10.128083] audit: type=1400 audit(1474476024.677:3): apparmor="STATUS" operation="profile_load" profile="unconfined" name="/usr/lib/NetworkManager/nm-dhcp-client.action" pid=463 comm="apparmor_parser"
[   10.128086] audit: type=1400 audit(1474476024.677:4): apparmor="STATUS" operation="profile_load" profile="unconfined" name="/usr/lib/connman/scripts/dhclient-script" pid=463 comm="apparmor_parser"
[   10.128098] audit: type=1400 audit(1474476024.677:5): apparmor="STATUS" operation="profile_replace" profile="unconfined" name="/sbin/dhclient" pid=456 comm="apparmor_parser"
[   10.128106] audit: type=1400 audit(1474476024.677:6): apparmor="STATUS" operation="profile_replace" profile="unconfined" name="/usr/lib/NetworkManager/nm-dhcp-client.action" pid=456 comm="apparmor_parser"
[   10.128109] audit: type=1400 audit(1474476024.677:7): apparmor="STATUS" operation="profile_replace" profile="unconfined" name="/usr/lib/connman/scripts/dhclient-script" pid=456 comm="apparmor_parser"
[   10.128401] audit: type=1400 audit(1474476024.677:8): apparmor="STATUS" operation="profile_replace" profile="unconfined" name="/usr/lib/NetworkManager/nm-dhcp-client.action" pid=463 comm="apparmor_parser"
[   10.128406] audit: type=1400 audit(1474476024.677:9): apparmor="STATUS" operation="profile_replace" profile="unconfined" name="/usr/lib/connman/scripts/dhclient-script" pid=463 comm="apparmor_parser"
[   10.128424] audit: type=1400 audit(1474476024.677:10): apparmor="STATUS" operation="profile_replace" profile="unconfined" name="/usr/lib/NetworkManager/nm-dhcp-client.action" pid=456 comm="apparmor_parser"
[   10.128428] audit: type=1400 audit(1474476024.677:11): apparmor="STATUS" operation="profile_replace" profile="unconfined" name="/usr/lib/connman/scripts/dhclient-script" pid=456 comm="apparmor_parser"
[   10.403928] [drm] radeon kernel modesetting enabled.
[   10.406508] AMD IOMMUv2 driver by Joerg Roedel <jroedel@suse.de>
[   10.406511] AMD IOMMUv2 functionality not available on this system
[   10.468278] Found CRAT image with size=1400
[   10.468281] Parsing CRAT table with 1 nodes
[   10.468283] Found CU entry in CRAT table with proximity_domain=0 caps=0
[   10.468284] CU CPU: cores=4 id_base=16
[   10.468286] Found memory entry in CRAT table with proximity_domain=0
[   10.468287] Found memory entry in CRAT table with proximity_domain=0
[   10.468288] Found memory entry in CRAT table with proximity_domain=0
[   10.468289] Found memory entry in CRAT table with proximity_domain=0
[   10.468290] Found cache entry in CRAT table with processor_id=16
[   10.468291] Found cache entry in CRAT table with processor_id=16
[   10.468292] Found cache entry in CRAT table with processor_id=16
[   10.468293] Found cache entry in CRAT table with processor_id=17
[   10.468293] Found cache entry in CRAT table with processor_id=18
[   10.468294] Found cache entry in CRAT table with processor_id=18
[   10.468295] Found cache entry in CRAT table with processor_id=18
[   10.468296] Found cache entry in CRAT table with processor_id=19
[   10.468297] Found TLB entry in CRAT table (not processing)
[   10.468298] Found TLB entry in CRAT table (not processing)
[   10.468299] Found TLB entry in CRAT table (not processing)
[   10.468299] Found TLB entry in CRAT table (not processing)
[   10.468300] Found TLB entry in CRAT table (not processing)
[   10.468301] Found TLB entry in CRAT table (not processing)
[   10.468301] Found TLB entry in CRAT table (not processing)
[   10.468302] Found TLB entry in CRAT table (not processing)
[   10.468303] Found TLB entry in CRAT table (not processing)
[   10.468304] Found TLB entry in CRAT table (not processing)
[   10.468305] Creating topology SYSFS entries
[   10.468331] Finished initializing topology ret=0
[   10.468523] kfd kfd: Initialized module
[   10.468892] checking generic (e0000000 500000) vs hw (e0000000 10000000)
[   10.468896] fb: switching to radeondrmfb from VESA VGA
[   10.468951] Console: switching to colour dummy device 80x25
[   10.469320] [drm] initializing kernel modesetting (ARUBA 0x1002:0x990E 0x1043:0x8526).
[   10.469334] [drm] register mmio base: 0xFEB00000
[   10.469335] [drm] register mmio size: 262144
[   10.469377] ATOM BIOS: 113
[   10.469433] radeon 0000:00:01.0: VRAM: 768M 0x0000000000000000 - 0x000000002FFFFFFF (768M used)
[   10.469435] radeon 0000:00:01.0: GTT: 1024M 0x0000000030000000 - 0x000000006FFFFFFF
[   10.469437] [drm] Detected VRAM RAM=768M, BAR=256M
[   10.469438] [drm] RAM width 64bits DDR
[   10.469517] [TTM] Zone  kernel: Available graphics memory: 7799622 kiB
[   10.469518] [TTM] Zone   dma32: Available graphics memory: 2097152 kiB
[   10.469519] [TTM] Initializing pool allocator
[   10.469523] [TTM] Initializing DMA pool allocator
[   10.469539] [drm] radeon: 768M of VRAM memory ready
[   10.469541] [drm] radeon: 1024M of GTT memory ready.
[   10.469553] [drm] Loading ARUBA Microcode
[   11.055515] [drm] Internal thermal controller without fan control
[   11.055695] [drm] radeon: dpm initialized
[   11.363669] [drm] Found VCE firmware/feedback version 50.0.1 / 17!
[   11.363691] [drm] GART: num cpu pages 262144, num gpu pages 262144
[   11.390611] [drm] PCIE GART of 1024M enabled (table at 0x00000000002E8000).
[   11.390732] radeon 0000:00:01.0: WB enabled
[   11.390735] radeon 0000:00:01.0: fence driver on ring 0 use gpu addr 0x0000000030000c00 and cpu addr 0xffff8800a8709c00
[   11.391465] radeon 0000:00:01.0: fence driver on ring 5 use gpu addr 0x0000000000075a18 and cpu addr 0xffffc90002035a18
[   11.411490] radeon 0000:00:01.0: fence driver on ring 6 use gpu addr 0x0000000030000c18 and cpu addr 0xffff8800a8709c18
[   11.411492] radeon 0000:00:01.0: fence driver on ring 7 use gpu addr 0x0000000030000c1c and cpu addr 0xffff8800a8709c1c
[   11.411494] radeon 0000:00:01.0: fence driver on ring 1 use gpu addr 0x0000000030000c04 and cpu addr 0xffff8800a8709c04
[   11.411495] radeon 0000:00:01.0: fence driver on ring 2 use gpu addr 0x0000000030000c08 and cpu addr 0xffff8800a8709c08
[   11.411497] radeon 0000:00:01.0: fence driver on ring 3 use gpu addr 0x0000000030000c0c and cpu addr 0xffff8800a8709c0c
[   11.411498] radeon 0000:00:01.0: fence driver on ring 4 use gpu addr 0x0000000030000c10 and cpu addr 0xffff8800a8709c10
[   11.411500] [drm] Supports vblank timestamp caching Rev 2 (21.10.2013).
[   11.411501] [drm] Driver supports precise vblank timestamp query.
[   11.411502] radeon 0000:00:01.0: radeon: MSI limited to 32-bit
[   11.411526] radeon 0000:00:01.0: radeon: using MSI.
[   11.411821] [drm] radeon: irq initialized.
[   11.430803] [drm] ring test on 0 succeeded in 3 usecs
[   11.430810] [drm] ring test on 3 succeeded in 3 usecs
[   11.430815] [drm] ring test on 4 succeeded in 3 usecs
[   11.476489] [drm] ring test on 5 succeeded in 1 usecs
[   11.496343] [drm] UVD initialized successfully.
[   11.605538] [drm] ring test on 6 succeeded in 17 usecs
[   11.605550] [drm] ring test on 7 succeeded in 3 usecs
[   11.605551] [drm] VCE initialized successfully.
[   11.606354] [drm] ib test on ring 0 succeeded in 0 usecs
[   11.606884] [drm] ib test on ring 3 succeeded in 0 usecs
[   11.607413] [drm] ib test on ring 4 succeeded in 0 usecs
[   11.627799] [drm] ib test on ring 5 succeeded
[   11.635586] EXT4-fs (sda1): re-mounted. Opts: errors=remount-ro
[   11.814729] init: failsafe main process (669) killed by TERM signal
[   12.145329] [drm] ib test on ring 6 succeeded
[   12.145860] [drm] ib test on ring 7 succeeded
[   12.146652] [drm] Radeon Display Connectors
[   12.146653] [drm] Connector 0:
[   12.146654] [drm]   HDMI-A-1
[   12.146655] [drm]   HPD1
[   12.146656] [drm]   DDC: 0x6530 0x6530 0x6534 0x6534 0x6538 0x6538 0x653c 0x653c
[   12.146657] [drm]   Encoders:
[   12.146658] [drm]     DFP1: INTERNAL_UNIPHY2
[   12.146659] [drm] Connector 1:
[   12.146659] [drm]   VGA-1
[   12.146660] [drm]   HPD2
[   12.146661] [drm]   DDC: 0x6540 0x6540 0x6544 0x6544 0x6548 0x6548 0x654c 0x654c
[   12.146662] [drm]   Encoders:
[   12.146662] [drm]     CRT1: INTERNAL_UNIPHY2
[   12.146663] [drm]     CRT1: NUTMEG
[   12.146664] [drm] Connector 2:
[   12.146664] [drm]   DVI-D-1
[   12.146665] [drm]   HPD3
[   12.146666] [drm]   DDC: 0x6550 0x6550 0x6554 0x6554 0x6558 0x6558 0x655c 0x655c
[   12.146667] [drm]   Encoders:
[   12.146667] [drm]     DFP2: INTERNAL_UNIPHY
[   12.250349] Bluetooth: Core ver 2.21
[   12.250368] NET: Registered protocol family 31
[   12.250370] Bluetooth: HCI device and connection manager initialized
[   12.250374] Bluetooth: HCI socket layer initialized
[   12.250377] Bluetooth: L2CAP socket layer initialized
[   12.250383] Bluetooth: SCO socket layer initialized
[   12.266318] Bluetooth: RFCOMM TTY layer initialized
[   12.266327] Bluetooth: RFCOMM socket layer initialized
[   12.266332] Bluetooth: RFCOMM ver 1.11
[   12.319808] Bluetooth: BNEP (Ethernet Emulation) ver 1.3
[   12.319812] Bluetooth: BNEP filters: protocol multicast
[   12.319817] Bluetooth: BNEP socket layer initialized
[   12.327106] [drm] fb mappable at 0xE04EC000
[   12.327109] [drm] vram apper at 0xE0000000
[   12.327110] [drm] size 5242880
[   12.327111] [drm] fb depth is 24
[   12.327112] [drm]    pitch is 5120
[   12.327245] fbcon: radeondrmfb (fb0) is primary device
[   12.327362] Console: switching to colour frame buffer device 160x64
[   12.327382] radeon 0000:00:01.0: fb0: radeondrmfb frame buffer device
[   12.345071] [drm] Initialized radeon 2.43.0 20080528 for 0000:00:01.0 on minor 0
[   12.380332] init: cups main process (741) killed by HUP signal
[   12.380342] init: cups main process ended, respawning
[   12.662652] usblp 7-1.3:1.0: usblp0: USB Bidirectional printer dev 4 if 0 alt 0 proto 2 vid 0x04F9 pid 0x003A
[   12.662681] usbcore: registered new interface driver usblp
[   13.287481] r8169 0000:04:00.0 eth0: link down
[   13.287486] r8169 0000:04:00.0 eth0: link down
[   13.287546] IPv6: ADDRCONF(NETDEV_UP): eth0: link is not ready
[   17.013791] r8169 0000:04:00.0 eth0: link up
[   17.013802] IPv6: ADDRCONF(NETDEV_CHANGE): eth0: link becomes ready
[   17.481298] [drm:atom_op_jump [radeon]] *ERROR* atombios stuck in loop for more than 5secs aborting
[   17.481314] [drm:atom_execute_table_locked [radeon]] *ERROR* atombios stuck executing E4DE (len 2650, WS 0, PS 8) @ 0xEAE6
[   17.497444] audit_printk_skb: 138 callbacks suppressed
[   17.497447] audit: type=1400 audit(1474476032.045:58): apparmor="STATUS" operation="profile_replace" profile="unconfined" name="/usr/bin/evince-previewer//sanitized_helper" pid=798 comm="apparmor_parser"
[   17.497454] audit: type=1400 audit(1474476032.045:59): apparmor="STATUS" operation="profile_replace" profile="unconfined" name="/usr/bin/evince-thumbnailer" pid=798 comm="apparmor_parser"
[   17.513452] audit: type=1400 audit(1474476032.061:60): apparmor="STATUS" operation="profile_replace" profile="unconfined" name="/usr/bin/evince-thumbnailer//sanitized_helper" pid=798 comm="apparmor_parser"
[   17.514572] audit: type=1400 audit(1474476032.061:61): apparmor="STATUS" operation="profile_replace" profile="unconfined" name="/usr/bin/evince-previewer//sanitized_helper" pid=798 comm="apparmor_parser"
[   17.514576] audit: type=1400 audit(1474476032.061:62): apparmor="STATUS" operation="profile_replace" profile="unconfined" name="/usr/bin/evince-thumbnailer" pid=798 comm="apparmor_parser"
[   17.529508] audit: type=1400 audit(1474476032.077:63): apparmor="STATUS" operation="profile_replace" profile="unconfined" name="/usr/bin/evince-thumbnailer//sanitized_helper" pid=798 comm="apparmor_parser"
[   17.529966] audit: type=1400 audit(1474476032.077:64): apparmor="STATUS" operation="profile_replace" profile="unconfined" name="/usr/bin/evince-thumbnailer" pid=798 comm="apparmor_parser"
[   17.553450] audit: type=1400 audit(1474476032.101:65): apparmor="STATUS" operation="profile_replace" profile="unconfined" name="/usr/bin/evince-thumbnailer//sanitized_helper" pid=798 comm="apparmor_parser"
[   17.553980] audit: type=1400 audit(1474476032.101:66): apparmor="STATUS" operation="profile_replace" profile="unconfined" name="/usr/bin/evince-thumbnailer//sanitized_helper" pid=798 comm="apparmor_parser"
[   17.576501] [drm:radeon_dp_link_train [radeon]] *ERROR* clock recovery reached max voltage
[   17.576521] [drm:radeon_dp_link_train [radeon]] *ERROR* clock recovery failed
[   18.224225] audit: type=1400 audit(1474476032.769:67): apparmor="STATUS" operation="profile_replace" profile="unconfined" name="/usr/sbin/cups-browsed" pid=1059 comm="apparmor_parser"
[   42.111866] audit: type=1400 audit(1474476056.659:68): apparmor="STATUS" operation="profile_replace" profile="unconfined" name="/usr/lib/cups/backend/cups-pdf" pid=2109 comm="apparmor_parser"
[   42.111874] audit: type=1400 audit(1474476056.659:69): apparmor="STATUS" operation="profile_replace" profile="unconfined" name="/usr/sbin/cupsd" pid=2109 comm="apparmor_parser"
[   42.112198] audit: type=1400 audit(1474476056.659:70): apparmor="STATUS" operation="profile_replace" profile="unconfined" name="/usr/sbin/cupsd" pid=2109 comm="apparmor_parser"
vadee@vadee:~$ systemd-analyze blame 
systemd-analyze : commande introuvable
vadee@vadee:~$ sudo smartctl  -s on  -a /dev/sda
[sudo] password for vadee: 
smartctl 6.2 2013-07-26 r3841 [x86_64-linux-4.4.0-36-generic] (local build)
Copyright (C) 2002-13, Bruce Allen, Christian Franke, www.smartmontools.org

=== START OF INFORMATION SECTION ===
Model Family:     Western Digital Caviar Blue (SATA 6Gb/s)
Device Model:     WDC WD10EZEX-00BN5A0
Serial Number:    WD-WCC3F6HLC90R
LU WWN Device Id: 5 0014ee 2617c8b1a
Firmware Version: 01.01A01
User Capacity:    1 000 204 886 016 bytes [1,00 TB]
Sector Sizes:     512 bytes logical, 4096 bytes physical
Rotation Rate:    7200 rpm
Device is:        In smartctl database [for details use: -P show]
ATA Version is:   ACS-2, ACS-3 T13/2161-D revision 3b
SATA Version is:  SATA 3.1, 6.0 Gb/s (current: 6.0 Gb/s)
Local Time is:    Wed Sep 21 19:48:23 2016 CEST
SMART support is: Available - device has SMART capability.
SMART support is: Enabled

=== START OF ENABLE/DISABLE COMMANDS SECTION ===
SMART Enabled.

=== START OF READ SMART DATA SECTION ===
SMART overall-health self-assessment test result: PASSED

General SMART Values:
Offline data collection status:  (0x82)	Offline data collection activity
					was completed without error.
					Auto Offline Data Collection: Enabled.
Self-test execution status:      (   0)	The previous self-test routine completed
					without error or no self-test has ever 
					been run.
Total time to complete Offline 
data collection: 		(10920) seconds.
Offline data collection
capabilities: 			 (0x7b) SMART execute Offline immediate.
					Auto Offline data collection on/off support.
					Suspend Offline collection upon new
					command.
					Offline surface scan supported.
					Self-test supported.
					Conveyance Self-test supported.
					Selective Self-test supported.
SMART capabilities:            (0x0003)	Saves SMART data before entering
					power-saving mode.
					Supports SMART auto save timer.
Error logging capability:        (0x01)	Error logging supported.
					General Purpose Logging supported.
Short self-test routine 
recommended polling time: 	 (   2) minutes.
Extended self-test routine
recommended polling time: 	 ( 114) minutes.
Conveyance self-test routine
recommended polling time: 	 (   5) minutes.
SCT capabilities: 	       (0x3035)	SCT Status supported.
					SCT Feature Control supported.
					SCT Data Table supported.

SMART Attributes Data Structure revision number: 16
Vendor Specific SMART Attributes with Thresholds:
ID# ATTRIBUTE_NAME          FLAG     VALUE WORST THRESH TYPE      UPDATED  WHEN_FAILED RAW_VALUE
  1 Raw_Read_Error_Rate     0x002f   200   200   051    Pre-fail  Always       -       0
  3 Spin_Up_Time            0x0027   174   171   021    Pre-fail  Always       -       2300
  4 Start_Stop_Count        0x0032   100   100   000    Old_age   Always       -       20
  5 Reallocated_Sector_Ct   0x0033   200   200   140    Pre-fail  Always       -       0
  7 Seek_Error_Rate         0x002e   200   200   000    Old_age   Always       -       0
  9 Power_On_Hours          0x0032   091   091   000    Old_age   Always       -       6688
 10 Spin_Retry_Count        0x0032   100   253   000    Old_age   Always       -       0
 11 Calibration_Retry_Count 0x0032   100   253   000    Old_age   Always       -       0
 12 Power_Cycle_Count       0x0032   100   100   000    Old_age   Always       -       20
192 Power-Off_Retract_Count 0x0032   200   200   000    Old_age   Always       -       10
193 Load_Cycle_Count        0x0032   200   200   000    Old_age   Always       -       19
194 Temperature_Celsius     0x0022   107   091   000    Old_age   Always       -       36
196 Reallocated_Event_Count 0x0032   200   200   000    Old_age   Always       -       0
197 Current_Pending_Sector  0x0032   200   200   000    Old_age   Always       -       0
198 Offline_Uncorrectable   0x0030   200   200   000    Old_age   Offline      -       0
199 UDMA_CRC_Error_Count    0x0032   200   200   000    Old_age   Always       -       0
200 Multi_Zone_Error_Rate   0x0008   200   200   000    Old_age   Offline      -       0

SMART Error Log Version: 1
No Errors Logged

SMART Self-test log structure revision number 1
No self-tests have been logged.  [To run self-tests, use: smartctl -t]


SMART Selective self-test log data structure revision number 1
 SPAN  MIN_LBA  MAX_LBA  CURRENT_TEST_STATUS
    1        0        0  Not_testing
    2        0        0  Not_testing
    3        0        0  Not_testing
    4        0        0  Not_testing
    5        0        0  Not_testing
Selective self-test flags (0x0):
  After scanning selected spans, do NOT read-scan remainder of disk.
If Selective self-test is pending on power-up, resume after 0 minute delay.

vadee@vadee:~$ 

Hors ligne

#18 Le 21/09/2016, à 20:32

grandtoubab

Re : un démarrage qui dure 25 min [résolu]

vadee@vadee:~$ systemd-analyze blame
systemd-analyze : commande introuvable

alors

pstree

Linux tout seul sur HP Pavilion DV7 et Acer Aspire T650, Canon MG3650 en wifi
Debian 11 Bullseye Gnome/Xorg, Gnome/Wayland avec SDDM
https://bidouilledebian.wordpress.com/
ON M'A VU DANS LE VERCORS, SAUTER A L'ELASTIQUE..... J'AI DANS LES BOTTES DES MONTAGNES DE QUESTIONS....

Hors ligne

#19 Le 21/09/2016, à 20:33

goliath60

Re : un démarrage qui dure 25 min [résolu]

grandtoubab:

vadee@vadee:~$ pstree
init─┬─ModemManager───2*[{ModemManager}]
     ├─NetworkManager─┬─dhclient
     │                ├─dnsmasq
     │                └─3*[{NetworkManager}]
     ├─accounts-daemon───2*[{accounts-daemon}]
     ├─acpid
     ├─avahi-daemon───avahi-daemon
     ├─bluetoothd
     ├─colord───2*[{colord}]
     ├─cron
     ├─cups-browsed
     ├─cupsd
     ├─dbus-daemon
     ├─6*[getty]
     ├─irqbalance
     ├─kerneloops
     ├─lightdm─┬─Xorg───{Xorg}
     │         ├─lightdm─┬─init─┬─at-spi-bus-laun─┬─dbus-daemon
     │         │         │      │                 └─3*[{at-spi-bus-laun}]
     │         │         │      ├─at-spi2-registr───{at-spi2-registr}
     │         │         │      ├─bamfdaemon───3*[{bamfdaemon}]
     │         │         │      ├─dbus-daemon
     │         │         │      ├─dconf-service───2*[{dconf-service}]
     │         │         │      ├─evolution-calen───4*[{evolution-calen}]
     │         │         │      ├─evolution-sourc───2*[{evolution-sourc}]
     │         │         │      ├─firefox───50*[{firefox}]
     │         │         │      ├─gconfd-2
     │         │         │      ├─geoclue-master───2*[{geoclue-master}]
     │         │         │      ├─gnome-keyring-d───5*[{gnome-keyring-d}]
     │         │         │      ├─gnome-session─┬─compiz───8*[{compiz}]
     │         │         │      │               ├─deja-dup-monito───2*[{deja-du+
     │         │         │      │               ├─nautilus───3*[{nautilus}]
     │         │         │      │               ├─nm-applet───2*[{nm-applet}]
     │         │         │      │               ├─polkit-gnome-au───3*[{polkit-+
     │         │         │      │               ├─telepathy-indic───2*[{telepat+
     │         │         │      │               ├─unity-fallback-───2*[{unity-f+
     │         │         │      │               ├─update-notifier───3*[{update-+
     │         │         │      │               ├─zeitgeist-datah───6*[{zeitgei+
     │         │         │      │               └─3*[{gnome-session}]
     │         │         │      ├─gnome-terminal─┬─bash───pstree
     │         │         │      │                ├─gnome-pty-helpe
     │         │         │      │                └─4*[{gnome-terminal}]
     │         │         │      ├─gvfs-afc-volume───2*[{gvfs-afc-volume}]
     │         │         │      ├─gvfs-gphoto2-vo───{gvfs-gphoto2-vo}
     │         │         │      ├─gvfs-mtp-volume───{gvfs-mtp-volume}
     │         │         │      ├─gvfs-udisks2-vo───2*[{gvfs-udisks2-vo}]
     │         │         │      ├─gvfsd───{gvfsd}
     │         │         │      ├─gvfsd-burn───2*[{gvfsd-burn}]
     │         │         │      ├─gvfsd-fuse───4*[{gvfsd-fuse}]
     │         │         │      ├─gvfsd-metadata───{gvfsd-metadata}
     │         │         │      ├─gvfsd-trash───2*[{gvfsd-trash}]
     │         │         │      ├─hud-service───3*[{hud-service}]
     │         │         │      ├─ibus-daemon─┬─ibus-dconf───3*[{ibus-dconf}]
     │         │         │      │             ├─ibus-engine-sim───2*[{ibus-engi+
     │         │         │      │             ├─ibus-ui-gtk3───3*[{ibus-ui-gtk3+
     │         │         │      │             └─3*[{ibus-daemon}]
     │         │         │      ├─ibus-x11───3*[{ibus-x11}]
     │         │         │      ├─indicator-appli───{indicator-appli}
     │         │         │      ├─indicator-bluet───2*[{indicator-bluet}]
     │         │         │      ├─indicator-datet───5*[{indicator-datet}]
     │         │         │      ├─indicator-keybo───2*[{indicator-keybo}]
     │         │         │      ├─indicator-messa───3*[{indicator-messa}]
     │         │         │      ├─indicator-power───2*[{indicator-power}]
     │         │         │      ├─indicator-print───2*[{indicator-print}]
     │         │         │      ├─indicator-sessi───3*[{indicator-sessi}]
     │         │         │      ├─indicator-sound───3*[{indicator-sound}]
     │         │         │      ├─mission-control───2*[{mission-control}]
     │         │         │      ├─notify-osd───2*[{notify-osd}]
     │         │         │      ├─pulseaudio───3*[{pulseaudio}]
     │         │         │      ├─su-to-root───gksu─┬─sudo───gsmartcontrol───{g+
     │         │         │      │                   └─3*[{gksu}]
     │         │         │      ├─ubuntu-geoip-pr───2*[{ubuntu-geoip-pr}]
     │         │         │      ├─unity-panel-ser───2*[{unity-panel-ser}]
     │         │         │      ├─unity-settings-───3*[{unity-settings-}]
     │         │         │      ├─2*[upstart-dbus-br]
     │         │         │      ├─upstart-event-b
     │         │         │      ├─upstart-file-br
     │         │         │      ├─window-stack-br
     │         │         │      ├─zeitgeist-daemo───{zeitgeist-daemo}
     │         │         │      └─zeitgeist-fts─┬─cat
     │         │         │                      └─2*[{zeitgeist-fts}]
     │         │         └─{lightdm}
     │         └─2*[{lightdm}]
     ├─polkitd───2*[{polkitd}]
     ├─rsyslogd───3*[{rsyslogd}]
     ├─rtkit-daemon───2*[{rtkit-daemon}]
     ├─systemd-logind
     ├─systemd-udevd
     ├─udisksd───4*[{udisksd}]
     ├─upowerd───2*[{upowerd}]
     ├─upstart-file-br
     ├─upstart-socket-
     ├─upstart-udev-br
     └─whoopsie───2*[{whoopsie}]
vadee@vadee:~$ 

Hors ligne

#20 Le 21/09/2016, à 20:38

grandtoubab

Re : un démarrage qui dure 25 min [résolu]

premier process init
systemd n'est pas activé sur ta machine
si tu veux systemd

sudo apt-get install systemd
sudo apt-get install systemd-sysv

et reboot

Dernière modification par grandtoubab (Le 21/09/2016, à 20:39)


Linux tout seul sur HP Pavilion DV7 et Acer Aspire T650, Canon MG3650 en wifi
Debian 11 Bullseye Gnome/Xorg, Gnome/Wayland avec SDDM
https://bidouilledebian.wordpress.com/
ON M'A VU DANS LE VERCORS, SAUTER A L'ELASTIQUE..... J'AI DANS LES BOTTES DES MONTAGNES DE QUESTIONS....

Hors ligne

#21 Le 21/09/2016, à 20:43

goliath60

Re : un démarrage qui dure 25 min [résolu]

vadee@vadee:~$ sudo apt-get install systemd
[sudo] password for vadee: 
Lecture des listes de paquets... Fait
Construction de l'arbre des dépendances       
Lecture des informations d'état... Fait
Aucune version du paquet systemd n'est disponible, mais il existe dans la base
de données. Cela signifie en général que le paquet est manquant, qu'il est devenu obsolète
ou qu'il n'est disponible que sur une autre source
Cependant les paquets suivants le remplacent :
  systemd-services:i386 systemd-services

E: Le paquet « systemd » n'a pas de version susceptible d'être installée
vadee@vadee:~$ sudo apt-get install systemd-sysv
Lecture des listes de paquets... Fait
Construction de l'arbre des dépendances       
Lecture des informations d'état... Fait
Aucune version du paquet systemd-sysv n'est disponible, mais il existe dans la base
de données. Cela signifie en général que le paquet est manquant, qu'il est devenu obsolète
ou qu'il n'est disponible que sur une autre source

E: Le paquet « systemd-sysv » n'a pas de version susceptible d'être installée
vadee@vadee:~$ 

Hors ligne

#22 Le 21/09/2016, à 20:49

grandtoubab

Re : un démarrage qui dure 25 min [résolu]

goliath60 a écrit :
vadee@vadee:~$ sudo apt-get install systemd
[sudo] password for vadee: 
Lecture des listes de paquets... Fait
Construction de l'arbre des dépendances       
Lecture des informations d'état... Fait
Aucune version du paquet systemd n'est disponible, mais il existe dans la base
de données. Cela signifie en général que le paquet est manquant, qu'il est devenu obsolète
ou qu'il n'est disponible que sur une autre source
Cependant les paquets suivants le remplacent :
  systemd-services:i386 systemd-services

E: Le paquet « systemd » n'a pas de version susceptible d'être installée
vadee@vadee:~$ sudo apt-get install systemd-sysv
Lecture des listes de paquets... Fait
Construction de l'arbre des dépendances       
Lecture des informations d'état... Fait
Aucune version du paquet systemd-sysv n'est disponible, mais il existe dans la base
de données. Cela signifie en général que le paquet est manquant, qu'il est devenu obsolète
ou qu'il n'est disponible que sur une autre source

E: Le paquet « systemd-sysv » n'a pas de version susceptible d'être installée
vadee@vadee:~$ 

t'es dans une ancienne version d'Ubuntu la derniere stable est la 16.04

tu boot sur Command line: BOOT_IMAGE=/boot/vmlinuz-4.4.0-36-generic root=UUID=496e9007-632f-49ee-a0ce-7f2a5eb1a540 ro quiet splash vt.handoff=7
appuis sur la touche "masjuscule" pour faire apparaitre le menu grub
quand tu vois le menu grub appuis tout de suite sur la touche e

retire les mots quiet splash
appuis sur les touches indiquées pour continuer le booy
tu auras tous les messages a l'écran et tu verras ou se situe le blocage

Dernière modification par grandtoubab (Le 21/09/2016, à 20:57)


Linux tout seul sur HP Pavilion DV7 et Acer Aspire T650, Canon MG3650 en wifi
Debian 11 Bullseye Gnome/Xorg, Gnome/Wayland avec SDDM
https://bidouilledebian.wordpress.com/
ON M'A VU DANS LE VERCORS, SAUTER A L'ELASTIQUE..... J'AI DANS LES BOTTES DES MONTAGNES DE QUESTIONS....

Hors ligne

#23 Le 21/09/2016, à 20:59

grandtoubab

Re : un démarrage qui dure 25 min [résolu]

goliath60 a écrit :

ok
clic sur la l'engrenage en haut à droite (à côté de l'heure), puis "a propos de cet ordinateur" puis "installer les mises à jour"
c'est bon comme ça?

fais le en ligne de commande pour avir des traces


sudo apt-get update
sudo apt-get upgrade
sudo apt-get dist-upgrade

Linux tout seul sur HP Pavilion DV7 et Acer Aspire T650, Canon MG3650 en wifi
Debian 11 Bullseye Gnome/Xorg, Gnome/Wayland avec SDDM
https://bidouilledebian.wordpress.com/
ON M'A VU DANS LE VERCORS, SAUTER A L'ELASTIQUE..... J'AI DANS LES BOTTES DES MONTAGNES DE QUESTIONS....

Hors ligne

#24 Le 21/09/2016, à 21:07

goliath60

Re : un démarrage qui dure 25 min [résolu]

grandtoubab je suis en train de faire tes 3 dernière commandes.
est ce que je dois aussi faire

tu boot sur Command line: BOOT_IMAGE=/boot/vmlinuz-4.4.0-36-generic root=UUID=496e9007-632f-49ee-a0ce-7f2a5eb1a540 ro quiet splash vt.handoff=7
appuis sur la touche "masjuscule" pour faire apparaitre le menu grub
quand tu vois le menu grub appuis tout de suite sur la touche e

retire les mots quiet splash
appuis sur les touches indiquées pour continuer le booy
tu auras tous les messages a l'écran et tu verras ou se situe le blocage

Hors ligne

#25 Le 21/09/2016, à 21:22

grandtoubab

Re : un démarrage qui dure 25 min [résolu]

goliath60 a écrit :

grandtoubab je suis en train de faire tes 3 dernière commandes.
est ce que je dois aussi faire

tu boot sur Command line: BOOT_IMAGE=/boot/vmlinuz-4.4.0-36-generic root=UUID=496e9007-632f-49ee-a0ce-7f2a5eb1a540 ro quiet splash vt.handoff=7
appuis sur la touche "masjuscule" pour faire apparaitre le menu grub
quand tu vois le menu grub appuis tout de suite sur la touche e

retire les mots quiet splash
appuis sur les touches indiquées pour continuer le booy
tu auras tous les messages a l'écran et tu verras ou se situe le blocage

non pas maintenant
fais les mises a jour d'abord


Linux tout seul sur HP Pavilion DV7 et Acer Aspire T650, Canon MG3650 en wifi
Debian 11 Bullseye Gnome/Xorg, Gnome/Wayland avec SDDM
https://bidouilledebian.wordpress.com/
ON M'A VU DANS LE VERCORS, SAUTER A L'ELASTIQUE..... J'AI DANS LES BOTTES DES MONTAGNES DE QUESTIONS....

Hors ligne